You are on page 1of 138

UNIVERSITY OF KWAZULU-NATAL

Department of Mathematics at Howard College


LAPLACE TRANSFORM and DIFFERENTIAL EQUATIONS

2009

Preliminaries

Chapter Zero

PRELIMINARIES

1. Improper Integrals If x0 f (x) dx exists for every M , and limM dene


M M x0

f (x) dx also exists and is nite, then we


M

f (x) dx = lim

x0

f (x) dx.
x0

We also say the integral converges. It may be shown that


x0

f (x) dx

x0

|f (x)| dx;

if the integral on the right converges, then the integral on the left also converges and we say it converges absolutely. An absolutely convergent integral is always convergent, too; but the converse is not necessarily true. To establish whether an improper integral converges or diverges, one may sometimes use the comparison test: suppose |f (x)| |g(x)| from some point onwards (say, for x > a): then
a

|f (x)| dx =

=
a

|g(x)| dx =

(both integrals diverge). Vice-versa,


a

|f (x)| dx <

|g(x)| dx <

(both integrals converge absolutely). The comparison test is useful, for example, when it is relatively easy to integrate f (x) but not g(x), or vice-versa. 2. Integration by Parts The product rule for dierentiation is usually written as (uv) = u v + uv . By simple manipulations, we get immediately u v = (uv) uv . Integrating with respect to the independent variable (let us call it x), we get u v dx = uv uv dx. (1)

Preliminaries

Using denite integration instead, we get


b a

u v dx = u v

b a

u v dx.

(2)

Both formulas (1) and (2) are called integration by parts. 3. An Important Result Observe that
0

ex dx = ex

= (0 1) = 1.

Then consider

xex dx: integrating by parts, we get


0

xex dx = xex
0

ex dx.

We need to nd xex

= lim = lim

M . M eM

M eM 0e0 =

The limit on the right is of the form /, and de lHospitals theorem is applicable: one gets immediately that this limit is zero. Substituting back, it follows
0

xex dx = 0 +
0

ex dx = 1.

In the same way, using integration by parts, we see that


0

x2 ex dx = lim

M2 02 + 0 eM e

2xex dx.

By de lHospitals rule (applied twice) lim M2 = 0; eM


0

hence
0

x2 ex dx = 0 + 2

xex dx = 2 1.

More generally: if n is a positive integer, then applying de lHospitals theorem n times, we may show that Mn lim M = 0; M e in other words, the exponential function eM always diverges faster than any (arbitrarily large) power M n , as M tends to innity. This statement remains true even if eM is replaced by eM , where is positive and arbitrarily small. Exponential growth is inherently stronger than polynomial growth of any degree.

Preliminaries

Now, using these results, and repeating integration by parts n times, it is easy to see that
0

xn ex dx = 0 +
0

0 0

nxn1 ex dx = n(n 1)xn2 ex dx = n(n 1)(n 2)xn3 ex dx = etc. etc.

=0+ =0+ = n! , where n! = 1 2 3 n. 4. Eulers Formula

Euler studied the properties of the quantity w = cos + i sin , where i2 = 1. It is easy to verify that the magnitude and argument of w are, respectively |w| = 1, arg(w) = .

If two such expressions are multiplied together, one gets this identity: (cos 1 + i sin 1 )(cos 2 + i sin 2 ) = cos(1 + 2 ) + i sin(1 + 2 ) (verify this). Following Euler, we dene the exponential of an imaginary quantity in the following way: def ei = cos + i sin . It is easy to see that this denition extends to imaginary exponentials all properties established for real exponentials: for example ei = 1 , ei ei1 ei2 = ei(1 +2 ) , d eic = iceic , d

and so on. The following formulas are also very useful and should be memorized: cos = ei + ei 2 sin = ei ei . i2

As an exercise, use Eulers formula to derive the identities cos A cos B = 1 [cos(A + B) + cos(A B)], 2 which you saw in high school. Euler did not call it a complex quantity, as we should have done, because complex numbers had not yet been properly dened. In a sense, he was ahead of his times.
1 sin A cos B = 2 [sin(A + B) + sin(A B)],

The Laplace Transform

Chapter One

THE LAPLACE TRANSFORM

1. Introduction Denition: Suppose f (t) is a given function of t, and the integral


0

est f (t) dt

(3)

converges for some value of the parameter s. Then the integral (3) is called the Laplace Transform of f (t) and is denoted either as L [f (t)] or F (s). Example 1 If f (t) = 3t + 4, L [3t + 4] =
0

(3t + 4) est dt.

If s > 0, by substituting st = x (so that t = + corresponds to x = +) we get immediately L [3t + 4] = 3 s2


0

xex dx +

4 s

ex dx =

4 3 + . 2 s s

[s > 0]

On the other hand, if s 0, then t = + is mapped into x = ; we get a divergent integral, hence the Laplace transform of 3t + 4 is not dened for negative s. Example 2 If f (t) = 7e2t , then L 7e2t =
0

7e2t est dt =

7e(2s)t 2s

7 . s2

Note that in this example the Laplace transform is dened only for s > 2. Example 3 Find L [f (t)], if f (t) = e7t + 3 . Solution: Expanding the square, we get e7t + 3 L [f (t)] =
0 2

= e14t + 6e7t + 9, and hence 1 6 9 + + . s 14 s 7 s

e14t + 6e7t + 9 est dt =

Note that in this example the Laplace transform is dened only for s > 14.

The Laplace Transform

Solution: One has immediately

Example 4 Find L [f (t)], if f (t) = 2 for 4 t 7; f (t) = 0 everywhere else.


7

L [f (t)] =

2est dt =
4

2est s

=
4

2e4s 2e7s . s

Comment: A function like f (t) in this example, is called a transient because it comes to life, so to speak, when t = 4, at which point it jumps to the value 2 (here we have a discontinuity); when t is increased beyond the point t = 7 (another discontinuity) f (t) vanishes for good. Before we begin to study the properties of the Laplace transform, which will take a fair amount of time, let us make some preliminary remarks. The integration variable is commonly called t, rather than x. This is because in most applications, t physically represents time. There may be exceptions to this rule. The Laplace transform denes a correspondence between functions of t and functions of s. Such a correspondence is a linear map: in other words, given two functions of t, f1 and f2 , and two constants c1 and c2 , then L c1 f1 (t) + c2 f2 (t) = c1 L f1 (t) + c2 L f2 (t) . (linearity)

We assume that s is real. This keeps the foregoing discussion as simple as possible; but be warned, certain properties of the Laplace transform are easier to study if s is treated as a complex variable. If 0 est f (t) dt converges absolutely for a certain s, then it converges absolutely for all values of s greater than that. This follows from the simple observation that if s > s0 , then |est | < |es0 t | for all positive t. If two functions, say f (t)and g(t) are identical for t 0, then clearly they have the same Laplace transform; their behavior for negative t is irrelevant. So, with no loss of generality, we may assume f (t) = 0 identically for negative t. For the integral (3) to converge for some s, it is sucient that |f (t)| be integrable to the right of the origin, and do not diverge faster than an exponential as t . We have already noted that all powers of t, and hence all polynomials, meet this requirement. 2. Notation. Uniqueness The function f (t) appearing in (3) is sometimes called the direct function or pre-image; the transform itself is usually called the image of f (t). Following an established tradition, we shall use the same letter of the alphabet for the direct function and for its transform, with the understanding that small letters such as f , g, x, y, etc. will be used for direct functions, and the corresponding capital letters F , G, X, Y , etc. for their transforms. Occasionally we shall also need Greek letters for dummy variables: in this case, recall that (tau) and (sigma) are the Greek equivalent of small t and small s, respectively. Derivatives with respect to t will generally be denoted by dots (Newtons notation), as it is common in physics and engineering. Derivatives with respect to s will be denoted by the more Warning: some books use small letters for transforms and capital letters for direct functions, which can be very confusing.

The Laplace Transform

usual primes. For example: x= dx , dt y= d2 y ; dt2

(F G) = F G + 2F G + F G =

dF dG d2 G d2 F G+2 +F . ds2 ds ds ds2

We conclude these preliminaries with a rather subtle point. It is obvious that if f (t) g(t), then F (s) = G(s). But it is not obvious at all whether from F (s) = G(s) one may deduce that f (t) = g(t) anywhere. In a sense, this is blatantly false, since f and g may take arbitrary values for t < 0; and that is precisely the reason why we stipulated that direct functions be re-dened to be identically zero for negative t. The question of equality of the original functions, when the transforms are equal, is the object of Lerchs theorem, which says that under very reasonable assumptions, if F (s) = G(s) then f (t) = g(t) for all positive t, except at most a nite number of isolated points. From the point of view of applications, Lerchs theorem says that if F (s) = G(s) then for all practical purposes f (t) = g(t). 3. Basic Transforms The following transforms must be committed to memory. L [ect ] = L [cosh t] = L [sinh t] = L [cos t] = L [sin t] = L [tc ] = The proof of (4) is straightforward: L [ect ] =
0

1 sc s 2 2 s 2 2 s s 2 + 2 s s2 + 2 c! . sc+1

[s > c] [s > ||] [s > ||] [s > 0] [s > 0] [s > 0, c > 1]

(4) (5) (6) (7) (8) (9)

ect est dt =

e(cs)t cs

=0

1 1 = , cs sc

as long as s > c. The proofs for (5) and (6) are corollaries: L [cosh t] = L L [sinh t] = L
1 2 1 2

et + et et et

1 1 s + = 2 , 2(s ) 2(s + ) s 2 1 1 = = 2 . 2(s ) 2(s + ) s 2 =

In order to prove (7) and (8), we may use Eulers formula: eit = cos t + i sin t. Maty Lerch (18601922), Czech. as (10)

The Laplace Transform

We have then by (4): L [eit ] = s + i 1 = 2 . s i s + 2

Now (by denition) L [cos t] and L [sin t] are certainly real: hence, separating real and imaginary part in the last expression, we get L [cos t] = Re s + i s ; = 2 2 + 2 s s + 2 s + i . = 2 L [sin t] = Im 2 s + 2 s + 2

Example 5 Find the Laplace transform of sin3 10t. Solution: First of all, we must put sin3 10t into a simple form, without powers. This may be done using the identities you learnt in high school, but perhaps the best way is by Eulers formula (10), which gives eit eit sin t = . i2 It follows 3 ei10t ei10t 3 ; sin 10t = i2 expanding the cube, we get sin3 t = ei30t 3ei10t + 3ei10t ei30t = i8 ei30t ei30t ei10t ei10t = 3 = i4 2 i4 2 3 1 = 4 sin 30t + 4 sin 10t.

From this identity, and formula (8), we get immediately that


1 L sin3 10t = 4

s2

30 + + 900

3 4

s2

10 . + 100

Example 6 Find the Laplace transform of sin 2t cos 9t. Solution: Simple manipulations (or Eulers formula) give sin 2t cos 9t = it follows immediately that L [sin 2t cos 9t] = s2 7/2 11/2 2 . + 121 s + 49
1 2

sin 11t

1 2

sin 7t;

The proof of (9) is bit more involved and requires a new concept, which is introduced in the next section. You will nd that, in applications, the few basic transforms (49) listed above are often all that one needs to know. Just like we seldom calculate derivatives as a limit of the form

The Laplace Transform

f (x + h) f (x) /h, but rather through the rules of dierential calculus, so there exists a Laplace calculus which allows us to calculate many Laplace transforms without going back to the denition (3). 4. The Factorial Function Consider L [tn ], n being a positive integer. We substitute st with x in the integral (3) (if s is positive, this maps t = + into x = +), and then integrate by parts n times: L [tn ] =
0

tn est dt =

1 sn+1
0

xn ex dx =

n! sn+1

[s > 0]

Essentially, we have repeated the procedure outlined in section 0.3 . It works because we started with an integer power tn ; but what about, for example, t, or more general powers of t? For positive s, integrating by parts once, we get L t =
0

t1/2 est dt =

1 s3/2
0

x1/2 ex dx = 0

1 s3/2
0

1 1/2 x 2

ex dx.

The integral on the right converges, but cannot be done analytically; another round of integra tion by parts leads nowhere, because 0 x3/2 ex dx is divergent (convince yourself of this). Integration by substitution is also useless; it has been shown that there is no change of variables that resolves this integral as a combination of elementary functions (polynomials, sine/cosine etc.). However, such an integral certainly f(x) exists. As the picture shows,
0

x1/2 ex dx = A,

0.4

where A is the area A below the graph of f (x) = x1/2 ex

0.2

and above the x axis, in the whole rst 0 x 0 2 4 6 8 quadrant. Its easy to show that even though this region is unbounded, its area is nite. In other words, A may be calculated with arbitrary precision (using a computer program, why not?). It may therefore be used to extend the denition of factorial that you learnt in rst year. Denition: If c is a real number greater than 1, we dene the generalized factorial of c by means of the expression c! =
0

xc ex dx.

(11)

If c is a positive integer, the integral on the right may be done by parts and is found to be equal to the product 1 2 3 c. This is all we need to conclude our proof of (9): for s > 0 we have immediately: L [tc ] =
0

tc est dt =

1 sc+1
0

xc ex dx =

c! sc+1

[c > 1]

The Laplace Transform

where c! is dened in (11). As an exercise, convince yourself that 0 xc ex dx = if c 1. Factorials possess a simple and useful recursive property: if c > 1, then (c + 1)! = (c + 1) c! (12)

If c is a positive integer, this follows immediately from the old denition of factorial. If c is an arbitrary real number greater than 1, then integrating by parts we get (c + 1)! =
0

xc+1 ex dx = xc+1 ex

+
0

(c + 1)xc ex dx.

The integrated part is zero: by assumption c + 1 > 0, hence


x0

lim xc+1 = 0;

also (see section 0.3),


x

lim xc+1 ex = 0.

It follows (c + 1)! = 0 +
0

(c + 1)xc ex dx = (c + 1) c! ,

which is (12), as required to prove. An important consequence of (12) is that once a table of generalized factorials has been calculated for any interval of unit length, it may be extended outside such an interval using ordinary multiplications. Example 7 Calculate 5.3273! , assuming that a table of c! is available only for 0 c 1. Solution: By (12), we may write 5.3273! = 5.3273 . 4.3273!. Repeating this procedure four more times, we get 5.3273! = 5.3273 . 4.3273 . 3.3273 . 2.3273 . 1.3273 . 0.3273! and at this point we look up 0.3273! in the table. There, we nd that 0.3273! 0.89371; therefore 5.3273! 211.7553, which lies between 5! = 120 and 6! = 720. We told you a half-lie. You probably thought that the only case when c! may be calculated in an easy way, is if c is a positive integer or zero. Whether this statement is true or false, depends on what you mean by easy. As it happens, using the Laplace transform it is possible to calculate c! when c is half-odd; we shall come back to this in chapter 2, where it is shown that
1 ( 2 )! =

From this, one gets immediately, using (12):


1 2! 3 2! 5 ! 2 7 2! 9 2!

1 ( 1 )! = 2 2 3 1! = 4 2 3 ! = 15 2 8 = 5 ! = 105 2 16 = etc. = = =
1 2 3 2 5 2 7 2

10

The Laplace Transform

Finally, a word of warning. Most books still introduce generalized factorials by means of Legendres so-called gamma function; that is, they write (c + 1) for c! . That extra +1 is there only for historical reasons, and is as necessary as the human appendix. Today, as the best computer programs (such as maxima, maple, mathematica, etc.) accept both notations, there is no need for us to persist with the bulkier gamma notation. Most important, the exclamation mark is a good reminder of the link with ordinary factorials. 5. Basic Properties; Multiplication by s Before we begin to study the analytic properties of the Laplace transform, we must make some assumptions on the function f (t) that appears in the denition (3). Clearly, it is not necessary that f (t) be continuous for the integral in (3) to converge: the function in example 4, for instance, is discontinuous. On the other hand, it is easy to produce functions with no Laplace transform 2 for any value of s: for example, f (t) = et or f (t) = 1/t. There is no special term in the mathematical literature describing a function that admits a Laplace transform, but we can easily do without it. However, we introduce now a couple of denitions that will help us through our discussion. Denition: A function f (t) is said to be sectionally continuous if it is continuous everywhere except at a nite number of discontinuities, and at each point of discontinuity both the limit from the right and the limit from the left, of f (t), exist and are nite. Example 8 Let f (t) = t for t [0, 1]; f (t) = t2 for t (1, ). Then f (t) is continuous for every t; f(t) is only sectionally continuous, because f(t) = 1 in (0, 1), f(t) = 2t in (1, ), limt1 f(t) = 1, limt1+ f(t) = 2. At t = 1, which is a point of discontinuity, the limit from the left and the limit from the right of f are dierent, but both are nite. Denition: A function f (t) is of exponential order if there exist two positive constants c and B such that |f (t)| < B ect , for all t from a certain point onwards. These denitions are broad enough to include most functions of practical relevance. Clearly, if a function is sectionally continuous and of exponential order, then the integral in (3) converges, and hence its Laplace transform existsfor suciently large s, possibly. It also follows that, if f (t) is of exponential order, and s is large enough, then
t

lim |f (t)|est = 0

(i.e., this is true for all s greater than some constant ). Suppose now that f (t), f(t) are of exponential order; f (t) is continuous and f(t) sectionally continuous. By denition, L [f(t)] = Integrating by parts, we get L [f(t)] = f (t) est
0

(df /dt) est dt.

f (t)(sest ) dt.

Quoted from G. Arfken, Mathematical Methods for Physicists, Wiley (USA) 1970.

The Laplace Transform

11

This may also be written: L [f(t)] = lim f (t) est lim f (t) est + s
t t0+ 0

f (t) est dt.

Since we assume that f (t) is of exponential order, the rst of the limits above is zero (for suciently large s). The second limit is equal to limt0+ f (t), i.e., the limit of f (t) as t approaches zero from the rightremember, by denition f (t) 0 if t is negative. We write
t0+

lim f (t) = f0 .

It follows immediately that L [f(t)] = f0 + sF (s). Example 9 Find the function x(t) such that 5x + 3x = 0 and x0 = 2. Solution: We take the Laplace transform of the equation. We get L [5x + 3x] = 0, where L [x] = X(s), and L [x] = x0 + sX(s) = 2 + sX(s). It follows 5(2 + sX) + 3X = 0, which yields X= Now, by inspection, we recognize that 1 = L e3t/5 ; s + 3/5 Therefore, it follows immediately that X = L 2e3t/5 , and nally (by Lerchs theorem) x = 2e3t/5 . 10 2 = . 5s + 3 s + 3/5 (13)

Transforms of higher-order derivatives may be calculated by repeated application of (13). For example, if f and f are continuous, and f is sectionally continuous, then L [f (t)] = f0 + sL [f(t)], where f0 denotes limt0+ f(t). Substituting for L [f(t)] its expression (13), we get L [f (t)] = f0 + s(f0 + sF (s)) = = f0 sf0 + s2 F (s). (14)

12

The Laplace Transform

Similarly, L f (t) = f0 + sL [f (t)]) = = f0 sf0 s2 f0 + s3 F (s),

(15)

where f0 denotes limt0+ f (t). Naturally, here, we must assume that f , f and f are continuous, and the third derivative of f (t) is sectionally continuous. Formulas for higher-order derivatives may be calculated in the same way, if needed. However, in each case, the assumptions must be extended: the highest derivative of f (t) must be sectionally continuous, and all lower-order derivatives (including f ) must be continuous. In typical engineering applications, these assumptions are generally satised. Example 10 Calculate L cos2 t with and without (13).

Solution: Without using (13), one may nd L cos2 t using the identity cos2 t = 1 (cos 2t + 1). 2 This gives 1 L [cos2 t] = 2 L [cos 2t] + 1 L [1] 2 =
1 2

s 1 + . s2 + 4 s

In order to use (13), we rst note that f0 = 1 and f = 2 sin t cos t = sin 2t. It then follows that 2 L [f ] = 2 , s +4 and, by (13), that L [f ] = sL [cos2 t] 1. By comparison, one gets that sL [cos2 t] 1 = which nally yields L [cos2 t] = Since
1 2

2 , s2 + 4

2 1 . 2 + 4) s s(s

s2

s 1 + +4 s

1 2 s2 + 2 = , 2 + 4) 2 + 4) s(s s s(s

the two results are identical.

This example works because cos2 t is continuous throughout, and so are its derivatives of any order. But the continuity requirements are crucial, as the following example shows. Example 11 Find F (s), if f (t) = t for 0 t < 2, and f (t) = 0 everywhere else.

Solution: Before we start: if you apply (13), you get the wrong answer. Let us see why. The graph of f (t) is pictured on the right; note the discontinuity at t = 2. It follows that

The Laplace Transform

13

f = Hence, by (3), L f =

1 0

for 0 t < 2 everywhere else.

f(t)

2 0

1 est dt =

1 e2s . s
0 2
2

Since f0 = 0, if you applied (13) you would get that L [f ] = (1 e Working from (3), and integrating by parts, we get
2

2s

)/s . But this is wrong!

L [f ] =

test dt =
0

test s

+
0

1 s

2 0

est dt =

1 e2s 2e2s + . s s2

This is the correct answer. Formula (13) may not be used because f (t) is not continuous at t = 2. Next example is very similar, except for a small detail: f (t) is continuous and f(t) is sectionally continuous. Therefore (13) is applicable. Example 12 Find F (s) if f (t) = 3t for 0 t 2; f (t) = 6 for t > 2. Solution: As the picture shows, f (t) is continuous. Observe that f = 3 0 if 0 t 2 everywhere else.

f(t) 6

Also, f(t) is sectionally continuous. By (3), we nd that L [f ] =


2

3est dt =
0

3(1 e2s ) . s
0 2

Since f is continuous and f sectionally continuous, (13) is applicable. Therefore, L [f ] = f0 + sF (s) = But f0 = 0, hence F (s) = 3(1 e2s ) . s

3(1 e2s ) . s2

On the other hand, one may calculate the transform from (3), integrating by parts:
2

F (s) =
0

3test dt +
2

6est dt

3(2s + 1)e2s 6e2s 3 + . = 2 s s2 s Its easy to see that the nal result is the same.

14

The Laplace Transform

6. Division by s It follows from the fundamental theorem of calculus that d dt


t

f ( ) d = f (t).
0

The integral on the left-hand side is an anti-derivative of f (t). We now call it g: g(t) = and observe that g(t) = f (t), Taking the Laplace transform of the identity L [g(t)] = L [f (t)] and using (13), we get g0 + sG(s) = F (s). But g0 = 0. Hence, G(s) = which may nally be expanded as
t def 0 t

f ( ) d,

g(0+) = 0.

F (s) , s

f ( ) d =
0

F (s) . s

(16)

Naturally, this formula may be iterated any number of times. For example, if we integrate between zero and t the function g dened at the beginning of this section, we get another function of t, let us call it h, such that = f (t), and h(0) = h(0) = 0; therefore L [h(t)] = F (s)/s2 . h(t) Then, integrating h between zero and t and reasoning in the same way, we obtain yet another function of t, and its Laplace transform will be F (s)/s3 . And so on: the process may be repeated as many times as needed. Example 13 Find the function x(t) such that 2x + 4x = 2 and x0 = 1. Solution: We take the Laplace transform of the equation. We get L [2x + 4x] = L [2], where L [x] = X(s), L [x] = x0 + sX(s) = 1 + sX(s), and L [2] = 2/s. It follows 2(1 + sX) + 4X = 2 , s

Be careful: the dummy variable of integration must not be t, because it appears as a limit of integration.

The Laplace Transform

15

whichsolving for Xyields 1 1 2 1 + . 2+ = 2s + 4 s s + 2 s(s + 2) Now, by inspection, we recognize that 1 = L e2t ; s+2 the second term may be simplied by rule (16): X=
t 1 e2 d = L =L s(s + 2) 0 Combining the two terms, it follows immediately that 1 2

1 e2t . 2
1 2 1 + 2 e2t ,

and nally (by Lerchs theorem) x = Example 14 Since

X = L e2t + L
1 2

1 2t 1 2 2e 1 2t . 2e

=L

tn =
0

n n1 dt, nL [tn1 ] . s est dt = 1 , s L [t4 ] = 4! , s5

we get immediately L [tn ] = In this way, starting from L [1] = we get L [t] =
0

1 2! 3! , L [t2 ] = 3 , L [t3 ] = 4 , 2 s s s and so on. We recover (9), though for integer powers only.

These transforms may also be understood in terms of convolution, a more advanced technique that well study is section 2.7. 7. Multiplication by t It may be shown that if the integral (3) converges absolutely, then F (s) may be dierentiated any number of times with respect to s. For example, d F (s) = f (t) test dt. f (t) est dt = f (t) est dt = ds 0 s 0 0 Repeating this process n times, we obtain dn F (s) . (17) L [tn f (t)] = (1)n dsn Example 15 Find L [t cosh 4t]. Solution: Since L [cosh 4t] = s/(s2 16), then by (17) L [t cosh 4t] = As an exercise, calculate same answer.
0

t cosh 4t est dt (integrating by parts) and verify that you get the

d s 1 2s2 s2 + 16 = 2 + 2 = 2 . ds s2 16 s 16 (s 16)2 (s 16)2

16

The Laplace Transform

Example 16 Find L [t4 ect ].

Solution: We have from (4): L [ect ] = 1/(s c). Hence, by (17): L [t4 ect ] = d ds
4

1 4! = . sc (s c)5

Example 17 Find L [teit ] and hence L [t cos t], L [t sin t]. Solution: From (4) we get L [eit ] = 1/(s i). It follows L [teit ] = 1 1 (s + i)2 s2 + i2s 2 d = = 2 = . ds s i (s i)2 (s + 2 )2 (s2 + 2 )2

Separating real and imaginary parts, it follows immediately L [t cos t] = s2 2 ; (s2 + 2 )2 2s L [t sin t] = 2 . (s + 2 )2
st e t cos t dt 0

As an exercise, calculate from rst principles that you get the same results.

or

st e t sin t dt, 0

and verify

Example 18 Find the function x(t) such that 2x + x = t3 et/2 , and x0 = 0. Solution: Taking the Laplace transform of the equation, we get 2L [x] + L [x] = L t3 et/2 . writing L [x] = X and using (13) we get immediately L [x] = x0 + sX = sX. Then, we note that L et/2 = 1 s + 1/2 = L t3 et/2 = d3 dt3 1 s + 1/2 = 3! ; (s + 1/2)4

this step (multiplication by t3 ) follows from (17). Substituting these expressions into the preceding equation yields: 6 . 2sX + X = (s + 1/2)4 Hence X= Now, we observe that 3 6 = . (2s + 1)(s + 1/2)4 (s + 1/2)5 d4 dt4 1 s + 1/2 4! ; (s + 1/2)5

The Laplace Transform

17

therefore,

3 3 d4 = (s + 1/2)5 4! dt4

1 s + 1/2

1 8

L t4 et/2 ;

this step (multiplication by t4 ) also follows from (17). Hence, X=L and nally, by Lerchs theorem,
1 x = 8 t4 et/2 1 4 t/2 8t e

is the required function. 8. Division by t If f (t) is of exponential order, then certainly lim
0

|f (t)| est dt = 0.

In other words, if a function of s does not approach zero as s , then it is not a Laplace transform, in the ordinary sense of the word. Given a function f (t) with a Laplace transform F (s), we introduce G(s) =
def s

F () d.

Note that is a dummy variable; s is the lower limit of integration and we treat it as a parameter. Convince yourself that this integral is dened in such a way that if s , then G(s) tends to zero. Therefore, G(s) may be the Laplace transform of some function g(t). By the fundamental theorem of calculus, G(s) is an anti-derivative of F (s): G (s) = F (s) = L [f (t)]. On the other hand, it follows from (17) that G (s) = L [tg(t)]. By comparison, we get f (t) = tg(t), which may be written g(t) = f (t) . t

Finally, tranforming again, we get G(s) = L [f (t)/t], which may be expanded as L f (t) = t
s

F () d.

(18)

This comment does not apply to generalized functions, which youll nd in more advanced courses. But generalized functions are not functions in the usual sense of the word.

18

The Laplace Transform

Example 19 Find L [sin t/t]. Solution: From (8) we get L [sin t] = 1/(s2 + 1). Hence, it follows from (18) that L sin t = t
s

d = arctan +1

1 = 2 arctan s = arccot s.

Comment: By denition, L [sin t/t] =


0

(sin t/t) est 0

dt. So, we get the interesting result [s > 0]

sin t st e dt = 1 arctan s. 2 t

For example, for s = 1 we get

t (e 0

1 1 sin t/t) dt = 2 arctan 1 = 4 . Letting s 0+ we get 0

sin t dt = 1 , 2 t

which is a famous example of a convergent improper integral that is not absolutely convergent. Note also that these integrals may not be done using elementary calculus, as the anti-derivatives may not be written in terms of elementary functions. We nd the surprising result that denite integration is elementary while indenite integration is not. Example 20 Using the result of example 19, nd G(s), given that g(t) = Solution: In example 19 we established that L sin t = arccot s. t
t 0

(sin / ) d .

Hence, applying (16) we get immediately that


t

L
t

sin arccot s d = . s

Comment: the function g(t) = 0 (sin / ) d is called integral sine, denoted si(t), and is used in many engineering applications. Example 21 Find L [(et 1)/t]. Solution: From (4) and (9) we get, respectively, L [et ] = 1/(s 1) and L [1] = 1/s. It follows that 1 1 et 1 = d = L t 1 s s 1 s1 = ln . = ln 1 ln = ln s s1 s Again, although the indenite integral 0 est (et 1)/t dt cannot be done in closed form, the Laplace transform has been found by means of (18). In principle, rule (18) may be iterated: division of f (t) by t2 corresponds to integrating F (s) twice, and so on. In practice, examples of this kind are likely to be fairly long. You may nd

The Laplace Transform

19

one at the end of this chapter (see section 1.11). Next example shows how double integration may, in some cases, be avoided. Example 22 Find the Laplace transform of sin2 t/t and hence the Laplace transform of sin2 t/t2 . Solution: The rst half of this problem is very simple: from the identity sin2 t = 1 (1 cos 2t) 2 we get immediately that L sin2 t = and hence that sin2 t 1 L = t 2
s

1 2

s 1 2 , s s +4 d =
1 2

1 2 +4

ln

s2 + 4 . s

Naturally, for this step we applied equation (18). For the second part, we could use (18) again, but the following shortcut is better. If we dene f (t) = then the preceding result may be written F (s) = Dierentiating f (t) we get that sin 2t sin2 t 2 sin t cos t sin2 t 2 = 2 ; f = t t t t Note that f (0+) = 0. It follows that sin 2t sin2 t = f 2 t t and hence, by (13), that sin2 t sin 2t sin 2t L =L s F (s) f (0+) = L 1 s ln 2 2 t t t s2 + 4 . s
1 2

sin2 t : t

ln

s2 + 4 . s

The rst term on the right-hand side is almost identical to what we found in example 19: L So, nally, we get: sin2 t 1 = arccot(s/2) 2 s ln L t2 sin 2t = arccot(s/2). t s2 + 4 . s

20

The Laplace Transform

OVERVIEW The rules discussed in sections 58 may, at rst, seem hard to grasp. They are not: let us put them all together in a table. f (t) f(t)
t

F (s) sF (s) f0 F (s) s F (s)


s

f ( ) d
0

t f (t) f (t) t

F () d

The rst line is simply a reminder that we use lower-case letter to denote functions of t, and the corresponding upper-case letters to denote their transforms. You should be able to spot the thread linking the other formulas: Dierentiation with respect to one variable, t or s, is associated with multiplication by the other; integration with respect to one variable is associated with division by the other. We are slightly over-simplifying, now; some ne points must also be borne in mind. For example, integration with respect to s ends at innity, whereas integration with respect to t starts at zero. The rule for L [f ] must account for the possibility that f (t) be discontinuous at t = 0, hence f0 appears in the second line; on the other hand, the rule for F (s) has a minus sign, which should not be overlooked. However, the common thread (in italics above) is easy to remember. Using the six basic transforms (49) as building blocks and combining the rules above, one may derive a wide variety of transforms, with no direct call for integral calculus. We now complement this table with two more basic rules: the shift rules. 9. Shifting s Consider a function f (t) with Laplace transform F (s). Let be a constant. By denition, L et f (t) = It follows immediately L et f (t) =
0 0

f (t) et est dt.

f (t) e(s)t dt =

(19) Example 23 Find L e4t t . Solution: By (9) we get that t = ( 1/2)! s3/2 . Therefore, L e4t t = ( 1/2)! (s + 4)3/2 . Example 24 Find L e5t cos 3t . Solution: Since L [cos 3t] = s/(s2 + 9), we get immediately that L e5t cos 3t =

= F (s + ).

s+5 s+5 = 2 . 2+9 (s + 5) s + 10s + 34

The Laplace Transform

21

Example 25 Find L t3 e2t . Solution: Since L [t3 ] = 6/s4 , we get immediately by (19) that L t3 e2t = 6 . (s 2)4

Note, however, that one may also start from L [e2t ] = 1/(s 2) and apply (17) three times: L t3 e2t = Example 26 Find L [sin 2t cosh 3t]. Solution: By denition, sin 2t cosh 3t = sin 2t e3t + e3t 2 = d3 ds3 1 s2 = 6 . (s 2)4

1 = 1 e3t sin 2t + 2 e3t sin 2t. 2

It follows immediately that L [sin 2t cosh 3t] =


t

1 1 + 2+4 (s 3) (s + 3)2 + 4

Example 27 Find F (s), if f (t) = t1 0 e3 sin d . Solution: We observe that f (t) is obtained by performing three operations on the function sin t, which are: (i) multiplication by e3t , (ii) integration with respect to t, (iii) division by t. The corresponding operations on F (s) are: (i) shift by 3 units to the left (ii) division by s, (iii) integration with respect to s. Proceeding along these lines, we get: 1 , +1 1 L e3t sin t = , (s + 3)2 + 1 L [sin t] = s2
t

(i) (ii) and nally: (iii) F (s) = L 1 t L

e3 sin d =

1 , s[(s + 3)2 + 1]

e3 sin d =
0 s

d . [( + 3)2 + 1]

22

The Laplace Transform

A simple substitution yields


s

d = [( + 3)2 + 1]

s+3

dx . (x 3)(x2 + 1)
1 10 (x + 3) , x2 + 1

It is possible to show that


1 1 = 10 (x 3)(x2 + 1) x3

as you can easily verify. The expansion above may be done by the methods you learnt in rst year, but well come back to this subject in the next chapter. So, integrating, we get: F (s) =
1 x+3 1 2 dx = 10 s+3 x 3 x + 1 1 1 ln |x 3| 2 ln |x2 + 1| 3 arctan x = = 10 s+3 1 x3 = ln = 3 arctan x 10 x2 + 1 s+3 s 1 ln 1 3 ln + 3 arctan(s + 3) . = 10 2 (s + 3)2 + 1

Simplifying, we nd that F (s) = 10. Shifting t We have discussed the the eects of a shift of the s axis. A shift of the t axis may be studied in a similar way. This may be useful, for instance, if the function f (t) is dened by dierent formulas over dierent pieces of the t axis. In such a case, the following approach may be useful. Suppose f (t) = 0 if t < T , (t T ) if t T ,
T T 1 10

ln

(s + 3)2 + 1 s

3 10

arccot(s + 3).

where (t) is a function of t having Laplace transform (s). Then, by denition,


T

F (s) =
0

f (t) est dt +

f (t) est dt =

=0+ Substituting t = T + , we get F (s) =

(t T ) est dt.

0 sT

( )esT s d = (s). (20)

=e

The Laplace Transform

23

Example 28 Find L [f (t)], if f (t) =

0 sin 2t Solution: We may not apply rule (20) as long as dence on t 3 is not explicit. To make it so, we

if t < 3, if t 3. f (t) is written in this way, because the depenmanipulate the function slightly:

sin 2t = sin 2(t 3 + 3) = sin 2(t 3) cos 6 + cos 2(t 3) sin 6. Then we dene a function which has the Laplace transform (s) = Since obviously sin 2t = (t 3), i.e., f (t) = 0 if t < 3, (t 3) if t 3, e3s (2 cos 6 + s sin 6) . s2 + 4 s sin 6 2 cos 6 + 2 . 2+4 s s +4 (t) = sin 2t cos 6 + cos 2t sin 6,
def

we are in a position to apply (20). It follows that F (s) =

In most applications, rule (20) is combined with a simple and useful mathematical tool: the step function. Denition: The function u(t), which is dened as 0 u(t) = 1
1 2

for negative t for t = 0 for positive t

is called the unit step function, or also Heavisides function.

Do not worry about the denition of u(t) for t = 0: it is purely conventional and does not aect the Laplace transform in any way. It nds its place in more advanced topics. Heavisides function jumps from 0 to 1 when its argument is increased across zero. So, for example, what is u(t 74)? Since t 74 < 0 when t < 74, and t 74 > 0 when t > 74, we see that u(t 74) jumps from 0 to 1 as t is increased across the point t = 74. By rule (20), the Laplace transform of this function would be simply F (s) = e74s L [1] = e74s . s

Very often two step functions are combined to form a function that is zero for t up to a certain point, and becomes zero again from another point onwards.

24

The Laplace Transform

Example 29 Consider the function f (t) = u(t 1) u(t 4),


f(t) 1

pictured on the right: until t = 1 both us are zero, so their dierence is zero. After t = 4, 1 4 t both us are = 1, hence their dierence is zero again. Between t = 1 and t = 4, only u(t 1) 1 is equal to one, while u(t 4) is still zero: hence, u(t 1) u(t 4) = 1 there. The graph of this function is a rectangle with unit height and a length of three units. Its Laplace transform is F (s) = L [u(t 1)] L [u(t 4)] = es e4s . s

Things get interesting when we multiply a given function of t by u(t a) u(t b): we get a new function that coincides with the old one between t = a and t = b, but is identically zero everywhere else. Such a function is called a transient. Example 30 Find F (s), if f (t) = sin 5t for t between 1 and 4; f (t) 0 everywhere else.

Solution: The graph of sin 5t extends, of course, from to +. However, if sin 5t is multiplied by u(t 1) u(t 4), the part of the graph lying outside the interval (1, 4) is wiped o, so to speak, while the part between 1 and 4 is not aected. The graph of f (t), pictured on the right, jumps from 0 to sin 5 at t = 1, and f(t) then from sin 20 to 0 at t = 4. We may write f (t) = sin 5t u(t 1) u(t 4) =

= sin 5t u(t 1) sin 5t u(t 4).

1 4 t

Proceeding like in example 28, we write f (t) = sin 5(t 1 + 1) u(t 1) sin 5(t 4 + 4) u(t 4). Simplifying, we get: f (t) = sin 5(t 1) cos 5 + cos 5(t 1) sin 5 u(t 1) and nally, by (20): F (s) = s sin 5 s s sin 20 4s 5 cos 20 5 cos 5 + 2 + 2 e 2 e . 2 + 25 s s + 25 s + 25 s + 25

sin 5(t 4) cos 20 + cos 5(t 4) sin 20 u(t 4),

The Laplace Transform

25

It should be noted that problems of this type may always be done from rst principles. However, with some practice youll nd that the method of this section is usually better. For instance, the last example may also be done by calculating
4

F (s) =
1

est sin 5t dt

using integration by parts or Eulers formula. Do it, as an exercise. Example 31 Find F (s), if f (t) =
1/ 2

Solution: First of all, recall that, by denition, |A| = A whenever A is negative. Hence, since the expression 1/2 t changes sign for t = 1/2, then | 1/2 t| = t 1/2 if t > 1/2. Also, recall that f (t) 0 by denition if t is negative. Therefore, if t is negative, 0 1/ t if 0 < t < 1/2, f(t) f (t) = 2 1/ t 2 if t > 1/2, as shown in the picture. Using Heavisides unit step function, we may write f (t) = ( 1/2 t) u(t) u(t 1/2) + (t 1/2) u(t 1/2). Simplifying, we get immediately: f (t) = ( 1/2 t) u(t) + 2(t 1/2) u(t 1/2). Considering the transforms L [ 1/2] = and applying (20), we get immediately F (s) = As an exercise, calculate same answer.
1/2 1 ( /2 0 1/ 2 1/ 2

t .

1/2 1/2

L [t] =

1 , s2

1 2es/2 + . s2 s2
(t 1/2

t)est dt +

1/2)est dt and verify that you get the

Example 32 Find F (s) if f (t) = 3t for 0 t 2; f (t) = 6 for t > 2 (this is the same as example 12). Solution: We note that f (t) = 3t u(t) u(t 2) + 6u(t 2) =

= 3t u(t) 3(t 2 + 2) u(t 2) + 6u(t 2) = = 3t u(t) 3(t 2) u(t 2).

26

The Laplace Transform

It follows immediately that F (s) = same result as in example 12.

3 3e2s : s2 s2

Example 33 Find F (s) if f (t) = 3t t2 for 0 < t < 3, f (t) 0 everywhere else. Solution: Write f (t) = (3t t2 ) u(t) u(t 3) . To get an idea of the graph of f (t), imagine that you take the parabola y = 3tt2 and remove everything that lies below the t axis, as shown in the picture. Now, we have f (t) = (3tt2 )u(t)(3tt2 )u(t3). The rst term may be left as it is, but we must do some groundwork on the second term, if we are to use the shift theorem (20). Hence, we write

f(t)
2

(3t t2 ) u(t 3) = 3(t 3 + 3) (t 3 + 3)2 u(t 3) = = 3(t 3) (t 3)2 u(t 3). L 3t t2 = we get by (20) that L 3(t 3) (t 3)2 u(t 3) = 2 3 3, 2 s s

= 3(t 3) + 9 (t 3)2 6(t 3) 9 u(t 3) =

In this way, the second term depends on t is only through the expression t 3. And since

3 2 + 3 e3s . 2 s s

So much for the second term. The rst term requires no rearrangements, and we get immediately: L (3t t2 ) u(t) = obviously e0s = 1. So, nally, we get: F (s) = SHIFT RULES Formulas (20) and (19) are often called shift properties. They are summarized in the following table. f (t) f (t T ) u(t T ) eat f (t) F (s) esT F (s) F (s + a) 3 2 3+ 2 s s 3 2 + 3 e3s . 2 s s 3 2 3 s2 s e0s ;

The Laplace Transform

27

11. Additional Examples As a rule, the linearity property L [f1 + f2 ] = L [f1 ] + L [f2 ] holds only if L [f1 ] and L [f2 ] exist each one on its own. It may happen, though, that the Laplace transform of f1 + f2 does exist, even if f1 and f2 do not have a Laplace transform. Example 34 Find F (s), if f (t) = (et 1) t3/2 . Solution: We would like to use the shift theorem (19). However,
0

et t3/2 est dt

t3/2 est dt =

which is clearly meaningless. So, F (s) may not be split as we did in several previous examples; see for instance example 26. On the other hand, its easy to see that F (s) exists, and we are going to nd it in two steps. First of all we write g(t) = et 1 , t1/2 f (t) = et 1 g(t) = , 3/2 t t

and consider G(s). This is easy, because L et t1/2 and L t1/2 exist separately: L t1/2 = ( 1/2)! , s1/2 L et t1/2 = ( 1/2)! ; (s 1)1/2

the second one comes via the shift theorem (19). Therefore, G(s) = ( 1/2)! ( 1/2)! 1/2 . 1/2 (s 1) s

Finally, we go back to f (t) and use (18)division by t corresponds to integration by s: F (s) =


s

( 1/2)! ( 1/2)! 1/2 d = 1/2 ( 1) ( 1)1/2 1/2 1 1/ /2 2


s

= ( 1/2)!

We should show that, in the equation above, the last expression in square brackets goes to zero as tends to innity; this is a good revision example in 1st-year calculus. Writing 1/2 ( 1)1/2 1/2 ( 1)1/2 + 1/2 ( 1)1/2 , 1 = 1/ 1/ /2 ( 1)1/2 + 1/2 2 2 and simplifying the numerator, we get: ( 1)1/2 1/2 2 . 1 = 1/ /2 ( 1)1/2 + 1/2 2

28

The Laplace Transform

Its now clear that the right-hand side goes to zero as , as required. So, nally: F (s) = 0 ( 1/2)! ( 1)1/2 1/2 1 1/ /2 2 =
=s

= 2( 1/2)! s1/2 (s 1)1/2 . In section 2.7 well see that ( 1/2)! = . See also the comments at the end of section 1.4. Example 35 Find L [(2e3t 3e2t + 1) t5/2 . Solution: This problem is very similar to the preceding one, so well look only at the main points. We dene h(t) = 2e3t 3e2t + 1 , t1/2 g(t) = 2e3t 3e2t + 1 h(t) = , 3/2 t t f (t) = g(t) 2e3t 3e2t + 1 = . 5/2 t t

We seek F (s). Neither f (t) nor g(t) may by split as we did previously; however, for h(t) its correct to write H(s) = L 2e3t t1/2 L 3e2t t1/2 + L t1/2 = 2( 1/2)! 3( 1/2)! ( 1/2)! = + 1/2 . 1/2 1/2 (s 3) (s 2) s Using (18)division by t corresponds to integration by s, we get: G(s) =
s

H() d = ( 1/2)!

3( 2)1/2 1/2 2( 3)1/2 + 1 1/ 1/ /2 2 2

Proceeding like in the previous example, its possible to see that the expression in square brackets goes to zero as tends to innity. Therefore, G(s) = 2( 1/2)! 2(s 3)1/2 + 3(s 2)1/2 s1/2 . One more application of (18) yields F (s) =
s s

G() d =

2( 1/2)!

3( 2)3/2 3/2 2( 3)3/2 + 3 3/ 3/ /2 2 2

Once again, its possible to show that the expression is square brackets goes to zero as tends to innity. So, the nal answer is
4 F (s) = 3 ( 1/2)! 2(s 3)3/2 3(s 2)3/2 + s3/2 .

As an exercise, ll in the details of this example. Example 36 Find F (s), if f (t) = te|t1| . Solution: First of all, note that f (t) = 0 te1t tet1 if t is negative, by denition; if 0 < t < 1, if t is greater than 1.

The Laplace Transform

29

Using Heavisides unit step function, we write Simplifying, we get: f (t) = te1t u(t) u(t 1) + tet1 u(t 1). f (t) = te1t u(t) + t et1 e1t u(t 1) =

Considering the transforms L tet = 1 , (s + 1)2

= etet u(t) + 2t sinh(t 1) u(t 1) = = etet u(t) + 2[1 + (t 1)] sinh(t 1) u(t 1). L [sinh t] = 1 , 1 L [t sinh t] = d ds 1 1 = 2s , 1)2

s2

s2

(s2

and applying (20), we get immediately F (s) =

As an exercise, nd F (s) from rst principles, i.e., calculating 0 te1t est dt + 1 tet1 est dt and verify that you get the same answer. Compare the amount of work required. Example 37 Find the Laplace transform of f (t) = e4t 0 e3z 0 e2y Solution: Begin with s L [cos t] = 2 . s +1 Applying (19) we get: s5 . L e5t cos t = (s 5)2 + 1 Applying (16) we get: t s5 e5x cos x dx = L . s [(s 5)2 + 1] 0 Applying (19) we get:
t t z y 0

s 2s e + 2es 2 + . (s + 1)2 s 1 (s2 1)2


1

e5x cos x dx dy dz.

L e2t Applying (16) we get:


t

e5x cos x dx =
0

s3 . (s + 2) [(s 3)2 + 1] s3 . s(s + 2) [(s 3)2 + 1] s . (s + 3)(s + 5) [s2 + 1] 1 . (s + 3)(s + 5) [s2 + 1] 1 . (s2 1) [(s 4)2 + 1]

L Applying (19) we get:

e2y
0

e5x cos x dx dy =

L e
t

3t 0

2y 0

e5x cos x dx dy =

Applying (16), we get:


z y

e3z
0 0

e2y
0

e5x cos x dx dy dz =

Finally, applying (19), we get:


t z y

F (s) = L e4t

e3z
0 0

e2y
0

e5x cos x dx dy dz =

30

Tutorial Problems

The Laplace Transform

PROBLEMS

Transforms of Elementary Functions 1. Find the Laplace transform of each of the following functions. In each case, specify the values of s for which the integral (3) converges. (g) 6 sin 2t 5 cos 2t (a) 2e4t (h) (t2 + 1)2 (b) 3e2t (i) (sin t cos t)2 (c) 5t 3 (j) 3 cosh 5t 4 sinh 5t (d) 2t2 et (k) (5e2t 3)2 (e) 3 cos 5t (l) 4 cos2 2t (f) 10 sin 6t 2. Find the following Laplace transforms. (a) L [cosh2 4t] (b) L [3t4 2t3 + 4e3t 2 sin 5t + 3 cos 2t] (c) L [cosh3 t] (d) L [sin5 t] (e) L [sin 3t cos 2t] (f) L [cos 2t cos 5t]. 3. Using the identity c! = (c + 1)!/(c + 1) [see (12)], show that Multiplication by s
t et if 0 < t < 1, and g(t) = e if 0 < t < 1, 0 if t > 1, e if t > 1. Find F (s), G(s) and show that rule (13) holds for g but not for f . Why is it so? c1+

computer package to observe this result numerically.

lim c! = . Use a good

4. Let f (t) =

2t if 0 t 1 t if t > 1, Does formula (13) hold? Explain. 5. Given f (t) = t2 if 0 t 1 0 if t > 1, Does formula (14) hold? Explain. 6. Given f (t) =

nd (a) F (s), (b) L [f(t)]. nd (a) F (s), (b) L [f (t)]. Division by s

7. Calculate L 0 ( 3 5 + sinh 2 ) d : (a) By doing the integral rst, (b) Using formula (16). Verify that you get the same answer. 8. Calculate L
t (2 sin 0

cos 7 ) d

applying formula (16).

Multiplication by t 9. Find the following Laplace transforms using formula (17). (a) L [t3 e3t ] (e) L [t cosh 3t] (b) L [(t + 2)2 et ] (f) L [t sinh 2t] (c) L [t(3 sin 2t 2 cos 2t)] (g) L [(t 1)(t 2) sin 3t] (d) L [t2 sin t] (h) L [t3 cos t] . (a)
0

10. Applying (17), calculate:

t e3t sin t dt,

(b)

2 t t e 0

cos t dt.

The Laplace Transform

Tutorial Problems

31

Division by t 11. Find the following Laplace transforms; read example 22 before attempting (d). (a) L [(1 et )/t] (c) L [(cosh t cos t)/t] (b) L [sinh2 t/t] (d) L [sinh2 t/t2 ].
t

12. Calculate (a) L

1 e d ,

(b) L t Shifting

sin d .

13. Find the following Laplace transforms. (a) L [et cos 2t] (b) L [2e3t sin 4t] (c) L [e2t (3 sin 4t 4 cos 4t)] (d) L [et (3 sinh 2t 5 cosh 2t)] sinh t 3 t 15. Find F (s), given: 2t if 0 t < 5, (a) f (t) = 0 everywhere else. (a) L 16. Find F (s), given: (a) f (t) = |t2 4| ANSWERS 14. Find:

(e) L [et sin2 t] (f) L [(1 + tet )3 ] (g) L [t2 u(t 5)] (h) L [e2t t2 u(t 1)]. (b) L e3t sin 2t t sin t 0 if 0 < t < 1, everywhere else.

(b) f (t) =

17. Find L |t2 7t + 12| .

(b) f (t) = |2 et |

(a) 2/(s 4), s > 4 (g) (12 5s)/(s2 + 4), s > 0 (b) 3/(s + 2), s > 2 (h) (s4 + 4s2 + 24)/s5 , s > 0 2 (c) (5 3s)/s , s > 0 (i) (s2 2s + 4)/s(s2 + 4), s > 0 3 3 (d) (4 + 4s s )/s (s + 1), s > 0 (j) (3s 20)/(s2 25), s > 5 2 (e) 3s/(s + 25), s > 0 (k) 25/(s 4) 30/(s 2) + 9/s, s > 4 2 (f) 60/(s + 36), s > 0 (l) 2/s + 2s/(s2 + 16), s > 0. 2 (a) (s2 32)/s(s2 64) (b) 72/s5 12/s4 + 4/(s + 3) 10/(s2 + 25) + 3s/(s2 + 4) (c) [s/(s2 9) + 3s/(s2 1)]/4 (d) [5/(s2 + 25) 15/(s2 + 9) + 10/(s2 + 1)]/16 (e) [5/(s2 + 25) + 1/(s2 + 1)]/2 (f) [s/(s2 + 49) + s/(s2 + 9)]/2. 3 Using the open-source package maxima one gets that (0.9)! 9.51, (0.999)! 999.42 and (0.999999)! 106 . As c gets closer to 1, c! becomes practically equal 1/(c + 1) [up to the 6th signicant digit]. 1 e1s e1s 1 e1s , G = + . Rule (13) does not hold for f because f is 4 F = s1 s1 s discontinuous at t = 1, whereas g is continuous throughout and g is sectionally continuous. 2 s 2 ] = 2/s es /s. 5 (a) F (s) = 2/s e (1/s + 1/s ); (b) L [f Formula (13) does not hold because f is not continuous. 1

32

Tutorial Problems

The Laplace Transform

6 (a) F (s) = 2/s3 es (2/s3 + 2/s2 + 1/s); (b) L [f ] = 2(1 es )/s. Formula (14) does not hold because f and f are not continuous. 6 1 1 8 . 8 2 + 64 2 + 36 s s s s (e) (s2 + 9)/(s2 9)2 9 (a) 6/(s + 3)4 (f) 4s/(s2 4)2 (b) (4s2 4s + 2)/(s 1)3 2 2 2 (g) (6s4 18s3 + 126s2 162s + 432)/(s2 + 9)3 (c) (8 + 12s 2s )/(s + 4) 2 2 3 (h) (6s4 36s2 + 6)/(s2 + 1)4 . (d) (6s 2)/(s + 1) 10 (a) 3/50, (b) 1/2. s+1 s2 + 1 1 11 (a) ln (c) 2 ln 2 s s 1 s s 1 1 (b) 2 ln (d) arcoth(s/2) 2 s ln 24 s s2 4 1 1 , (b) (arccot s)/s2 + 1/s(s2 + 1). 12 (a) ln 1 + s s 13 (a) (s + 1)/(s2 + 2s + 5) (e) 2/(s + 1)(s2 + 2s + 5) 2 (b) 8/(s 6s + 25) (f) 1/s + 3/(s + 1)2 + 6/(s + 2)3 + 6/(s + 3)4 2 (c) 4(5 s)/(s 4s + 20) (g) (2/s3 + 10/s2 + 25/s) e5s 2 (h) 2/(s+2)3 +2/(s+2)2 +1/(s+2) e2s (d) (1 5s)/(s + 2s 3) 14 15 16 17 ( 1/3)! ( 1/3)! , 2/3 2(s 1) 2(s + 1)2/3 2 (2 + 10s) e5s (a) F (s) = 2 , s s2 8 2 4 4 (a) F (s) = 3 + 2 + 3 e2s , s s s s 2 7 12 2 1 2+ 2 3 2 e3s + 2 3 s s s s s (a) F (s) = (b) F (s) = arccot(s + 3)/2. (1 + es ) . (s2 + 2 ) 4 1 4 s 2 + (b) F (s) = 2 . s s1 s1 s 2 1 + 2 e4s . 3 s s (b) F (s) =

The Inverse Transform

33

Chapter Two

THE INVERSE TRANSFORM

1. Introduction Denition: If F (s) is the Laplace transform of f (t), then f (t) is called the inverse Laplace transform of F (s), and is denoted f (t) = L1 F (s) .

There is a formula (Mellins formula) expressing L1 [F ] as a denite integral, analogous to the integral (3) which denes L [f ]. However, Mellins formula requires some knowledge of the theory of complex variables, which at this stage you dont have. On the other hand, it is still possible to nd the inverse Laplace transform in a wide range of cases, using only the basic transforms (4)(9), the rules described in chapter 1, and some skill. And since skill comes only through practice, the best advice, at this point, is that you go through as many examples as possible, then more, and then a few more. Laplace calculus uses a variety of techniques, some of which are routine, some tricky: it is similar, in this respect, to integral calculus. You must build your own personal library of such tricks. Moreover, as youll get good, youll begin to spot other ways of doing the examples in these notes, and youll want to know whether your method is better than the one presented here. Try your way, then; be bold. Compare the methods; nd points going for/against each one. Example 38 Find L1 [(s 2)4 /s6 ]. Solution: Expand the numerator by the binomial formula: s4 8s3 + 24s2 32s + 16 1 8 24 32 16 (s 2)4 = = 2 3 + 4 5 + 6. 6 6 s s s s s s s Then, by (9), it follows immediately: L1 24 32 16 8 (s 2)4 = t t2 + t3 t4 + t5 = s6 2! 3! 4! 5! 4 2 = t 4t2 + 4t3 t4 + t5 . 3 15

Example 39 Find L1 [1/(s a)b+1 ]. Solution: Since

b! sb+1

= L [tb ],

34

The Inverse Transform

we apply the s-shift property (19): we get immediately L1 1 tb eat . = (s a)b+1 b!

Comment: if b is an integer, we may proceed dierently, noting that db 1 b! = (1)b b = b+1 (s a) ds s a then it follows, by rule (13) [multiplication by t]: 1 tb eat =L . (s a)b+1 b! However, the rst method is more general because it works even when b is not integer. Example 40 Find f (t), if F (s) = (3s 5)/(s 1)4 . 3(s 1 + 1) 5 3 2 3s 5 = = . 4 4 3 (s 1) (s 1) (s 1) (s 1)4 F (s) = 3 L t2 et 1 L t3 et , 2 3 and nally f (t) = 1 t2 et 9 2t . 6 Example 41 Find L1 [(s 16)/(s2 2s 24)]. d ds
b

L eat ;

Solution: Write

Now, proceeding like in the previous example, we get immediately

Solution: Write s2 2s 24 = (s 1)2 25 (this step is called completing the square). It follows s 16 s1 15 = . 2 2s 24 2 25 s (s 1) (s 1)2 25 But s = L [cosh 5t] 25 5 = L [sinh 5t]; 2 25 s s2

hence, by the s-shift property (19), it follows

s1 = L [et cosh 5t] (s 1)2 25 15 = L [3et sinh 5t]. (s 1)2 25 and nally L1 s2 s 16 = et cosh 5t 3 sinh 5t . 2s 24

The Inverse Transform

35

Example 42 Find f (t), if F (s) = (2s3 + s2 + 2s + 2)/(s5 + 2s4 + s3 ). Solution: Observe that s3 may be factored out in the denominator. Therefore, 2s3 + s2 + 2s + 2 2s3 + s2 + 2s + 2 . = 3 2 s5 + 2s4 + s3 s (s + 2s + 2) Now, we split the numerator: 2s3 s2 + 2s + 2 2 1 2s3 + s2 + 2s + 2 = 3 2 + 3 2 = 2 + . s3 (s2 + 2s + 2) s (s + 2s + 2) s (s + 2s + 2) s + 2s + 2 s3 Finally, we observe that s2 + 2s + 2 = (s + 1)2 + 1. So, F (s) = and nally f (t) = 2 et sin t + 1 t2 . 2 Example 43 Find f (t), if F (s) = s/ (s + 4)5 . Solution 1: Rewrite F (s): F (s) = s+44 (s + = 1 (s + 4)3 4 (s + 4)5 . 2 1 + 3, 2 +1 (s + 1) s

4)5

The inverse-transform of the right-hand side may be found by applying the shift theorem (19), which reduces it to a pair of basic transforms of the form (9): L1 1 (s + 4)3 4 (s + 4)5 = e4t L1 1 s3/2 4 s5/2 = 4e4t t3/2 e4t t1/2 . 1 3 2! 2!

Solution 2: Multiplication by s corresponds to dierentiation by t. Therefore, using (13), we get: L1 d s = L1 5 dt (s + 4) 1 (s + 4)5 = d e4t t3/2 3 dt 2! =
3 4t 1/2 e t 4e4t t3/2 + 2 3 . 3 2! 2!

Note that this step relies also on the obvious fact that limt0+ e4t t3/2 = 0 (Why?). After simplications, this result becomes identical to the one obtained before. 2. Partial Fractions: Heavisides Method Expansion in partial fractions is a well known technique and is usually taught in a rst-year calculus course. Here, we shall only discuss some of its main features with a view to applications. Suppose N (s) , F (s) = D(s)

36

The Inverse Transform

where both the numerator and the denominator are polynomials in s, and degree (N ) < degree (D). Also, suppose initially that D(s) may be written as the product of m linear factors, i.e., D(s) = (s r1 )(s r2 ) (s rm ). (21)

This means that D(s) has only simple roots and there are m of them, where m = degree (D); it also means that when (21) is expanded out in terms of powers of s, the coecient of the leading term (which is sm ) is exactly 1. Under these assumptions, one may write N (s) c1 c2 cm = + + + , D(s) s r1 s r2 s rm (22)

and the constants c1 . . . cm are found by Heavisides cover-up method. Here is how Heavisides method works: multiply both sides of (22) by (s rk ); we get s rk s rk s rk s rk (s rk )N (s) = c1 + c2 + + ck + + cm . D(s) s r1 s r2 s rk s rm Now simplify the k-th fraction on the right-hand side, which is clearly equal to 1. The resulting formula is true for every s; in particular, if s = rk , the right-hand side is R. H. S. = c1 0 + c2 0 + + ck 1 + + cm 0 = ck . The left-hand side may be simplied too, because by (21), L. H. S. = (s rk )N (s) . (s r1 )(s r2 ) (s rk ) (s rm )

Simplifying the common factor (s rk ) is tantamount to covering up the same factor in the denominator of the original fraction (hence the slang name): therefore N (rk ) = ck . (rk r1 )(rk r2 ) (rk rk1 ) (rk rk+1 ) (rk rm ) Example 44 Expand (3s + 7)/(s 3)(s + 1) in partial fractions. Solution: Write 3s + 7 A B = + . (s 3)(s + 1) s3 s+1 (23)

Apply (23). Covering up s 3 on the left-hand side and letting s = 3, we get A = 4. Covering up s + 1 and letting s = 1 we get B = 1. So, nally, 3s + 7 4 1 = . (s 3)(s + 1) s3 s+1

If the degree of N is not less than the degree of D, then one may always divide N by D by long division, eventually getting N/D = Q + R/D, where Q is the quotient of the division and R, the remainder, is of a degree less than D. Oliver Heaviside (18501925), English.

The Inverse Transform

37

Example 45 Solve the dierential equation x 2x 24x = 0, x(0+) = 14. Solution: The transformed equation is 14 s + s2 X 2(1 + sX) 24X = 0, or (s2 2s 24)X = s 16.

given x(0+) = 1, and

Hence, X = (s 16)/(s2 2s 24). This is precisely the transform of example 41, so we could use the result found there and relax. However, we need to practice partial fractions. Therefore, we rst nd the roots of the denominator, i.e., we solve s2 2s 24 = 0, and get s = 6 and s = 4. Then we write X= s 16 c1 c2 = + . (s 6)(s + 4) s6 s+4

Now, covering up the factor (s 6) in the expression in the middle and setting s = 6, we get c1 = 6 16 = 1. 6+4

Similarly, covering up s + 4 and setting s = 4, we get c2 = It follows X= and nally x = e6t + 2e4t . Is this the same answer found in example 41? Yes it is: et cosh 5t 3et sinh 5t = as expected. Example 46 Expand (s3 2s2 + 3s 5)/s(s 1)(s 2)(s + 3).
1 2

4 16 = 2. 4 6 1 2 + , s6 s+4

et+5t + et5t

3 2

et+5t et5t = e6t + 2e4t ,

Solution: Identify the roots of D(s): they are equal to 0, 1, 2, 3. Prepare for expansion: c1 c2 c3 c4 s3 2s2 + 3s 5 = + + + . s(s 1)(s 2)(s + 3) s s1 s2 s+3

38

The Inverse Transform

By Heavisides method, we get: cover up s: cover up s 1: cover up s 2: cover up s + 3: It follows that 5 3 1 59 s3 2s2 + 3s 5 = + + + . s(s 1)(s 2)(s + 3) 6s 4(s 1) 10(s 2) 60(s + 3) 03 2 02 + 3 0 5 5 = (0 1)(0 2)(0 + 3) 6 3 2 1 21 +315 3 c2 = = 1(1 2)(1 + 3) 4 3 2 1 2 22 +325 = c3 = 2(2 1)(2 + 3) 10 3 2 (3) 2 (3) + 3 (3) 5 59 c4 = = . (3)(3 1)(3 2) 60 c1 =

One more comment, very useful: go back to (22). If we let s , we certainly get 0 = 0, because the degree of D(s) is assumed to be greater than the degree of N (s) by at least one unit. However, if we multiply both sides by s and then let s , we nd that the left-hand side may approach a nite limit, as well as zero; and on the right-hand side we nd m limits which may all be done by inspection. This procedure, called testing the transform at innity may be used (i) as a quick numerical check on the calculations, or (ii) to nd a coecient, when all but one have been computed. Example 47 Go back to the last example. Consider the expansion c1 c2 c3 c4 s3 2s2 + 3s 5 = + + + . s(s 1)(s 2)(s + 3) s s1 s2 s+3 and multiply both sides by s: we get s(s3 2s2 + 3s 5) s s s s = c1 + c2 + c3 + c4 . s(s 1)(s 2)(s + 3) s s1 s2 s+3 Note that: L. H. S. = hence
s

s4 + lower powers of s : s4 + lower powers of s

lim [L. H. S.] = 1.

By inspection,
s

lim [R. H. S.] = c1 + c2 + c3 + c4 . 1 59 5 3 + , 1= + + 6 4 10 60

And indeed

As a rule, before embarking on a partial fractions expansion, you should always verify that this is the case.

The Inverse Transform

39

as expected. In this way we have checked our calculations. 3. Partial Fractions: Multiple Roots.

When D(s) has one multiple root, or more, then Heavisides method does not yield all coecients. However, it still works ne for all the simple roots; it also produces immediately one coecient for each multiple root. Recall that by testing at innity one may nd one more coecient, in addition to the ones found by the standard cover up method. If only one coecient is missing, this is enough to complete an expansion. Example 48 Expand (3s 2)/(s + 5)(s 1)2 . 3s 2 b1 b2 c + + = . 2 (s + 5)(s 1) s + 5 s 1 (s 1)2 Multiplying both sides of this equation by s + 5 and setting s = 5, we nd c= 17 3 (5) 2 = . 2 (5 1) 36

Solution: We seek an expansion of the form

Similarly, multiplying both sides by (s 1)2 and setting s = 1 we nd b2 = 312 1 = . 1+5 6

Now only b1 remains to be found. Testing the transform at innity, we see that sN (s) 3s2 + lower powers of s = 3 , D(s) s + lower powers of s hence sN (s)/D(s) tends to 0 as s . Therefore 0= and nally b1 = 17/36. Example 49 Expand (s3 + 11)/(s 1)2 (s + 2)2 . s3 + 11 c1 c2 b1 b2 = + + + . 2 (s + 2)2 2 (s 1) s 1 (s 1) s + 2 (s + 2)2 We nd c2 by multiplying both sides of this equation by (s 1)2 and setting s = 1: c2 = 4 1 + 11 = . 2 (1 + 2) 3 17 + b1 , 36

Solution: Write

40

The Inverse Transform

Similarly, multiplying both sides by (s + 2)2 and setting s = 2, we nd b2 : b2 = At this point we have s3 + 11 c1 4 b1 1 = + + + . 2 (s + 2)2 2 (s 1) s 1 3(s 1) s + 2 3(s + 2)2 Multiplying both sides by s and letting s we get 1 = c1 + b1 . We need one more bit of information; we get it by testing one numerical value of s in (24): any value not used so far would work, but s = 0 seems to be easy enough. We get c1 4 b1 1 11 = + + + , 4 1 3 2 12 1 4 = c1 + b1 , 3 2 (24) 8 + 11 1 = . (2 1)2 3

or

hence b1 = 14/9, c1 = 5/9.

Broadly speaking, nding the coecients in the presence of multiple roots requires more work, be it with pencil and paper, or computer time. Several generalized Heavisides methods have been devised to handle multiple roots, but none of them ultimately avoids a fair amount of tedious calculations. A good discussion may be found in G. Doetsch, Guide to the applications of the Laplace and Z-transforms, van Nostrand (1971). One method thats easy to remember but not particularly fast, consists of moving terms with known coecients from the right-hand side to the left-hand side, rearranging and simplifying. It is best described by an example. Example 50 Find f (t), given that F (s) = 16/(s2 3s + 2)s4 . Solution: We expand F (s) in partial fractions. We write 16 16 A B C1 C2 C3 C4 = = + + + 2 + 3 + 4. (s2 3s + 2)s4 (s 1)(s 2)s4 s2 s1 s s s s By the cover-up method we get: 16 = 1, 24 16 B= = 16, 1 16 = 8. C4 = (1)(2) A= By testing the transform at innity, we get 0 = 1 16 + C1 ,

The Inverse Transform

41

hence C1 = 15. So far we have established that (s2 16 16 15 C2 1 C3 8 + + 2 + 3 + 4. = 4 3s + 2)s s2 s1 s s s s

Now, we move the term 8/s4 across to the left-hand side. It follows that 16 16 15 C2 8 1 C3 + + 2 + 3. 4 = 4 3s + 2)s s s2 s1 s s s

(s2

Simplify the left-hand side: 8 8s2 + 24s 8s + 24 16 4 = 2 = 2 . 2 3s + 2)s4 4 (s s (s 3s + 2)s (s 3s + 2)s3 Substite back: it follows that (s2 1 16 15 C2 C3 8s + 24 = + + 2 + 3, 3 3s + 2)s s2 s1 s s s

and C3 may be now found by Heavisides method: C3 = 24 = 12. 2

Repeat the procedure. Move the term 12/s3 to the left-hand side: it follows that 16 15 C2 12 1 8s + 24 + + 2. 3 = 3 3s + 2)s s s2 s1 s s

(s2

Simplify the left-hand side: 12 12s2 + 28s 12s + 28 8s + 24 3 = 2 = 2 . 2 3s + 2)s3 3 (s s (s 3s + 2)s (s 3s + 2)s2 Sobstitute back: it follows that (s2 12s + 28 1 16 15 C2 = + + 2, 2 3s + 2)s s2 s1 s s

and C2 may be found by Heavisides method: C2 = All the coecients have been found: (s2 16 15 14 12 1 8 16 + + 2 + 3 + 4. = 4 3s + 2)s s2 s1 s s s s
4 3 3 t .

28 = 14. 2

So, nally, f (t) = e2t 16et + 15 + 14t + 6t2 +

42

The Inverse Transform

Another method, which at rst seems simple, is to plug in as many values of s as there are missing coecients. These values may be freely chosen, as long as none of them coincides with a root of D(s). In this way one gets a system of n linear equations with n unknowns. The drawback of this method is that the amount of work needed to solve a system of n linear equations grows (for large n) with speed of n3 ; it can be rather laborious even for n = 3. Yet another approach to the previous example is to apply rule (16) [division by s is associated with integration with respect to t]. Well come back to this idea in section 5. The so-called calculus of residues is probably much better than any of the methods described so far, but since it requires some knowledge of the theory of complex variables, we leave it for a more advanced course. 4. Partial Fractions: Complex Roots In principle, if D(s) has complex roots, the methods described in the sections 23 are still applicable. The only dierence is that the coecients of the expansion are, in general, complex. Complex arithmetic is inherently more time-consuming than real arithmetic. In applications, real trigonometric functions are generally preferable to complex exponentials, though a good case may sometimes be made for using the latter. In engineering applications N (s) and D(s) are virtually certain to be real polynomials. It may be shown, then, that the complex roots of D(s), if present, come in complex conjugate pairs, and the corresponding coecients in the partial fractions expansion are also complex conjugate. To x the ideas, consider a simple case where D(s) is real and has a complex root a + ib with multiplicity 1. Then a ib is also a root, and the expansion has the form c + id c id N (s) = + + (other terms). D(s) s a ib s a + ib Grouping terms on the right-hand side, we get N (s) Bs + C = + (other terms), D(s) (s a)2 + b2 where B and C are real, and may be expressed in terms of the old constants a,b, c and d (convince yourself of this). However there is no need to nd c and d, since one may nd B and C directly. The following examples illustrate this method. Example 51 Expand (2s2 2s + 1)/(s 1)(s2 + 4) in partial fractions. Solution: Both following expansions are possible: c1 c2 id2 c2 + id2 2s2 2s + 1 = + + , 2 + 4) (s 1)(s s1 (s i2) (s + i2) A Bs + C = + 2 , s1 s +4 but the rst requires complex arithmetic, the second one does not. Let us do the latter. We nd immediately A= 1 22+1 = 1+4 5

Both when done by a computer and when done by pencil and paper.

The Inverse Transform

43

(by Heavisides method: nothing new here). To nd B and C, we may either continue with the cover-up method, or use a couple of shortcuts. Method 1: By the cover-up method. Multiply both sides by s2 + 4 and simplify: it follows (2s2 2s + 1)(s2 + 4) A(s2 + 4) (Bs + C)(s2 + 4) = + (s 1)(s2 + 4) s1 s2 + 4 2s2 2s + 1 A(s2 + 4) = + Bs + C. s1 s1 If we now set s = i2 , we make s2 + 4 = 0. Either root may be used; at the end the nal results will be the same. For instance, setting s = i2 we get 8 i4 + 1 = 0 + i2B + C i2 1 = 1 + i18 = i2B + C. 5

Hence, separating real and imaginary part, we nd B = 9/5 and C = 1/5. So, nally, 1/5 (9s 1)/5 2s2 2s + 1 = + . (s 1)(s2 + 4) s1 s2 + 4 Method 2: Use the test at innity. Multiply both sides of the expansion by s and let s . We get 2s3 + s/5 Bs2 + Cs = + , s3 + s1 s2 + 4 and hence (in the limit of s ) 2 = 1/5 + B = B = 9/5.

At this point, only C is missing. We nd it by inspection, by substituting for s an arbitrary value that hasnt yet been used. For example, s = 0, why not? We get immediately 1 1/5 C = + 4 1 4 and nally = C = 1/5,

We get the same expansion, as expected.

1/5 (9s 1)/5 2s2 2s + 1 = + . 2 + 4) (s 1)(s s1 s2 + 4

Example 52 Expand (s2 16s + 23)/(s2 2s + 5)(s 1)(s 3) in partial fractions. Solution: The quadratic factor in the denominator is the product of two complex-conjugate pairs: s2 2s + 5 = (s 1 + i2)(s 1 i2). Hence, D(s) has two complex roots and two real roots, namely 1 + i2, 1 i2, 1 and 3. We seek an expansion of the form A B Cs + D s2 16s + 23 = + + 2 2 2s + 5)(s 1)(s 3) (s s 1 s 3 s 2s + 5

44

The Inverse Transform

Coecients A and B are found immediately by Heavisides method: 1 16 + 23 = 1 (1 2 + 5)(1 3) 9 48 + 23 B= = 1 (9 6 + 5)(3 1) A= To nd C and D, we have again the choice of two simple methods. Method 1: Continuing with the cover-up method, we multiply both sides of the expansion by s2 2s + 5, s2 2s + 5 s2 2s + 5 (Cs + D)(s2 2s + 5) (s2 16s + 23)(s2 2s + 5) =A +B + , (s2 2s + 5)(s 1)(s 3) s1 s3 s2 2s + 5 simplify s2 16s + 23 s2 2s + 5 s2 2s + 5 =A +B + Cs + D, (s 1)(s 3) s1 s3 (1 + i2)2 16(1 + i2) + 23 = 0 + 0 + C(1 + i2) + D, (1 + i2 1)(1 + i2 3) hence 4 i28 = C + D + i2C, 4 i4 1 i7 (1 i7)(1 + i) = = 3 + i4 = 1 i 2 = C + D + i2C. Separating the imaginary part we get i4 = i2C, Method 2: By the test at innity. Note that F (s) is of second degree in the numerator, fourth degree in the denominator, hence sF (s) 0 as s tends to innity. On the other hand, s A B Cs + D + + 2 s 1 s 3 s 2s + 5 A+B+C as s . hence C = 2, and nally D = 3 C = 1.

and set s = 1 + i2 (recall that s2 2s + 5 = 0 if s = 1 i2). We get

which yields

Since we already found that A = 1 and B = 1, we get immediately that C = 2. We still miss D, but when only one coecient is missing, we may nd it by inspection, substituting for s any number that hasnt been used. Again, s = 0 is the obvious choice. We get immediately 1 D 23 =1+ + , 5(1)(3) 3 5 and hence D = 1.

The Inverse Transform

45

We get the impression that the second method is slightly faster than Heavisides cover-up method. The problem is, it may be used only once. For a partial-fractions expansion with two complex roots or more, you must use the cover-up method at least once, and nish the job by the method described above. Example 53 Expand (s2 4s 10)/(s2 2s + 10)(s2 + 4) in partial fractions. Solution: The denominator has two pairs of complex conjugate roots, namely s = 1 i3 and s = i2. We seek an expansion of the form s2 4s 10 As + B Cs + D = 2 + 2 . 2 2s + 10)(s2 + 4) (s s 2s + 10 s +4 Multiplying both sides of this equation by s2 + 4 and setting s = i2 we get (i2)2 4(i2) 10 = C(i2) + D, (i2)2 2(i2) + 10 or 14 i8 = i2C + D. 6 i4

Simplifying the left-hand side, we get

14 i8 7 i4 (7 i4)(3 + i2) = = = 1 i2, 6 i4 3 i2 13 and nally (comparing real and imaginary part) C = 1, D = 1. Now we test at innity: s2 4s 10 = 0, s (s2 2s + 10)(s2 + 4) Cs + D As + B + 2 = A + C. lim s s s2 2s + 10 s +4 lim s But we already know that C = 1, hence A = 1. Finally, substituting s = 0 (any real number would do), we get 10 B D = + : (10)(4) 10 4 having found before that D = 1, we deduce that B = 0. Example 54 Expand (s3 + 1)/(s 1)(s2 + 1)2 in partial fractions. Solution: We seek an expansion of the form A Bs + C Ds + E s3 + 1 = + 2 + 2 . (s 1)(s2 + 1)2 s1 s +1 (s + 1)2 By Heavisides method, 1+1 =A+0+0 (1 + 1)2 = A = 1/2.

46

The Inverse Transform

Continuing with Heavisides method, we multiply through by (s2 + 1)2 and simplify: we get s3 + 1 A(s2 + 1)2 = + (Bs + C)(s2 + 1) + Ds + E. (s 1) s1 Substituting s = i we get i + 1 = 0 + 0 + iD + E. i1

The left-hand side of this equation is equal to 1, hence it follows immediately that D = 0 and E = 1. Testing at innity we get the equation s4 + As Bs2 + Cs Ds2 + Es = + + 4 ; s5 + s1 s2 + 1 s + as s this yields 0 = A+B. Having already found that A = 1/2, we deduce that B = 1/2. At this point, only C remains to be found: so, we substitute s = 0 in the expansion. We obtain 1 = A + C + E, where A = 1/2 and E = 1. It follows that C = 1/2. 5. Manipulations of the Transform In some applications it may happen that it is much easier to nd L1 [s F (s)], for a given F (s), than L1 [F (s)] . In these cases L1 [F (s)] may be found using property (16) [division by s is associated with integration with respect to t]. Example 55 Find L1 1/s(s2 + 49) .

Solution: Instead of doing a direct partial fractions expansion, which would require some complex arithmetic, we note that 1 1 = L 7 sin 7t . 2 + 49 s Hence 1 =L 2 + 49) s(s Integrating, we get L1 1 1 cos 7t . = s(s2 + 49) 49
0 t

sin 7 d . 7

Solution: First of all we note that

Example 56 Find L1 (3s 2)/s3 (s + 1) , without using partial fractions. 3s 2 3 2 = 2 3 , + 1) s (s + 1) s (s + 1) 1 = et . s+1

s3 (s and

L1

The Inverse Transform

47

It follows by (16) L1 L1 L1 So, nally, L1 3s 2 = 3 t 1 + et 2 s3 (s + 1)


1 2 t 2

1 = s(s + 1) 1 = 2 (s + 1) s 1 = 3 (s + 1) s

t 0 t 0 t 0

e d = 1 et , 1 e d = t 1 + et ,
1 1 + e d = 2 t2 t + 1 et .

t + 1 et =

= t2 + 5t 5 + 5et . Example 57 Find L1 [50/s2 (s2 + 6s + 10)], without using partial fractions. Solution: Completing the square in the denominator, we get s2 From L1 we deduce immediately L1 and nally L1 50 = 2 (s2 + 6s + 10) s
t 0

50 50 = . + 6s + 10 (s + 3)2 + 1 50 = 50e3t sin t (s + 3)2 + 1

50 = 50 s(s2 + 6s + 10)

t 0

e3 sin d = 5 e3t (5 cos t + 15 sin t),

5 e3 (5 cos + 15 sin ) d = 5t 3 + e3t (3 cos t + 4 sin t).

Note that the integrals in the equations above may be done by parts or by Eulers formula; the latter is more advisable. In other problems it may happen that L1 [F (s)] may be found more easily than L1 [F (s)]. In these cases, we obtain L1 [F (s)] by using property (18)integration with respect to s corresponds to division by t. Example 58 Find f (t), if F (s) = arcoth s. Solution: You might think this problem is intractable, until you notice that F (s) = therefore, you may write F (s) =
s

1 : s2 1 1 d. 1

48

The Inverse Transform

Now you see the light: integration with respect to s corresponds to division by t, and s2 It follows immediately by (18) that f (t) = sinh t . t 1 = L [sinh t]. 1

Example 59 Find f (t), if F (s) = arcoth s2 . Solution: Dierentiating with respect to s we get F (s) = Noting the partial fractions expansion s4 we get F (s) = It follows immediately that F (s) =
s

s4

2s . 1

[s > 1]

2s s s 2s = 2 = 2 2 , 2 + 1) 1 (s 1)(s s 1 s +1 s2 s s 2 = L [cosh t cos t]. 1 s +1

2 d, 1 +1

and nally, by (18), that f (t) = cosh t cos t t

Example 60 Find f (t), if F (s) = ln[(s a)/(s b)], where 0 < a < b. Solution: Dierentiating with respect to s and changing sign, we get F (s) = It follows immediately that F (s) =
s

1 1 = L ebt eat . sb sa 1 1 d, b a ebt eat . t

[s > a]

and nally, by (18), f (t) =

We have just seen examples where L1 [F (s)] was easier to nd that L1 [F (s)], and so F (s) was conveniently written as an integral with respect to s. Similarly, one may sometimes use integration with respect to t to simplify a transform. In this case, of course, instead of (18), we use (16): division by s corresponds to integration with respect to t.

The Inverse Transform

49

Example 61 Find L1 [s/(s2 + 2 )2 ] and hence L1 [1/(s2 + 2 )2 ] (this is a classic). Solution: consider the identity L sin t 1 . = 2 s + 2

Dierentiating with respect to s, and making use of (17), we get L It follows immediately that L1 (s2 t sin t s = 2 )2 + 2 t sin t 1 d = 2 + 2 ds s = 2s . + 2 )2

(s2

This answers the rst question. To answer the second part, we write 1 s 1 = 2 . (s2 + 2 )2 s (s + 2 )2 Recalling (16) [division by s corresponds to integration with respect to t], we get L1 1 1 = 2 + 2 )2 (s 2
t

sin d.
0

The integral above is trivial, and one nally obtains L1 1 sin t t cos t . = 2 )2 + 2 3

(s2

Part of this problem has already been solved by means of Eulers formula: see example 17 in chapter 1. The method shown here is less elegant, but still instructive. Both methods may be generalized to transforms of the form s/(s2 + 2 )n or s/(s2 + 2 )n , but the calculations become laborious as n increases. Example 62 Continue example 54, and nd f (t) if F (s) = (s3 + 1)/(s 1)(s2 + 1)2 . Solution: In example 54 it was found that F (s) = we note immediately that L1 1 = et , s1 and L1 s1 = cos t sin t. s2 + 1
1 2

s1

1 2 (s 1) s2 + 1

(s2

1 ; + 1)2

We conclude by following example 61, with = 1. We get immediately that L1 1 sin t t cos t , = 2 + 1) 2

(s2

50

The Inverse Transform

and so, nally,

1 f (t) = 1 et 2 (cos t sin t) 1 (sin t t cos t) = 2 2 = 1 (et cos t + t cos t). 2

6. Miscellaneous Examples Example 63 Find L1 [(s + 1)/(s2 10s + 29)2 ]. Solution: First of all, complete the square in the denominator: s2 10s + 29 = (s 5)2 + 4. Now write (s2 Dene G(s) = s+1 s5+6 = 2 10x + 29) (s 5)2 + 4 s+6 . (s2 + 4)2
2.

By denition, F (s) = G(s 5); hence, by the s-shift property (19), f (t) = e5t g(t). But g(t) may be found immediately, substituting = 2 in the results of example 61: L1 L1 Hence, we get s = 1 t sin 2t, 4 (s2 + 4)2 6 = 3 sin 2t 8 2 + 4)2 (s
3 8

3 4

t cos 2t.

g(t) = 1 t sin 2t + 4 f (t) = e5t


1 4t

sin 2t +

sin 2t 3 t cos 2t, 4


3 8

3 sin 2t 4 t cos 2t .

Example 64 Find L1 [(1 + e2s )/(s2 + 1 )]. 4 Solution: We break F (s) into two pieces. For the rst piece, we have: L1 1 s2 +
1 4 1 = 2 sin 2 t.

Hence, by the t-shift property (20), the second piece gives us: L1 e2s 1 s2 + 4 = u(t 2) 2 sin 1 (t 2) = 2
1 = u(t 2) 2 sin( 2 t ) = 1 2 sin 2 t 0

= 2u(t 2) sin 1 t = 2 Combining the two results, we get: f (t) =

if t 2, if t 2.

2 sin 1 t if 0 t 2, 2 0 everywhere else.

The Inverse Transform

51

Example 65 Find f (t), if F (s) = (s2 + 19)/(s2 + 4)(s2 + 9). Solution: Note that only even powers of s appear in this expansion. Hence, consider the following expansion (obtained by substituting x for s2 ): x + 19 A B = + . (x + 4)(x + 9) x+4 x+9 Heavisides method gives immediately A = 3 and B = 2. Now, substituting back x = s2 we obtain 3 2 s2 + 19 = 2 2 . 2 + 4)(s2 + 9) (s s +4 s +9 Finally, it follows that f (t) =
3 2

sin 2t

2 3

sin 3t.

Example 66 Find the inverse transform of F (s) = (2s2 3)/s3 (s2 + 1). Solution: Ignore for a moment the fact that F (s) contains an odd power of s. Consider rst the expansion 2x 3 A B = + . x(x + 1) x x+1 Heavisides method gives immediately A = 3 and B = 5. Hence, substituting x = s2 , we get that 2s2 3 3 5 = 2 + 2 2 (s2 + 1) s s s +1 and hence that L1 2s2 3 = 3t + 5 sin t. s2 (s2 + 1)

The transform above is not quite F (s), but if we divide it by s (i.e., turn s2 into s3 ) we get precisely F (s). Now, division by s corresponds to integration by t; recall (16) from chapter 1, section 6. Therefore, L1 2s2 3 = s3 (s2 + 1)
t 0 3 (3 + 5 sin ) d = 2 t2 5 cos t + 5.

Example 67 Find f (t) if F (s) = s ln[(s + 2)/s] 2. Solution: This problem diers from all the other ones seen so far in that F (s) comes into two pieces. These pieces must be kept together because, separately, neither s[ln(s + 2)/s] nor 2 tends to zero as s goes to innity, which is a necessary requirement for a Laplace transform in the ordinary sense (see section 1.8). However, if we keep them together, it is easy to see that lims s ln (s + 2)/s 2 = 0; convince yourself of this. Now, consider the identity ln s+2 = s
s

1 1 +2

d.

By the methods of section 1.8, in particular equation (18), its easy to see that L 1 e2t s+2 . = ln t s

52

The Inverse Transform

Multiplication by s corresponds to dierentiation by t: hence, if we set g(t) = 1 e2t t G(s) = ln s+2 , s

and note that g(0+) = 2 (this is a good revision example on de lHospital theorem), we get immediately that s+2 L g = s G(s) g(0+) = s ln 2. s On the right-hand side we nd precisely what we need, that is, F (s). So, in the end: f (t) = 7. Convolution The Laplace transform of a sum is equal to the sum of the corresponding Laplace transforms. Unfortunately, a similar rule for multiplication does not hold: the transform of a product is not equal to the product of the transforms. Convolution is an operation that allows us to deal with products of Laplace transforms in a relatively simple way. Denition: Let f (t) and g(t) be two functions that possess a Laplace transform. The convolution of f (t) and g(t), denoted f g(t), is dened as
t

d 1 e2t dt t

1 e2t 2e2t . t t2

f g(t) =

f ( ) g(t ) d.

(25)

Example 68 Let f (t) = t and g(t) = et ; nd f g(t).


t

Solution: Applying the denition 25, we get immediately:


t

f g(t) =

et d = et
0 0

e d =

= 1 t + et ; the last step follows after integration by parts. We observe immediately that convolution is a linear operation: f (c1 g + c2 h) = c1 f g + c2 f h, where c1 and c2 are constant quantities. It also has the commutative property of usual multiplication: this is easy to show, so lets see it. Substituting =tu d = du,

The Inverse Transform

53

in the denition 25, and noting that = 0 corresponds to u = t, and = t to u = 0, we get that
t 0

f g = =

0 t 0

f ( ) g(t ) d = g(u) f (t u) du =

f (t u) g(u) du =

= g f, which completes the proof. The most interesting property of convolution may be seen when one considers its Laplace transform. From denitions (3) and (25), we derive that L [f g] =
0 t

est
0

f ( ) g(t ) d dt.

This may be seen as a double integral in the t plane: L [f g] =


0 0 t

t=

st

f ( ) g(t ) d dt.

The region of integration is innite, and is shown on the left. We introduce a new pair of variables u and v, through the denition = u, t = u + v. Clearly, the line t = is mapped into the line v = 0 in the uv plane, i.e., the u axis; similarly, the line = 0 (the t axis) is mapped into the line u = 0 (the v axis). In other words, the region of integration in the uv plane is the whole rst quadrant. Changes of variables in improper double integrals should always be treated with some care, but in this case the presence of the exponential term est guarantees that, for suciently large s, the change of variables is allowed. The Jacobian of the transformation is (, t) 1 0 = = 1. 1 1 (u, v) Switching to the new variables, as shown in the adjoining picture, we nd immediately that L [f g] = =
0 0 0

es(u+v) f (u) g(v) du dv =


0

esu f (u) du

esv g(v) dv = (26)

= L [f ] L [g].

Therefore, we have established the important result that the Laplace transform of the convolution is equal to the product of the corresponding transforms.

54

The Inverse Transform

Example 69 Find the Laplace transform of f (t) = Solution: Note that f (t) = t t; therefore L [f (t)] = L [t] L It follows immediately that
t

t 0

t d .

t .

1 ( 1/2)! t d = L t t = L [t] L t = 3/2 = 5/2 . s s 2s


t (t 0

Example 70 Find the Laplace transform of f (t) = Solution: Note that


t 0

)3 cosh d .

(t )3 cosh d = t3 cosh t; L [cosh t] = s , s2 1

since L [t3 ] = it follows immediately that

3! , s4

F (s) =

3! 6 s = 3 2 . 4 s2 1 s s (s 1)
t

Example 71 Find the Laplace transform of f (t) = 0 2 e4 d using convolution. Solution: The integral in this example is not a convolution product. Hence, we write
t

f (t) =
0

2 e4 d = e4t
0

2 e4(t ) d = e4t g(t),

where g(t) is indeed a convolution product: g(t) = t2 e4t . By (26), we obtain that G(s) = s3 (s 2 ; 4)

so nally, by the shift theorem (19) of section 1.10, we get: F (s) = G(s + 4) = 2 . (s + 4)3 s

Obviously, this problem could also be done without using convolution: F (s) = Verify that the answer is the same. 1 d2 L e4t . s ds2

In applications one often needs to identify the original function f (t), knowing its transform F (s). Therefore, it is also useful to write (26) backwards, in the form L1 F (s) G(s) = f g(t), as illustrated by the following examples. (27)

The Inverse Transform

55

Example 72 Find f (t), if F (s) = 1/s5 (s 1)2 . Solution: We write F (s) = G(s) H(s), with G(s) = H(s) = It follows immediately that 1 , s5 1 . (s 1)2

g(t) = t4 /4!, h(t) = tet ,


t 1 24

and hence that f (t) = g h(t) = = =


1 t 24 e 1 4 t 24 t 0 0

4 (t )et d =

(t 4 5 )e d =

1 + 3 t3 + 3 t2 + 4t + 5 + (t 5)et . 2

Example 73 Find L1 [1/(s2 + 2 )2 ] (again). Solution: Write 1 ; F (s) = 2 s + 2 it follows immediately that (s2 and also that f (t) = Hence L1 (s2 1 = F (s) F (s), + 2 )2 sin t .

1 = f f (t) = + 2 )2
t

=
0

sin sin (t ) d. 2

This integral may be done by means of the well-known identity sin A sin B = It follows that L1
1 2

cos(A B) cos(A + B) .
t 0

1 = 2 + 2 )2 (s =

cos (2 t) cos t d = 2 2
t t

sin (2 t) cos t = 3 4 2 2 0 0 2 sin t t cos t . = 4 3 2 2 After simplication, this may be seen to be identical to the second result of example 61.

56

The Inverse Transform

Example 74 Find u(t 2) u(t 3). Solution: Taking the Laplace transform of this function, we nd that L u(t 2) u(t 3) = Therefore, u(t 2) u(t 3) = L1 Method 2: From rst principles.
t

e2s e3s e5s = 2 . s s s

e5s = (t 5) u(t 5) = s2

0 if t < 5, t 5 if t 5.

u(t 2) u(t 3) =

u( 2)u(t 3) d

As an exercise, verify that the answer is the same.

Example 75 Find the convolution of t1/2 with itself and hence calculate ( 1/2)!. Solution: We start from the basic transform of tc ; see (9) in chapter 1. With c = 1/2, we get the equation 1 ( 1/2)! L = . s t Hence, by convolution, we get that ( 1/2)! 1 1 ( 1/2)! ( 1/2)! L = = s s s t t Expanding this equation, we obtain
t 2

d t

= ( 1/2)!

L [1]

and hence (inverting the Laplace transform)


t 0

d 2 = ( 1/2)! . t

The integral on the left-hand side is elementary: substituting = tx2 , d = 2tx dx, we get immediately 1 2 dx 2 = = ( 1/2)! , 2 1x 0 and nally ( 1/2)! = . You might be surprised to learn that f (t) 1 is not the unit of convolution product; in other words 1 f (t) is not in general equal to f (t). Indeed, the problem of nding a function (t) with the property that f (t) = f (t) for every f (t) leads straight into operational calculus, which youll encounter at a later stage of your studies.

The Inverse Transform

57

Example 76 Find 1 t. Solution: Since 1 t = t 1, we get immediately:


t

t1=
1 In other words, 1 t = 2 t2 .

d = 1 t2 . 2

8. Additional Examples Using the Laplace transform it is possible to calculate some denite integrals that cannot be done by elementary calculus. Broadly speaking, all the following examples use the same trick, i.e., an interchange (done at the right time) of the order of integration. As mentioned before, a rigorous justication of this step (though not dicult) is beyond the scope of these notes. Example 77 Calculate 0 cos tx dx/(1 + x2 ), where t is a positive parameter. Solution: Dene an auxiliary function a(t) as follows: a(t) =
0

cos tx dx. 1 + x2

Taking the Laplace transform of a(t), we get: A(s) =


0

est
0

cos tx dx dt. 1 + x2

This may be seen as a double integral over the whole rst quadrant of the xt plane (an improper integral, of course). Swapping the order of integration, we get: A(s) =
0

1 1 + x2

est cos tx dt dx.

The inner integral is just the Laplace transform of cos tx:


0

est cos tx dt =

s . s 2 + x2

Substituting this result into the preceding equation, we obtain: A(s) =


0

s dx. (1 + x2 )(s2 + x2 )

This integral is elementary. Evaluating it (partial fractions are required), one gets A(s) = s arctan(x/s) arctan x s2 1 s
1 2 0

s = 2 s 1 =
1 2

1 2

s+1

= 1 L et . 2

58

The Inverse Transform

It follows

cos tx 1 dx = a(t) = 2 et . 1 + x2 0 Corollary: Dierentiating this equation with respect to t, we get immediately that x sin tx 1 dx = a(t) = 2 et . 1 + x2 0 Example 78 Calculate (1/2)! without using convolution. Solution: Dene the auxiliary function a(t) =

etu du. dx, we get


1 = 2 t1/2 1 2

[t > 0]

1 2 ! . A(s) = s1/2 We now calculate A(s) again, this time from rst principles: 1 2

0 2 1/2 1 1/2 Substituting tu = x, which yields du = t 2x 1 a(t) = 2 t1/2 x1/2 ex dx 0 1 and hence, substituting c = 2 into (9),

!,

(28)

A(s) =
0

est
0 0

etu du dt.

Swapping the order of integration we get A(s) =


0

est etu dt du.

The inner integral is a basic transform (see (4) in section 1.3): 1 A(s) = du. s + u2 0 Integrating the right-hand side we get 1 arctan(u/s1/2 ) 2 A(s) = = 1/2 . s1/2 s 0 Comparing (28) and (29) we get
1 2 1 2 ! s1/2 2

(29)

1 2 . s1/2

Simplifying, it follows a(1) =


0 1 2

!=

Corollary: Substituting t = 1 in (28), we get

eu du =
2

1 2

which by symmetry yields

eu du =

a formula of great importance in probability theory and statistics.

The Inverse Transform

59

Example 79 Calculate Solution: We note that

c s ds/(1 0

+ s), where 0 < c < 1. 1 = L et s+1

and hence
0

sc ds = 1+s =

0 0

sc L et ds = sc
0

et est dt ds.

Swapping the order of integration, and substituting st = u, the right-hand side becomes
0

et
0

sc est ds dt =
0

et
0

uc u du e dt = tc t
0

et tc1

uc eu du dt.

In terms of generalized factorials (11), this may be written


0

et tc1 dt (c)! = (c 1)! (c)!.


0

So, nally, we get

sc ds = (c 1)! (c)! 1+s

Comment: If c is irrational, this integral may not be done by the methods of elementary calculus. However, if c = m/n with integer m and n, then the substitution s = un rationalizes the integrand. In particular, if c = 1/2 we get
0

s1/2 2 1 ds = ( 2 )! , 1+s

but the left-hand side (substituting s1/2 = u) becomes 2


0 1 It follows that ( 2 )! =

du = . 1 + u2

; the same result found in examples 75 and 78.

60

Tutorial Problems

The Inverse Transform

PROBLEMS

Elementary Inverse Transforms 18. Find the following inverse Laplace transforms. (a) L1 [s5 ] (f) L1 [( s 1)2 /s2 ] (b) L1 [8s/(s2 + 16)] (g) L1 [ /(s + 3)5/2 ] 1 (h) L1 [s/(s2 + 2s + 37] (c) L [12/(4 3s)] (i) L1 [(2s + 3)/ (s + 1)3 ] (d) L1 [(2s 5)/(s2 9)] 1 7/2 (e) L [1/s ] (j) L1 [(s + 1)2 /(s 1)3 ] Partial Fractions 19. Use partial fractions expansion to nd the inverse Laplace transform of the following functions. (d) 1/s2 (s2 + 1) (a) 1/(s2 + 4s + 5) (e) s/(s3 + 1) (b) s/(s + 1)2 3 2 2 (c) (s + 1)/(s s)(s 4) (f) (2s + 3)/(s3 + 4s2 + 5s) 20. Use partial fractions expansion to nd the inverse Laplace transform of the following functions. (a) (2s 1)/(s3 s) (f) 2(7s 31)/(s3 + 3s2 25s + 21) (b) (27 12s)/(s + 4)(s2 + 9) (g) 27s/(s + 1)(s 2)3 3 4 2 (c) (s + 16s 24)/(s + 20s + 64) (h) (s + 1)/(s2 + 2s + 2)2 2 2 (i) (s2 + 1)/s(s2 + 2)(s2 + 3) (d) s/(s 2s + 2)(s + 2s + 2) 2 (j) (5s2 18s + 15)/(s 1)(s 2)3 (e) (11s 2s + 5)/(s 2)(s + 1)(2s 1) Shifting 21. Find f (t), if F (s) is dened as follows. e3s (a) F (s) = (s + 1)2 es sinh s (b) F (s) = s 22. Find: (a) L 1 1 es + e2s s2 + 1 (b) L 1 Dirty Tricks 23. Find the following inverse Laplace transforms. Read ex. 67 before doing question (h). 2 s4 (a) L1 (e) L1 ln 4 s(s4 1) s 1 s+3 s2 + 4s + 13 (b) L1 ln (f) L1 ln 2 s s + 4s + 5 + 1 s2 + 1 (g) L1 arccot s (c) L1 ln s+4 1 s2 + 49 (h) L1 s arccot s 1 (d) L1 ln 2 2s s + 25 1 es s es/3 s(s2 + 1) 2(1 e3s ) 3(1 + e3s ) (d) F (s) = s3 s2
2

(c) F (s) =

The Inverse Transform

Tutorial Problems

61

Convolution 24. Find (1 1) sin t and 1 (1 sin t), and verify that you get the same answer. 26. Show that et et = (sinh t2 )/t. (a) L
t (t 0

25. Find: (a) et et ,


2

(b) t u(t 8),

(c) t2 u(t 5).

27. Use the convolution theorem to calculate the following Laplace transforms. )4 sin 3 d (b) L
t (t 0

)137 e4 d

28. Show that (a) cos t cos t + sin t sin t = sin t, 29. Use the convolution theorem to calculate the following inverse Laplace transforms. (a) L1 1 (s + 1)(s2 + 2s + 50) (b) L1 1 (s 1)3 (s + 1) Calculation of Integrals e2t sin2 t dt = 1 ln 2. 4 t 0 3t 6t e e 31. Show that dt = ln 2. t 0 cos 2t cos 14t dt = ln 7. 32. Show that t 0 2 x4 ex dx = 3 /8. 33. Show that 30. Show that
0

(b) cosh t cosh t sinh t sinh t = sinh t.

ANSWERS 18 (a) t4 /24 (b) 8 cos 4t (c) 4e4t/3 5 (d) 2 cosh 3t 3 sinh 3t (e) 8t5/2 /15 (a) e2t sin t (b) (1 t) et 9 7 (c) 1 2 et + 8 e2t + 24 e2t 4 3
3 (a) 1 2 et + 1 et 2 (b) 3e4t 3 cos 3t

(f) 1 + t 4 t/ (g) 4 t3/2 e3t 3


1 (h) et (cos 6t 6 sin 6t) (i) 2et (1 + t)/ t

19

(f)

(j) et (1 + 4t + 2t2 ) (d) t sin t 1 (e) 3 et/2 cos 3t/2 + 3 sin 3t/2 1 et 3
1 5

20

(c) (d) (e)

1 2 sin 4t + cos 2t sin 2t 1 2 sin t sinh t 5e2t 3 et/2 + 2et 2

21

(a) f (t) = (t 3)e(t3) u(t 3) =

0 (t 3)e(t3)

(h) 1 tet sin t 2 1 (i) 6 + 1 cos 2t 2 cos 3t 2 3 1 (j) 2et + (2 + 3t 2 t2 )e2t if t < 3 if t > 3

(f) 3et e3t 2e7t (g) et + (9t2 + 3t 1)e2t

3 + e2t (4 sin t 3 cos t)

62

Tutorial Problems

The Inverse Transform

if 0 < t < 2 0 everywhere else. 0 if t < 1/3 (c) f (t) = 1 1 cos(t 3 ) if t > 1/3 t(t 3) if 0 < t < 3 (d) f (t) = 0 everywhere else; see example 33. if t is negative, 0 sin t if 0 < t < , 22 (a) f (t) = sin 2t if < t < 2, sin 3t if t is greater than 2. t if 0 < t < , (b) f (t) = 2 t if < t < 2, 0 everywhere else. 2 cosh t + 2 cos t 4 (e) 23 (a) cosh t + cos t 2 t et e3t 2e2t (cos t cos 3t) (b) (f) t t 4t e cos t sin t (c) (g) t t t cos 5 cos 7 cos t sin t d (d) (h) 2 . t t 0 Note: The integral in 23(d) cannot be done by elementary calculus. (b) f (t) =
1 2

u(t) u(t 2) =

1 2

24 25 26 27 29 33

t sin t. (a) sinh t,


t

(b)
2

d = ? Hint: e e 0 5 2 (a) 72/s (s + 9), (a) et (1 cos 7t)/49,

1 (t 8)2 2 (t )2

u(t 8),

(c)

1 3

(t 5)3 u(t 5).

Hint: Substitute x2 = u. Recall the denition of generalized factorial.

(b) 137!/s138 (s + 4). 1 (b) 1 t2 1 t + 4 et 1 et . 2 2 4

Linear Dierential Equations

63

Chapter Three

LINEAR DIFFERENTIAL EQUATIONS

1. Linearity An expression of the form c2 x + c1 x + c0 x = f, (30) where x is an unknown function of one variable, whereas c2 , c1 , c0 and f are given functions, is a linear dierential equation of 2nd order. If the term c2 x is missing, the equation is said to be of 1st order; linear dierential equations of order 3, 4 and up, are dened in the obvious way. In this chapter well continue to use t as the independent variable, as we did in the preceding chapters. Example 80 The equation t2 x (t2 + 2t) x + (t + 2) x = t4 is 2nd-order linear. We nd that c2 = t2 , c1 = (t2 + 2t), c0 = t + 2 and f = t4 . The quantities c2 , c1 and c0 are also called the coecients of the equation. The special case where the coecients are all constant is very important in applications, and well study it in greater detail. Example 81 The equation 5 8x + 3x = sin t is a linear equation with constant coecients; x we nd that c2 = 5, c1 = 8, c0 = 3 and f (t) = sin t. Before we proceed, we shall quickly review the concept linearity in general. Linear dierential equations are members of the larger family of linear equations. Linear equations have certain important common features, which connect directly to the methods of linear algebra that you have already studied in other courses. Lets take a quick look at some of these unifying properties. Denition: An equation is called linear if it may be expressed in the form O x = f, where x is an unknown quantity, f is given, and O is an operator having the property O (s1 x1 + s2 x2 ) = s1 O x1 + s2 O x2 for all operands x1 , x2 and scalars s1 , s2 . Most books use x as independent variable, and y(x) as dependent.

64

Linear Dierential Equations

This denition is extremely general. For example: If the unknown quantity x is a column vector, and O is a matrix, then we have a system of linear equations of the kind you met in rst-year linear algebra. The systems of equations you saw in high school also fall in this group. If x is a function of t, and O a linear combination of derivatives, then we have a linear ordinary dierential equation [ODE for short] like (30), or also like examples 8081. If x is a function of t, and O x is dened through an integral like
b

O x(t) =

t2

x( ) d + 2

then we have a linear integral equation. If x is a column of functions of t, and O a matrix where every row is a linear combination of derivatives, we have a system of coupled dierential equations. We shall see examples further on (in section 3.9). If x is the angular velocity of a rigid body, f is the torque on the body, and O is its inertia tensor, then we get Eulers equations for rigid-body motion, which are particularly important for mechanical engineers. If x is a function of y and z, and O is a linear combination of partial derivatives, like 5xyy + 3xzz = f (y, z), then we have a linear partial dierential equation [PDE for short]. The list of examples is virtually endless; among the ones shown above, some are already too dicult for this course. Note that the symbol x represents a dierent mathematical object in each of these examples. As a rule, the set of all possible x that may go under the operation O, has the structure of a vector space. This space could be the familiar R3 of physics, or Rn , like in your linear algebra coursesor even more abstract, as youll see later in the course of your studies. However, the structure of the equation is the same in all cases. Several denitions are applicable to all these examples; they also have certain common properties. For example: If the right-hand side is zero the equation is called homogeneous. Solutions of an homogeneous linear equation form a vector subspace (i.e., they are closed under addition and scalar multiplication), called the kernel or null-space of O. If there is a number such that the equation O x = x has non-trivial solutions, then is called an eigenvalue of O. Every non-homogeneous linear equation may be associated with a corresponding homogeneous equation, by replacing the right-hand side with zero. If x1 and x2 are solutions of the same non-homogeneous linear equation, their dierence x1 x2 is a solution of the corresponding homogeneous equation. A quick revisitation of linear algebra will help make these ideas clear. Example 82 Consider the system 5 17 3 10 3 2 x1 x2 = 3 . 0 x3 (31)

Linear Dierential Equations

65

Simple row-reduction yields: x1 = 30 4x3 x2 = 9 + x3 , hence, writing x3 = t, we get: x1 30 4 x2 = 9 + t 1 . x3 0 1 This equation shows that the system has an innity of solutions, parametrized by t. One such solution (corresponding to t = 0) is represented by [ 30 9 0 ]T , the rst vector on the right-hand side. Other solutions are obtained by xing t to dierent values. For example x1 26 x2 = 8 , x3 1 6 0 , 9 10 1 , 10

or

or

...

are also solutions. Note that the dierence between any two of them is a scalar multiple of T [ 4 1 1 ] , which in turn is a solution of the corresponding homogeneous system: 5 3 h1 17 3 h2 = 0 , 0 10 2 h3 h1 4 h2 = t 1 . h3 1 (32)

We see that the null-space of the matrix for the system (32) is one-dimensional, and its basis T consists of the single vector [ 4 1 1 ] . Observe also that if we complement the system (31) with one auxiliary condition of the form, say, x1 = 0, then the resulting problem has a unique solution, corresponding to t = 7.5: x1 30 4 0 x2 = 9 + 7.5 1 = 1.5 . x3 0 1 7.5

Although this example is elementary, it has the same structure of the more advanced problems in dierential equations and systems of equations that we are about to study. Note, in particular, the dierence between the general solution, which contains the free parameter t, and a particular solution that meets an additional requirement. It is not surprising that dierential equations with constant coecients and systems of linear algebraic equations like (31), should be so similar. The link between them is represented precisely by the Laplace transform, which transforms linear dierential equations into plain linear equations.

66

Linear Dierential Equations

2. Linear ODEs: Preliminary Comments As a rule, a dierential equation is regarded as solved if its possible to write down the solution in terms of integrals, even if such integrals cannot be done by elementary calculus. Example 83 The function x = t (sin t/t2 ) dt is a solution (not the only one) of the equation tx x = sin t, as you may check using the product rule. However, (sin t/t2 ) dt may not be done by the rule of elementary calculus. Broadly speaking, the problem of solving ODEs tends to get more and more complicated as the order of the equation increases. For example, well see that linear equations of order 1 may always be solved, in the sense given above; on the other hand, there is no general, all-purpose method for solving arbitrary 2nd-order linear ODEs like (30), or ODEs of higher order. So, in moving from rst-order to second-order, we already encounter a major complication. However, for linear ODEs with constant coecients (recall, they are the equations where the coecients c0 , c1 , c2 etc are all constant) it is always possible to solve the equation, regardless of its order. One way to do that is by the Laplace transform. Example 84 Solve the dierential equation x + 7x + 10x = 0, given that x(0+) = 2 and x(0+) = 1. Solution: The coecients are 1, 7 and 10. Applying the Laplace transform, we get: L [x] = X, L [x] = sX 2, L [] = s2 X 2s + 1. x

Substituting these expressions into the equation, we obtain an equation for X: s2 X 2s + 1 + 7(sX 2) + 10X = 0, which yields X= s2 2s + 13 2s + 13 3 1 = = . + 7s + 10 (s + 2)(s + 5) s+2 s+5

Finally, applying the inverse transform, we get that x = 3e2t e5t . This is the solution we seek and, by Lerchs theorem, its unique.

Example 85 Solve the dierential equation y + 9y = sin 2t, given that y(0+) = 7 and y(0+) = 3. Solution: The coecients are 1, 0 and 9. Applying the Laplace transform to both sides of the equation, we get that L [y] = Y , L [] = s2 Y 7s + 3 and L [sin 2t] = 2/(s2 + 4). Hence, y s2 Y 7s + 3 + 9Y = Y = s2 2 +4

2 7s 3 + . (s2 + 4)(s2 + 9) s2 + 9

This is, by denition, the order of the highest derivative of the solution.

Linear Dierential Equations

67

Simple manipulations yield


2/ 2/ 2 5 5 = 2 2 . 2 + 9) + 4)(s s +4 s +9

(s2 Hence,

Y =

2/ 5

s2

+4

17/ 5

s2

+9

s2

7s . +9

Finally, inverting the transform, we nd that our solution is y=


1 5

sin 2t

17 15

sin 3t + 7 cos 3t.

These examples illustrate some important ideas. First of all, the equation in example 84 is homogeneous while the one in example 85 is non-homogeneous, but the Laplace transform deals with them in essentially the same way. Another point to be noted is that both examples are second-order: well see that for equations with constant coecients, problems of third and higher order may be tackled in the same way as second-order problems. Hence, most of our examples will be second-order problems. Both examples came with exactly two additional bits of information, one about the unknown and one about its rst derivative. Technically, these are called initial conditions, because they prescribe the value of two unknown functions at the same t (which was t = 0 in both examples). So, we have seen how a second-order linear ODE with constant coecients with two initial conditions may be solved completely and uniquely. If you followed so far, youll probably guess that three initial conditions would be needed for a third-order equation, four initial conditions for a fourth-order one, and so on; and your guess would be correct. An important property of linear ODEs is that, for an equation of order n, there is always one (and only one) solution satisfying n initial conditions, on the unknown and its rst n 1 derivatives, set at the same time t0 . Mathematicians call this result an existence and uniqueness theorem. Linear initial-value problems have several nice features, and this is one of them. From an engineers point of view, though, the uniqueness bit is probably the more useful one. In simple words, it means that if we have been able, somehow, to nd a solution, then we know we found the solution, because it is the only one: we have to look no further. In more advanced engineering problems, youll encounter problems where requirements must be made at dierent ts, for example at t = 0 and t = 4. These are called boundary conditions, but well not discuss them here. Finally, in some applications one may need to solve a linear dierential equation without specifying any initial conditions. This may be done by treating the initial data as free parameters. The solution obtained in this way is the general solution, as opposed to a particular solution corresponding to a set of initial conditions. It is easy to show that the solutions of a linear homogeneous ODE form a vector space (after all, they are just the kernel of a linear operator). So, nding the general solution of a linear homogeneous ODE means nding a basis for such a space. Denition: The set of all solutions of a homogeneous linear dierential equation is called the solution space of the equation.

68

Linear Dierential Equations

It is also possible to prove that the dimension of the solution space is always equal to the order of the equation: a third-order linear homogeneous ODE has always a three-dimensional solution space, a fourth-order has a four-dimensional solution space, and so on. But remember that this principle applies only to homogeneous linear ODEs; the solution set of a non-homogeneous linear ODE is never a vector space. To make this point clear, let us go back to examples 84 and 85, which are both second-order problems. Example 86 Check that any linear combination of e2t and e5t is a solution of the equation x + 7x + 10x = 0. Solution: Indeed, if we write x = Ae2t + Be5t for arbitrary A and B, then we see that x = 2Ae2t 5Be5t , x = 4Ae2t + 25Be5t . Substituting these expressions into the equation, we get: 4Ae2t + 25Be5t + 7(2Ae2t 5Be5t ) + 10(Ae2t + Be5t ) = 0, and nally, for any value of A and B, we get 0 = 0, which completes the check. On ther other hand, if the ODE is non-homogeneous, the check fails. Example 87 Go back to example 85, where the equation was y + 9y = sin 2t. Take two 1 1 solutions: for instance y1 = sin 3t + 5 sin 2t and y2 = cos 3t + 5 sin 2t are solutions. First of all, we verify that y1 and y2 are indeed solutions. We have: y1 = 9 sin 3t
4 5

sin 2t

y2 = 9 cos 3t

4 5

sin 2t.

Substituting these expressions into the equation, we obtain:


9 4 y1 + 9y1 = (9 + 9) sin 3t + ( 5 + 5 ) sin 2t = = sin 2t. 9 y2 + 9y2 = (9 + 9) cos 3t + ( 4 + 5 ) sin 2t = 5 = sin 2t.

So, both y1 and y2 yield the identity sin 2t = sin 2t, which means they are solutions. However, if we substitute a simple linear combination such as y = 7y1 3y2 into the equation, we get (7 3) sin 2t = sin 2t, i.e., 4 = 1, which is absurd. This shows that our linear combination of solutions is not a solution. 3. Linear Homogeneous ODEs with Constant Coecients The general solution of a homogeneous equation with constant coecients may be always found following a few simple rules. Well rst look at some numeric examples, after which it will be easy to see the general procedure.

Linear Dierential Equations

69

Example 88 Find the general solution of the equation x 5x 14x = 0. Solution: As the initial values of x and x are not specied, we treat them as parameters. We set x(0+) = a = free, it follows, by the Laplace transform: s2 X sa b 5X + 5a 14X = 0. Solving for X, we get: X= sa + b 5a sa + b 5a = . 2 5s 14 s (s 7)(s + 2)
1 (2a 9 1 + b) (7a b) + 9 . s7 s+2

x(0+) = b = free;

The right-hand side may be expanded by Heavisides method: X(s) =

However, we dont need to keep X in such a complicated form. Since a and b are free and independent parameters, we introduce two new parameters
1 A = 9 (2a + b)

B = 1 (7a b) : 9 A B + , s7 s+2

clearly, A and B are also free and independent. Therefore, we may write X= where A and B are free, and nally x(t) = Ae7t + Be2t ; we may leave the general solution in this form. Example 89 Find the general solution of the equation x 4x + 4x = 0. Solution: Transforming the equation, we get that s2 X sa b 4 sX a + 4X = 0, where a = x(0+) and b = x(0+). It follows that (s2 4s + 4) X = sa + b + 4a. Reasoning like in example 88, we write X= sa + b + 4a sa + b + 4a A B = = + , s2 4s + 4 (s 2)2 s 2 (s 2)2

where A and B are free parameters linked to the initial data on x and x. Inverting the Laplace transform, we get x(t) = Ae2t + Bte2t , and this is the general solution.

70

Linear Dierential Equations

THE CHARACTERISTIC EQUATION Having seen two examples, lets try to understand how the method works from a higher point of view. Consider a linear homogeneous 2nd-order ODE with constant coecients, like c2 x + c1 x + c0 x = 0 where c2 , c1 and c0 are constants. We take the Laplace transform of this equation, treating the initial data as free parameters. Eventually, we get for X an expression like (A rst degree polynomial in s) X= . c2 s2 + c1 s + c0 The denominator D(s) on the right-hand side is called the characteristic polynomial of the equation, and its obtained by replacing x with s2 and x with s in the original equation. If the characteristic equation c2 s2 + c1 s + c0 = 0 has two roots s = r1 and s = r2 , then the characteristic polynomial D(s) may be factored as This, in turn, leads to a partial fractions expansion of the form B A + , X= s r1 s r2 where A and B are free, and nally to the general solution x = Aer1 t + Ber2 t . If, on the other hand, the characteristic equation has only one root s = r, then the characteristic polynomial may be factored as which leads to a partial fractions expansion of the form A B X= + s r (s r)2 and nally to the general solution x = Aert + Btert . Conclusion: In practice one may bypass the Laplace transform, since the general solution depends only on the characteristic roots (or root): once they have been determined, the solution may be written down immediately. The extension of this procedure to higher order equations of the form cn x(n) + . . . + c2 x + c1 x + c0 x = 0. (33) is obvious, so lets see it quickly. By taking the Laplace transform of (33), one arrives at a solution of the form (A polynomial of degree n 1 in s) , (34) X(s) = D(s) where D(s) is the characteristic polynomialobtained by replacing x(n) with sn , x(n1) with sn1 , and so on and so forth, in equation (33)and the numerator depends on n free parameters, one for each initial condition. Then, D(s) is broken into factors: this step always requires nding the roots of the equation D(s) = 0, which is the characteristic equation for the given dierential equation. c2 s2 + c1 s + c0 = c2 (s r)2 , c2 s2 + c1 s + c0 = c2 (s r1 )(s r2 ).

Linear Dierential Equations

71

Denition: The roots of the characteristic equation are called characteristic roots.

If the linear factors of D(s) are all distinct we say that the roots are simple. For instance, in example 88 we saw D(s) could be written as the product of (s + 2) and (s 7) : so 2 and 7 are simple roots. But in example 89, we saw D(s) could only be written as (s 2)(s 2), so the (only) root s = 2 was not simple. Denition: If a polynomial D(s) may be written as a product like D(s) = (sr)m Q(s), where Q(s) is another polynomial and Q(r) = 0, we say that r is a root of D(s) with multiplicity m. Roots with multiplicity 1, 2 and 3 are often called simple, double and triple, respectively. Example 90 The root s = 2 in example 89 is double. The characteristic polynomial is D(s) = s2 4s + 4 and may be factored as D(s) = (s 2)2 . Example 91 The polynomial D(s) = s7 + 2s5 + s3 may be factored as D(s) = s3 (s2 + 1)2 = s3 (s i)2 (s + i)2 . Hence, s = 0 is a triple root and s = i, i are double complex roots. Example 92 Using the free computer package maxima, one may quickly see that for the polynomial D(s) = s8 11s7 + 33s6 5s5 50s4 one has D(s) = s4 (s 2)(s 5)2 (s + 1). Therefore, 2 and 1 are simple roots; 5 is a double root; 0 is a root with multiplicity 4. Note that in the last example the multiplicities add up to n = 8, which is also the degree of the characteristic equation. This is no coincidence: indeed, it is easy to prove that a polynomial of degree n has either n simple roots in the complex eld or, if some roots are multiple, the sum of all the multiplicities is n. If all the roots of the characteristic polynomial are simple, then, continuing from (34) we get an expansion of the form X(s) = A B C D + + + + s r1 s r2 s r3 s r4

and the number of terms on the right-hand side is equal to the order of the equation. Inverting the Laplace transform we get x(t) = Aer1 t + Ber2 t + Cer3 t + Der4 t + ; the coecients A, B, C, D, . . . are free parameters in the general solution. SIMPLE ROOTS: EXAMPLES Example 93 Find the general solution of the equation x(4) 10 + 9x = 0. x Solution: The characteristic polynomial is D(s) = s4 10s2 + 9 = (s2 9)(s2 1). It is really a corollary of the fundamental theorem of algebra which, however, is buried deep into the theory of complex variables.

72

Linear Dierential Equations

Its roots are r1 = 3, r2 = 3, r3 = 1, r4 = 1. Each root is simple; note that there are four roots, and the order of the equation is four. Hence, the general solution may be written x(t) = Ae3t + Be3t + Cet + Det . Comment: the general solution may also be written x(t) = M cosh 3t + N sinh 3t + P cosh t + Q sinh t; convince yourself of this. We found here two equally good bases for the solution space. Each basis has, of course, four elements. Complex exponentials (if any) may be converted to sines/cosines through Eulers formula. Example 94 Find the general solution of the equation 2 6x + 5x = 0. x Solution: The characteristic polynomial is D(s) = 2s2 6s + 5. Its roots are 9 10 3i s= = . 2 2 Therefore, the general solution may be written as 3 x(t) = c1 e(3+i)t/2 + c2 e(3i)t/2 = e3t/2 (c1 eit/2 + c2 eit/2 ). Applying Eulers formula, this becomes
1 1 x(t) = e3t/2 (c1 cos 1 t + ic1 sin 2 t + c2 cos 2 t ic2 sin 1 t) = 2 2 1 = e3t/2 (A cos 2 t + B sin 1 t), 2

where A = c1 + c2 , B = i(c1 c2 ). Note that the rst form of the general solution requires complex arithmetic, the second one does not. Example 95 Find the general solution of 3x + x 75x 25x = 0. Solution: The characteristic polynomial is D(s) = 3s3 + s2 75s 25 = s2 (3s + 1) 25(3s + 1). Therefore, the characteristic equation may be factorized as (3s + 1)(s2 25) = 0 = 3s + 1 = 0, s2 25 = 0.

The characteristic roots are s = 1/3, s = 5 and s = 5. So, nally, the general solution is x = A et/3 + B e5t + C e5t . An alternative solution is x = A et/3 + M cosh 5t + N sinh 5t (convince yourself of this).

Linear Dierential Equations

73

Solution: The characteristic equation of this 6th-order equation is s6 9s3 + 8 = 0;

Example 96 Find the general solution of the equation x(6) 9x(3) + 8x = 0.

substituting s3 = u this becomes u2 9u + 8 = 0, which factorizes as (u 1)(u 8) = 0. Therefore the characteristic roots are given by the equations s3 1 = 0 Simple manipulations yield 1 + i 3 /2 s= 1 i 3 /2 1 2 s = 1 + i3 1 i 3 3t + P sin 3t . and s3 8 = 0.

and

Therefore, the general solution in real form is x = Aet + et/2 B cos 3t/2 + C sin

3t/2 + M e2t + et N cos

MULTIPLE ROOTS: EXAMPLES If the characteristic polynomial has a double root r, and some other roots, we proceed like in example 89: going back to (34), we get X(s) = A B + + (n 2 terms that do not depend on r), s r (s r)2

where A and B are free. The general solution will then have the form x(t) = Aert + Btert + (n 2 exponentials that do not depend on r). Similarly, if the characteristic polynomial has a triple root r, and perhaps other roots, continuing from (34), we get X(s) = B C A + + + (n 3 terms that do not depend on r), s r (s r)2 (s r)3

and the general solution will have the form


1 x(t) = Aert + Btert + 2 Ct2 ert + (n 3 exponentials that do not depend on r). 1 Note, however, that the factor 2 attached to C may be removed without loss of generality, because C is free, anyway. The pattern is now clear: a quadruple characteristic root r would contribute a combination of solutions of the form Aert + Btert + Ct2 ert + Dt3 ert to the general solution, and so on. A characteristic root of multiplicity m would contribute a linear combination of m terms. Again, complex exponentials (if any) may be handled by means of Eulers formula.

74

Linear Dierential Equations

Example 97 Find the general solution of the equation x(4) 6 + 8x 3x = 0. x Solution: The characteristic polynomial is

D(s) = s4 6s2 + 8s 3 By inspection, D(1) = 0; hence s = 1 is a root. By long division, s4 6s2 + 8s 3 = s3 + s2 5s + 3. s1 By inspection, the right-hand side is zero if s = 1: hence, s = 1 is at least a double root. Again by long division, s3 + s2 5s + 3 = s2 + 2s 3. s1 Now, the right-hand side may be factored as (s 1)(s + 3). So, putting everything together: D(s) = s4 6s2 + 8s 3 = = (s 1)(s3 + s2 5s + 3) =

= (s 1)3 (s + 3).

= (s 1)2 (s2 + 2s 3) =

The last line shows that D(s) has only two roots, namely s = 1 (triple), and s = 3 (simple). Therefore, the general solution is x(t) = A + Bt + Ct2 et + M e3t , where A, B, C and M are free. Example 98 Find the general solution of the equation y (4) + 18 + 81y = 0. y Solution: The characteristic polynomial is D(s) = s4 + 18s2 + 81 = (s2 + 9)2 . We see that the roots are i3 and i3, each with multiplicity 2. Hence the general solution may be written as y(t) = A1 + A2 t ei3t + B1 + B2 t ei3t , but also (through a suitable new set of parameters) as y(t) = M + N t cos 3t + P + Qt sin 3t.

Linear Dierential Equations

75

4. The Non-Homogeneous Equation A non-homogeneous linear ODE of order n with constant coecients has the form cn x(n) + + c2 x + c1 x + c0 x = f (t). (35)

Suppose x(t) and y(t) are two solutions of (35), leaving the initial conditions un-specied. Except for the requirement that they be distinct, x and y are completely arbitrary. This, in other words, means that cn x(n) + + c2 x + c1 x + c0 x = f (t), cn y (n) + + c2 y + c1 y + c0 y = f (t) Subtracting one equation from the other, we get that cn x(n) y (n) + + c2 x y + c1 x y + c0 x y = 0. This shows that z = xy is a solution of the associate homogeneous equation: cn z (n) + + c2 z + c1 z + c0 z = 0. So, if we have determined just one solution y of (35), regardless of the initial conditions, we may generate any other solution x by simply adding to y the general solution of the associate homogeneous equation. In a sense, solving the associate homogeneous equation is the heart of the problem. Having solved that, it does not matter which special solution y we add, nor how we manage to nd it. For equations with constant coecients, the Laplace transform is often a good way to nd a special solution. As for the initial conditions, they are free, so we may set them all to zero, which seems to be the simplest choice. Solution: Begin with the associate homogeneous equation, which has characteristic polynomial D(s) = s2 2s 3. Equating this to zero yields the characteristic roots s = 1 and s = 3, both simple. Hence the general solution of the associate homogeneous equation is z = Aet + Be3t . A particular solution y of the full, non-homogeneous equation, satisfying the initial conditions y(0+) = y(0+) = 0, is then found by the Laplace transform: this gives (s2 2s 3)Y = (s 3)(s + 1)Y = 1 1 Example 99 Find the general solution of x 2x 3x = sinh t.
def

s2

1 . (s 1)(s + 1)

This statement remains true for all linear equations, but a general discussion of linear equations with variable coecients (beyond rst-order) would be too advanced for these notes.

76

Linear Dierential Equations

Solving for Y we nd Y =
1/ 1/ 3/ 1/ 1 32 8 32 8 = + + . 2 (s 3)(s 1)(s + 1) s 3 s 1 s + 1 (s + 1)2 1 3t 32 e

(36)

Inverting the transform, it follows:

y=
1 3t e 32

1 et + 8
3 t e 32

3 t 32 e

1 + 8 tet .

Finally, the general solution x = y + z may be written: x= = (A +


1 8 et + 3t 1 32 )e 1 + 8 tet + Ae3t + Bet , 1 + (B 8 )et 1 et + 1 tet . 8 8 1 32

Recall, however, that A and B are free parameters; hence, if we dene M = A + N = B 1 , we may write simply that 8 x = M e3t + N et 1 et + 1 tet , 8 8

and

where M and N are free. With hindsight, we realize that only two coecients in the partial fractions expansion (36) were needed: as a rule, whenever a number is added to, or multiplied by, an undetermined parameter, it may be dropped without loss of generality. Example 100 Find the general solution of x 3 + 3x x = t2 et . x Solution: The associate homogeneous equation has the particularly simple characteristic equation D(s) = s3 3s2 + 3s 1 = 0, which becomes showing immediately that its general solution z is a linear combination of et , t et and t2 et . Proceeding like in the preceding example, we take the Laplace transform of the nonhomogeneous equation setting all initial conditions to zero. Noting that L t2 et = we get that 2! (s 1)3 2! . (s 1)3 (s 1)3 = 0,

(s3 3s2 + 3s 1)Y (s) = Solving for Y we nd that Y (s) = inverting the transform we obtain y(t) = 2! ; (s 1)6

2! t5 et t5 et = . 5! 60 The general solution of the non-homogeneous equation is x = y + z = A + Bt + Ct2 +


1 5 60 t

et .

Example 101 Find the general solution of x(4) 5 36x = cos 2t. x 4 Solution: The associate characteristic equation is s 5s2 36 = 0, which factors immediately

Linear Dierential Equations

77

as (s2 9)(s2 + 4) = 0. Hence, the characteristic roots are 3 and i2, all simple. The general solution of the associate homogeneous equation may be written z = A cos 2t + B sin 2t + Ce3t + De3t , z = A cos 2t + B sin 2t + M cosh 3t + N sinh 3t

but also

(take your choice). To nd a particular solution y of the non-homogeneous equation, we set the initial conditions to zero. By the Laplace transform, we get that (s4 5s2 36)Y = and hence that Y = Expanding in partial fractions, we get: Y = s/169 s/169 s/13 2 2 . 29 s s + 4 (s + 4)2 (s2 s2 s , +4

s . 9)(s2 + 4)2

The last term on the right-hand side leads us back to example 61: 2 1 d s/13 = 2 2+4 + 4) 52 ds s

(s2

Therefore, inverting the Laplace transform, we get: y=


1 169

cosh 3t

1 169

cos 2t

1 52 t sin 2t.

Finally, the general solution may be written x = y + z = A cos 2t + B sin 2t + M cosh 3t + N sinh 3t Note that, since A and B are free, it is not wrong to drop the terms from the general solution (same argument as in example 99).
1 t sin 2t. 52 1 cosh 3t and 169 cos 2t

1 169

Example 102 Find the general solution of the equation x 2x = u(t) u(t 8), where u(t) is Heavisides step function. Solution: The characteristic polynomial of the associated homogeneous equation is D(s) = s2 2, with roots s = 2. Hence its solution may be written z = Ae
2t

+ B e

2t

or also

z = M cosh

2t + N sinh 2t

(the two forms are equivalent). To nd a particular solution, we set all initial conditions to zero. By the Laplace transform, we get: (s2 2)Y = 1 e8s . s

78

Linear Dierential Equations

Solving for Y , we nd: Y = By Heavisides method, we obtain that

1 e8s . s(s2 2)

1 1 s 1 = 22 2 = s(s2 2) s 2 s cosh 2t 1 =L = L sinh2 t/ 2 . 2

Therefore, Y = and hence, Finally, x = A e solution.


2t

1 2 s s2 2

1 2

1 e8s ,

y = sinh2 t/ 2 u(t) sinh2 (t 8)/ 2 u(t 8). + B e 2t + sinh2 t/ 2 u(t) sinh2 (t 8)/ 2 u(t 8) is the general

INITIAL VALUE PROBLEMS If the values at t = 0 of the unknown function are set, then one seeks a particular solution that ts the given conditions, rather than the general solution. For equations with constant coecients, in that case, probably the best option is to use the Laplace transform right from the start. The distinction between homogeneous and non-homogeneous equations then becomes un-necessary. Example 103 Solve the equation y + 9y = sin 2t, given that y(0+) = 0 and y(0+) = 1. Solution: Taking the Laplace transform of both sides of the equation, we get: s2 Y 1 + 9Y = Y = s2 s2 2 , +4

1 2 + 2 . + 9 (s + 9)(s2 + 4)

A straightforward partial fractions expansion yields


2/ 2/ 2 5 5 = 2 2 . 2 + 4) + 9)(s s +4 s +9

(s2 Hence,

Y = Finally, y=

2/ 5

s2 + 4
1 5

3/ 5

s2 + 9
1 5

sin 2t +

sin 3t

is the particular solution tting the given initial values.

Linear Dierential Equations

79

Solution: Taking the Laplace transform of both sides of the equation, we get: s2 F s 0 1 4F = Solving for F yields F = s2 + 1 . s2 (s2 4) 1 . s2

Example 104 Solve the equation f 4f = t, given that f (0+) = 0 and f(0+) = 1.

The right-hand side may be simplied observing the identity


5 1 x+1 = 4 4. x(x 4) x4 x

Therefore, F =
5 1 s2 + 1 = 2 4 42 = s2 (s2 4) s 4 s 5 8 1 sinh 2t 4 t . 5 8 1 sinh 2t 4 t.

=L

Therefore, the particular solution is f (t) =

Solution: Taking the Laplace transform of the equation, we get L [] 6L [x] + 9L [x] = 0; x by (13) and (14) we get It follows that

Example 105 Solve x 6x + 9x = 0, with the initial conditions x0 = 1, and x0 = 4.

4 s + s2 X 6(1 + sX) + 9X = 0, or (s2 6s + 9)X = s 2. Hence X= Now, from (4) we get 1 = L e3t , s3 1 1 d = L te3t ; = 2 (s 3) ds s 3 x = e3t + te3t = (1 + t)e3t . s2 s2 s3+1 1 1 s2 = = = + . 2 2 6s + 9 (s 3) (s 3) s 3 (s 3)2

and

Hence,

80

Linear Dierential Equations

5. A View From Above This section deals with more advanced topics, and may be safely skipped without aecting your understanding of the remaining sections. Leave it out if youre reading these notes for the rst time. Go back to the n-th order linear non-homogeneous ODE with constant coecients (35); suppose that n initial conditions have been specied. We saw that, applying the Laplace transform to the equation, one may eventually derive the equation: cn sn + + c2 s2 + c1 s + c0 X(s) (powers not higher than sn1 ) = F (s). (37)

The polynomial of degree n that multiplies X is, of course, the characteristic polynomial D(s). Note that all the other terms on the left-hand side of (37) may be grouped as a polynomial K(s) of degree at most n 1, which vanishes if all initial conditions are set equal to zero (convince yourself of this). Equation (37) may be written in the more compact form DX K = F, where X is unknown. Solving for X, one gets: X = D 1 K + D 1 F. Inverting the Laplace transform, it follows that x(t) = L1 [D 1 K] + L1 [D 1 F ]. Introducing a function g(t) through the equations: G(s) = D 1 g(t) = L1 [D 1 ], we may express (38) using a convolution product (see section 2.7) as follows: x(t) = L1 [D 1 K] + g f (t), (39).
def

(38)

In some other course, youll probably learn that g is called the Greens function for equation (35) with initial conditions all set to zero. Such initial conditions are called homogeneous initial conditions. Let us now interpret equations (38)(39), because they give us some insight into the structure of the solution. They tell us that the solution x(t) consists of two parts: x(t) = a(t) + b(t), where a(t) = L1 [GK], b(t) = L1 [GF ] = g f (t);

The rst part, a(t), is the solution that we would get if we kept all the initial conditions as given, but replaced the right-hand function f (t) with zero, i.e., if we solved the corresponding

Linear Dierential Equations

81

homogeneous equation. The second part, b(t), is the solution that we would obtain if we did set (n1) = 0 (which would make K(s) = 0 all initial conditions to zero, i.e., x0 = x0 = x0 = = x0 identically), but solved the full, non-homogeneous, equation. Conclusion: We see that the solution of (35) is obtained by combining the solutions of two related, but rather easier, problems. Hence the tongue-twisting statement to solve a nonhomogeneous problem one must combine the solution of the non-homogeneous equation with homogeneous initial values, and the solution of the homogeneous equation with non-homogeneous initial values. Actually, this statement remains true for all linear equations, including those with variable coecients: unfortunately, as we mentioned before, it is impossible to give a method for solving all linear homogeneous equations with variable coecients in a closed form (that is, leaving out computer-generated solutions). Example 106 Find the solution of x + x = tan t that satises the homogeneous initial conditions x(0+) = x(0+) = 0. Solution: We note that D(s) = s2 + 1, hence the Greens function is g(t) = L1 Therefore, x = sin t tan t =
t 0

s2
t

1 = sin t. +1

sin(t ) tan d =

=
0

(sin t cos cos t sin ) tan d.

Recalling that cos tan d = cos + C, we get immediately that x = sin t [1 cos t] cos t [ sin t + artanh(sin t)] = sin t cos t artanh(sin t). sin tan d = sin + artanh(sin ) + C,

Example 107 Solve 5 8x + 3x = sin t, subject to the homogeneous initial conditions x x(0+) = 0, x(0+) = 0. (This is example 81.) Solution: The characteristic polynomial is D(s) = 5s2 8s + 3, and the roots are s = 1 and s = 3/5. Hence, 1/ 1/ 1 2 2 . = G= 3 s1 s 3 5(s 1)(s 5 ) 5 The Greens function is g(t) = L1
1/ 2 1/ 2

s1

3 5

1 = 2 (et e3t/5 ).

82

Linear Dierential Equations

Clearly, K(s) = 0. Therefore:


t

x = g f (t) = 1 (et e3t/5 ) sin t = 2


t

0 1 2

sin
t 0

1 2

et e3(t )/5 d =

1 2

et
0

e sin d

e3t/5

e3 /5 sin d.

Integrating by parts, we get: x(t) = g f (t) =


1 2

et

1 2

sin + +

1 2

cos e
1 34

t 0

1 2

e3t/5

15 34

sin +

25 34

cos e3 /5

t 0

= 1 et 4

25 3t/5 68 e

2 17

cos t

sin t

THE INDEPENDENCE OF SOLUTIONS We have shown that any solution of a linear homogeneous ODE with constant coecients of order n may always be expressed as a linear combination of n functions. When the characteristic equation has n simple roots, these functions are just n exponentials; when there are multiple roots, each exponential is multiplied by powers of t ranging from t0 to tm1 , where m is the multiplicity. Since the multiplicities always add up to n, one always nd n functions. We took for granted that such functions form an independent set, but we never proved it. The proof is actually quite easy and it may be seen as follows. Consider the easier case where all the characteristic roots are simple. Suppose there are n constants C1 . . . Cn such that C1 er1 t + C2 er2 t + + Cn ern t 0. Then (by taking the Laplace transform of this identity), it follows immediately that C2 Cn C1 + + + = 0. s r1 s r2 s rn If you multiply both sides of this equation by s r1 , simplify and let s = r1 , you get C1 = 0. Do the same for every term: you get Ck = 0 for every k, which proves that the functions are independent. The case where some roots are multiple is handled in exactly the same way (but do it, as an exercise). In summary, we have proven that every linear homogeneous ODE with constant coecients of order n has an n-dimensional solution space. This theorem remains true in a more general context, where the coecients are functions of t. However, the proof for equations with variable coecients (using Wronskian determinants) falls beyond the scope of these notes. 6. First Order Equations: Variation of Parameters We now turn to linear rst-order dierential equations with non-constant coecients, i.e., equations of the form c1 (t) x(t) + c0 (t) x(t) = f (t), (40) obtained by letting c2 0 in the equation (30) that we saw at the beginning of this chapter. The Laplace transform is denitely not the method of choice for such equations. [for all t].

Linear Dierential Equations

83

There is a very nice method for solving equation (40), and it is known as Lagranges method or the method of variation of parameters. It consists in looking for a solution that has the form x(t) = u(t) v(t), where both u and v are initially un-specied. Dierentiating (41), we get x = uv + uv; substituting back into (40), we obtain: c1 uv + c1 uv + c0 uv = f (t). (42) (41)

This result is true in general, for arbitrary u and v. Now we choose v so that the expression in square brackets vanish: c1 uv + c0 uv = 0. c1 v + c0 v = 0.

(43)

If v is chosen in this way, and substituted back into (42), then only the part outside the brackets remains: c1 uv = f (t), (44)

and this may be integrated immediately. Thats it! Youll recognize that (43) is the homogeneous equation associated with (40). However, it is also separable: dv = c0 v, c1 dt hence dv c0 = dt v c1 and dv = v c0 dt. c1

Therefore, v too may be determined by an ordinary integration. Finally, having found v from (43) and hence u from (44), we reconstruct x according to (41). Note that two integrations are performed, hence apparently two integration constants should be accounted for. But the integration constant from the intermediate step (nding v) falls o: keener students should verify this statement. For practical purposes, the integration constant for the intermediate step may generally be set to zero. As an exercise, set it to 137 and verify that a factor e137 eventually falls o.

84

Linear Dierential Equations

Example 108 Find the general solution of t3 x + 5t2 x = et . Solution: Write x = uv, x = uv + uv, and substitute: it follows that t3 uv + t3 uv + 5t2 uv = et . Impose that the part in brackets vanish: it follows that t3 uv + 5t2 uv = 0, which simplies to dv 5 dt = . v t ln |v| = 5 ln |t|. t3 u t5 = et . This yields immediately: u = t2 et u= Putting everything together: x(t) = uv = t2 et 2tet + 2et + C t5 = t3 et 2t4 et + 2t5 et + Ct5 . t2 et dt = t2 et 2tet + 2et + C

Integrating this equation, we get Hence we may let v = t5 and substitute into what is left of the equation, which is not much:

Example 109 Find the general solution of the equation (1 + et )y + 2et y = sinh t. Solution: Write y = uv, y = uv + uv, and proceed like in the preceding example. It follows that (1 + et )u v + (1 + et )u v + 2et u v = sinh t; imposing that (1 + et )u v + 2et u v = 0 we get the equation for v, which is v 2et = . v 1 + et 1 . (1 + et )2

This may be solved immediately: ln |v| = 2 ln |1 + et | Substituting back, we get the equation for u: 1 + et u = sinh t (1 + et )2 = u = (1 + et ) sinh t = sinh t + 1 (e2t 1). 2 = v=

Linear Dierential Equations

85

Integrating, we nd that u = cosh t + 1 e2t 1 t + C, 4 2 and nally that y = uv = cosh t + 1 e2t 1 t + C 4 2 . (1 + et )2

Example 110 Find the general solution of x cos t x sin t = t2 in a neighborhood of t = 0. Solution: Write x = uv, x = uv + uv, and substitute: it follows that uv cos t + uv cos t uv sin t = t2 . Impose uv cos t uv sin t = 0, and simplify: it follows that dv sin t dt = . v cos t dv sin t dt d(cos t) = = , v cos t cos t ln |v| = ln | cos t|; In the vicinity of t = 0, we nd that | cos t| = cos t. Therefore, we set v= substituting back, we get: u cos t = t2 , cos t and nally x = uv = u = t2 ,
1 3 t 3

Integrating, we get

1 ; cos t

u = 1 t3 + C, 3

+C . cos t

Comment: Note that x(t) becomes innite for t = 1 , regardless of C. That is why one must 2 specify in a neighborhood of t = 0 in the statement of the problem. Example 111 Find the solution of x + x cot t = 5ecos t , satisfying the initial condition x(/2) = 4. Solution: Proceeding like in the previous examples, one nds uv + uv + uv cot t = 5ecos t and separating the part in brackets, d(sin t) dv = cot t dt = . v sin t

86

Linear Dierential Equations

Integrating, it follows that ln |v| = ln | sin t| 1 v= . sin t Substituting back: u = 5ecos t , sin t u = 5 The general solution is x = uv = sin t ecos t dt = 5 ecos t d(cos t)

hence = 5ecos t + C. 5ecos t + C . sin t

Imposing the initial condition x(/2) = 4, we get 5ecos /2 + C = 4, sin /2 5+C = 4, 1 hence C = 1, and nally the required particular solution: x= 5ecos t 1 . sin t

7. Higher Order Equations with Variable Coecients: The Taylor Series Method When the coecients are not constant, most second-order equations cannot be solved in a simple way; linear equations with constant coecients, of course, are a pleasant exception. Sometimes a solution may be obtained in the form of a power series which can be useful for further calculations, even if it cannot be expressed in terms of elementary functions. Before we go any further, let us see how the method works. Example 112 Find the solution of the eqaution y + t2 y = 0. Solution: Note that this 2nd-order equation has a variable coecient (namely t2 , attached to y); none of the methods seen so far is applicable. We assume y(t) may be expressed as a series of powers of t, like the following: y(t) = a0 + a1 t + a2 t2 + a3 t3 + a4 t4 + a5 t5 + , where the coecients a0 , a1 , a2 . . . are numbers that will determined at a later stage. Dierentiating (45) twice, we get y(t) = a1 + 2 a2 t + 3 a3 t2 + 4 a4 t3 + 5 a5 t4 + ; (45)

y (t) = 2 a2 + 3 2 a3 t + 4 3 a4 t2 + 5 4 a5 t3 + .

Linear Dierential Equations

87

We substitute these expressions into the dierential equation. We get: 2 a2 + 3 2 a3 t + 4 3 a4 t2 + 5 4 a5 t3 + 6 5 a6 t4 + 7 6 a7 t5 + + +t2 (a0 + a1 t + a2 t2 + a3 t3 + a4 t4 + a5 t5 + ) = 0. Grouping similar monomials, we get: 2 a2 + 3 2 a3 t + (4 3 a4 + a0 )t2 + (5 4 a5 + a1 )t3 + (6 5 a6 + a2 )t4 + (7 6 a7 + a3 )t5 + +(8 7 a8 + a4 )t6 + (9 8 a9 + a5 )t7 + (10 9 a10 + a6 )t8 + (11 10 a11 + a7 )t9 + = 0. We now impose that, on the left-hand side, each coecient attached to a power of t be individually equal zero. This will obviously make the whole left-hand vanish for every t. We get a set of equations like: 2 a2 = 0 6 5 a6 + a2 = 0 10 9 a10 + a6 = 0 3 2 a3 = 0 7 6 a7 + a3 = 0 11 10 a11 + a7 = 0 4 3 a4 + a0 = 0 8 7 a8 + a4 = 0 12 11 a12 + a8 = 0 5 4 a5 + a1 = 0 9 8 a9 + a5 = 0 (and so on.)

Although we have an innity of equations, we may solve them one-by-one. For instance, the rst equation yields a2 = 0, and the second equation gives a3 = 0. So, we have immediately found two coecients. The third equation is dierent: it has two unknowns, 4 3 a4 + a0 = 0, so one of them must be treated as a parameter. We write a0 = A = free, a4 = A , 43 B . 54

and carry on. The fourth equation, like the previous one, has two unknowns, so we write a1 = B = free, a5 =

At this point we easily see that no more free parameters will be needed, because the next equations yield: a6 = a2 =0 65 a7 = a3 =0 76 a8 = a4 A = 87 8743 a9 = a5 B = , 98 9854 B , 13 12 9 8 5 4

and then a10 = 0 a11 = 0 a12 = A , 12 11 8 7 4 3 a13 =

and so on. Clearly, an = 0 if n divided by 4 leaves a remainder of 2 or 3. The pattern in which the coecients may be formed is now clear, but the solution must be left in the form y =A 1 t8 t12 t16 t4 + + + 4 3 8 7 4 3 12 11 8 7 4 3 16 15 12 11 8 7 4 3 t5 t9 t13 t17 +B t + + + 5 4 9 8 5 4 13 12 9 8 5 4 17 16 13 12 9 8 5 4

(46)

88

Linear Dierential Equations

We have obtained an algorithm for calculating the solution with arbitrary accuracy, but we cannot express it in a simple way in terms of elementary functions such as sines, cosines, polynomials etc. In case you wonder, it may be shown that the solution belongs to the family of Bessel functions, which are very important in advanced engineering mathematics. How reliable is a solution obtained in this way? Lets stay with example 112, and let A = 1 and B = 0. Consider an approximate solution of the form y= 1 t4 43 ,
0.5

1.5 1

obtained by truncating the series solution after only two non-zero terms. The picture on the right, obexact 0 tained by the free package gnuplot, shows on the same scale the exact solution and the approximate soapproximate lution. Clearly, the dierence between the two (which 0.5 is zero for t = 0) remains very small if t is not too big. Using our knowledge of series, we can make a simple 1 0 0.5 1 1.5 2 2.5 3 estimate the error. If t is positive, both series appearing in (46) are alternating series where the error is less than the rst term left out. For instance, if t = 0.1 the relative error is less than 1 in 67 billion (verify this) but if t = 1 this estimate drops to 1 in 672. This may still be acceptable for some applications; however, for t = 2.5 the approximate solution y is clearly useless. In that case, more terms of the expansion (46) would be needed, in order to get a useful solution. In applications, one usually seeks a solution that is accurate to a certain number of decimal digits within a certain interval. Once such accuracy has been set, its easy to see that if the range of |t| is increased, then more and more coecients an in (45) must be found and stored in a computer memory, in order to achieve the desired accuracy. The reliability of the series solution (45) goes down as t goes up. As a rule, a power series like (45) is always associated with a radius of convergence: this means that it will diverge if |t| exceeds a certain critical value, and even below such critical value it might become numerically unreliable. As engineers, you should always be very cautious when using power series, especially if the radius of convergence is unknown. There are however series of the form (45) that converge for every t, in which case we say the radius of convergence is innite. As it happens, the series obtained in example 112 does converge for every t, but we shall not discuss this detail here. Example 113 The exponential series et = 1 + t2 t3 t4 t + + + + 1! 2! 3! 4!

converges for every t, hence its radius of convergence is innite. On the other hand, the geometric series 1 = 1 + t + t2 + t3 + t4 + 1t diverges if t = 1, hence its radius of convergence is 1.

Linear Dierential Equations

89

Now, let us put this example in a general context. Denition: A function f (t) is called analytic at a point c if it may be expressed as a power series of the form f (t) = a0 + a1 (t c) + a2 (t c)2 + a3 (t c)3 + a4 (t c)4 + for all t that are suciently near to c. Clearly, the power series (45) corresponds to the special case where c = 0. Example 114 Mercators Series, which you encountered in rst year, (t 1)2 (t 1)3 (t 1)4 + + 2 3 4 diverges for t = 0 but converges between 0 and 2. Hence, ln t is analytic at t = 1. ln t = (t 1)

We shall not discuss the theory of analytic functions in these notes. One point is worth mentioning, though: in rst year you studied Taylors Formula, which allows you to expand a function as a power series using the values of its derivatives at a given point. If a function may be expanded in a Taylor series about c, then its obviously analytic at c; it may be shown that the converse is also true, but well skip the proof. In other words, a function is analytic at a given point if and only if it may be expanded in a Taylor series about that point. Analytic functions have several important properties. The ones that are crucial for the method of this section are: For a given c, the coecients an are uniquely determined, and Power series may be integrated or dierentiated term-by-term inside their interval of convergence. Using these properties, it may be shown that if a linear ODE has an analytic solution at a certain point, and initial conditions are given in the usual way at the same point, then its possible to determine the coecients of the expansion, one after the other, as we have done in example 112. The rst if should not be taken for granted. Indeed, some of the most useful equations in engineering do not meet this requirement. In such cases, the Taylor series method must be modied, but youll learn about these ideas (Frobenius method, Fuchs theorem) in other courses. Example 112 was perhaps easy to follow because the equation was relatively simple. In general, though, it is better to use a more ecient notation, like in the following examples. Example 115 Find a solution of the equation t + 3y ty = 0. y Solution: We write (45) in summation form: y(t) =
n=0

an tn .

To save ourselves some time, let us agree that from now on, whenever the beginning and the end of a sum are not indicated, itll be understood that the dummy index runs from zero to innity. For example, dierentiating this equation we get: y(t) =
n

n an tn1 n(n 1) an tn2 .

y(t) =
n

90

Linear Dierential Equations

Substituting these expressions in the original equation, we get: n(n 1) an tn1 + 3 n an tn1 an tn+1 = 0.
n

Now, the rst two sums in this expression may be combined into one, but the third sum may not, because the power tn+1 is not the the same as tn1 . Make sure you understand this point. It follows that n(n 1) + 3n an tn1 n(n + 2) an tn1 an tn+1 = 0.
n

Simplifying, we get: an tn+1 = 0.


n

(47)

We now note that the powers of t in the two sums in (47) dier by 2 units. If in the rst sum we write n = 2 + m, then n = 0 corresponds to m = 2, and n corresponds to m . So, the rst half of (47) may be rewritten as n(n + 2) an tn1 =
n

(m + 2)(m + 4)am+2 tm+1 .

m=2

We then separate from the sum on the right, those terms that correspond to a negative m:

(m + 2)(m + 4)am+2 t

m+1

m=2

= 0 2 a0 t

+ 1 3 a1 t +

(m + 2)(m + 4)am+2 tm+1 .

m=0

Going back to (47), we dont touch the second half, except that we replace the dummy variable n with m. So, (47) becomes 0 a0 t1 + 3 a1 + (m + 2)(m + 4)am+2 tm+1 am tm+1 = 0.
m

Now both sums contain identical powers of t and run from zero to innity, so we may at last combine them into one. It follows that 0 a0 t1 + 3 a1 + (m + 2)(m + 4)am+2 am tm+1 = 0.

Proceeding like in example 112, we impose that all monomials vanish separately. Again, this yields an innite set of equations: 0 a0 = 0 3 a1 = 0 (m + 2)(m + 4)am+2 am = 0

[m = 0, 1, 2, 3, . . .]

The rst equation is always true, regardless of a0 . Hence a0 may take any value: its a free parameter. The second equation is satised only if a1 = 0. The other ones yield am+2 = am . (m + 2)(m + 4) [m = 0, 1, 2, 3, . . .]

Linear Dierential Equations

91

In this way, all even coecients may be computed one after the other. For example, we get a2 = a0 24 a4 = a0 2446 a6 = a0 244668 a8 = a0 . 2 4 4 6 6 8 8 10

Similarly, from a1 = 0 we get that a3 = 0, hence a5 = 0, and so on: all odd coecients vanish. It is easy to spot a pattern: a2 = a0 , 2 1! 2! 2 a4 = a0 , 4 2! 3! 2 a6 = a0 , 6 3! 4! 2 a8 = a0 , 8 4! 5! 2 a10 = a0 , 10 5! 6! 2 etc.

So, we may write our solution in compact form: y = a0 t2m , 22m m! (m + 1)!

where a0 is free. This is another member of the Bessel functions family.

We mentioned in section 3.3 that the solution space of a 2nd-order linear homogeneous ODE is two-dimensional. Indeed, in example 112 we found two solutions. In this example, instead, we found only one solution (apart from the scalar factor a0 ). In other words, there are two independent solutions but we found only one. This is an unavoidable limitation of the Taylor series method: the other solution was not found because it is not analytic at t = 0. A full discussion of this topic would lead us too far away. In the next example, two independent solutions are found. Example 116 Find the solutions of the equation t 3x + t5 x = 0. x Solution: We repeat all the steps of the preceding example. We write (45) in compact form: x(t) = Then, dierentiating this equation we get: x(t) =
n n=0

an tn .

n an tn1 n(n 1) an tn2 .

x(t) =
n

Substituting these expressions in the dierential equation, we get: n(n 1)an tn1 3 nan tn1 +
n n

an tn+5 = 0.

Again, we may combine the rst two sums; simplifying, we get: n(n 1) 3n an tn1 +
n

an tn+5 = 0,
n

n(n 4) an t

n1

+
n

an tn+5 = 0.

(48)

92

Linear Dierential Equations

We now note that the powers of t in the two sums in (48) dier by 6 units. Therefore, we must write n = 6 + m in the rst sum: in this way n = 0 corresponds to m = 6, and n corresponds to m . In other words, we write: n(n 4) an tn1 =
m=6

(m + 6)(m + 2) am+6 tm+5 .

Now, from the right-hand side we separate the terms that correspond to a negative m:

m=6

(m + 6)(m + 2)am+6 tm+5 = 0 (4)a0 t1 + 1 (3)a1 t0 + 2 (2)a2 t + 3 (1)a3 t2 + + 4 0 a4 t3 + 5 1 a5 t4 +

(m + 6)(m + 2)am+6 tm+5 .

m=0

Going back to (48), in the second sum we replace the dummy index n with m. So, after minor simplications, (48) becomes 0a0 t1 3a1 4a2 t 3a3 t2 + 0a4 t3 + 5a5 t4 + (m + 6)(m + 2)am+6 tm+5 +
m m

am tm+5 = 0.

The sums in this equation contain identical powers of t, and m runs from zero to innity in both, hence we may combine them. It follows that 0a0 t1 3a1 4a2 t 3a3 t2 + 0a4 t3 + 5a5 t4 + We impose that: 0a0 = 0 3a1 = 0 4a2 = 0 3a3 = 0 (m + 6)(m + 2)am+6 + am tm+5 = 0.
m

0a4 = 0 5a5 = 0 (m + 6)(m + 2)am+6 + am = 0 We deduce that a0 = free, a4 = free,

[m = 0, 1, 2, 3, . . .]

because they are multiplied by zero. We also see immediately that it must be a1 = 0, a2 = 0, a3 = 0, a5 = 0.

All other coecients are determined by the equation am+6 = am (m + 6)(m + 2) [m = 0, 1, 2, 3, . . .]

Linear Dierential Equations

93

We obtain: a6 = and a10 = a4 a4 = 10 6 60 a16 = a10 a0 = 16 12 11520 a22 = a16 a0 = 22 18 4561920 (etc) a0 a0 = 62 12 a12 = a6 a0 = 12 8 1152 a18 = a12 a0 = 18 14 290304 (etc)

All the other coecients vanish: am = 0 if m leaves a remainder of 1,2,3 or 5 when divided by 6. The solution may be left in the form x = a0 1 t12 t18 t16 t22 t10 t6 + + + a4 t4 + + , 12 1152 290304 60 11520 4561920

where a0 and a4 are free. Clearly the numbers quickly become unwieldy as m grows, but it would be fairly easy to program a computer to calculate them up to any desired order. If a solution is a polynomial, the algorithm will stop automatically, once the whole polynomial has been determined. Example 117 Find the solutions of the equation (1 + t4 ) 8 t3 z + 20 t2 z = 0. z Solution: Proceeding like in the preceding examples, we write z(t) =
n

an tn , nan tn1 ,
n

z(t) = z (t) =
n

n(n 1)an tn2 .

Substituting these expressions in the dierential equation, we obtain: n(n 1)an tn2 + n(n 1)an tn+2 8 nan tn+2 + 20
n n

an tn+2 .

The second, third and fourth sums may be combined. It follows: n(n 1)an tn2 +
n

n(n 1) 8n + 20 an tn+2 = 0,
n

n(n 1)an tn2 +

(n2 9n + 20) tn+2 = 0.

We now replace n with m + 4 in the rst sum, and n with m in the second sum. It follows:

(m + 4)(m + 3)am+4 tm+2 +


m

m=4

(m2 9m + 20) tm+2 = 0.

94

Linear Dierential Equations

Separating the rst four terms from the rst sum, we obtain (after minor simplications): 0 a0 t2 + 0 a1 t1 + 2 a2 t0 + 6 a3 t1 +

(m + 4)(m + 3)am+4 tm+2 +


m

m=0

(m2 9m + 20) tm+2 = 0.

At this point the two sums may be combined. We get: 0 a0 t2 + 0 a1 t1 + 2 a2 t0 + 6 a3 t1 +


m

(m + 4)(m + 3)am+4 + (m2 9m + 20)am tm+2 = 0.

We impose that 0 a0 = 0 0 a1 = 0 2 a2 = 0 6 a3 = 0 (m + 4)(m + 3)am+4 + (m2 9m + 20)am = 0 The rst two equations yield that a0 = free, Similarly, the third and fourth equations yield that a2 = 0, All other coecients are determined by the equation am+4 = (m2 9m + 20) am (m + 4)(m + 3) (m 4)(m 5) am = (m + 4)(m + 3) a3 = 0. a1 = free. [m = 0, 1, 2, 3, . . .]

[m = 0, 1, 2, 3 . . .]

Note that, for m = 0, 1, 2, 3, . . . and so on, the denominator of the fraction above is always positive (we never divide by zero). This guarantees that this algorithm may be used indenitely to calculate each group of four ans from the preceding four. For example, substituting m = 0, 1, 2 and 3, we get: a4 = 20 a0 , 12 a5 = 12 a1 , 20 a6 = 6 a2 = 0, 30 a7 = 2 a3 = 0. 42

But when we substitute m = 4, 5, 6 and 7 we nd that a8 = 0 a4 = 0, 56 a9 = 0 a5 = 0, 72 a10 = 2 a6 = 0, 90 a11 = 6 a6 = 0. 110

Since we found that four consecutive ans are zero, it is obvious that all the following ones must also be zero. The algorithm stops; the solution, therefore, is simply z = a0 1 5t4 3 + a1 t 3t5 5 ,

Linear Dierential Equations

95

where a0 and a1 are free. Comment: In this example, and in all examples where the solution is a polynomial, the Taylor series method yields the exact solution, rather than an approximation. 8. Linear ODEs with Variable Coecients: Special Techniques REDUCTION OF ORDER Sometimes, for a given linear homogeneous equation, one may (somehow) nd fewer independent solutions than the order of the equation; in other words, one may get an incomplete solution. For instance, we have seen in the preceding section (example 115) that the Taylor series method may fail in this way. In cases like this, it is always possible to use the incomplete solution to get a new dierential equation of a lower order. The method is a simple generalization of Lagranges method of variation of parameters. One substitutes x(t) = u(t) v(t), x being the original unknown, and v the incomplete solution. Simplifying, one gets an equation of lower order for u. If the original equation was second order, the resulting equation is rst-order linear, which may then be solved. Example 118 Solve completely the equation (t + 1) x (3t + 4) x + (2t + 3) x = 0 given that it admits the solution v = et . Solution: Let x = u et x = (u + u) et x = ( + 2u + u) et . u Substituting back and simplifying, we get: (t + 1) ( + 2u + u) (3t + 4) (u + u) + (2t + 3) u = 0 u

(t + 1) u (t + 2) u = 0.

Note that u has fallen o: this is the main feature of the method of reduction of order, and it may be used as a check on the calculations. In other words, if u doesnt fall o there must be a mistake. The last equation is separable; we nd that du t+2 = dt = u t+1 Integrating, it follows immediately: ln |u| = t + ln |t + 1| + C, u = A et (t + 1), where A = eC is a free constant. Integrating again by parts, it follows that u = A tet + B. Finally, recalling that x = u et , we nd that the general solution is x = A te2t + B et . 1+ 1 t+1 dt.

96

Linear Dierential Equations

3 Example 119 Solve completely the equation t x (1 + t) x + ( 4 t1 + 1 ) x = 0 given that it 2 admits the solution v = t. Solution: Let x = t u(t); dierentiating this expression, one gets

x = t1/2 u

1 x = 2 t1/2 u + t1/2 u

1 x = 4 t3/2 u + t1/2 u + t1/2 u.

Substituting back into the equation yields:


1 4 t1/2 u + t1/2 u + t3/2 u 1 1/2 2t 1 u + t1/2 u + 1 t1/2 u + t3/2 u + 3 t1/2 u + 2 t1/2 u = 0. 2 4

Simplifying, one gets immediately t3/2 u t3/2 u = 0 u = u. The last equation is separable, and it yields immediately u = A et = u = A et + B.

Hence, the complete solution is x = uv = A tet + B t.

This procedure is applicable to non-homogeneous linear equations as well, provided an incomplete solution of the associated homogeneous equation is available. If the original equation was second-order, then the resulting equation is rst-order linear and therefore integrable (see section 3.6). Example 120 Solve completely the equation t2 y (t2 + 2t) y + (t + 2) y = t4 given that the associate homogeneous equation has a solution of the form z = t. (This is example 80.) Solution: First of all, note that the associate homogeneous equation t2 z (t2 + 2t) z + (t + 2) z = 0. may be also solved by the Taylor series method; its actually a good revision problemit yields the general solution. Do it as an exercise. In this example, lets pretend that the incomplete solution z = t has been found by inspection. Hence, we let y = u t, where u is the new unknown. Dierentiating, it follows: y = u t + u, substituting back into the equation, we nd: t2 ( t + 2u) (t2 + 2t) (u t + u) + (t + 2)u t = t4 . u Simplifying, we get: t3 u + (2t2 t3 2t2 )u + 0 u = t4 , u u = t. y = u t + 2u.

Linear Dierential Equations

97

Note again that u has fallen o, as expected. We now have a rst-order linear equation in u, which may be solved by the method variation of parameters (see section 3.6). The letters u and v have alredy been used, though, so instead of (41) we write u = p q, It follows that p q + p q p q = t; solving for q one nds q = et . Substituting back, one gets p = t et , which (integrating by parts) yields p = tet et + C. Hence, u = p q = (tet et + C) et = t 1 + Cet . One last integration yields
1 which nally gives the general solution y = u t = 2 t3 t2 + Ctet + Bt.

and hence

u = p q + p q.

u = 1 t2 t + Cet + B, 2

EULER LINEAR EQUATIONS The homogeneous Euler equation is a linear ODE of the form cn tn x(n) + . . . + c2 t2 x + c1 t x + c0 x = 0, where c0 , c1 , c2 etc. are constant. This equation is easy to recognize because the nth derivative of the unknown is always multiplied by xn . An important property of the homogeneous Euler equation is that it always admits a solution of the form x = A tm , where m is a constant to be determined, and A is free. Substituting x = tm , x = mtm1 , x = m(m 1)tm2 , and so on,

one gets an equation for m, called indicial equation. The degree of the indicial equation always matches the order of the dierential equation. If the roots are all simple, then the general solution is a linear combination of all the solutions found in this way. Example 121 Solve the equation 2t2 x 9tx + 12x = 0. m Solution: Let x = t ; dierentiate with respect to t and substitute back into the equation. It follows immediately: 2m(m 1)tm2+2 9mtm1+1 + 12tm = 0. The term Euler equations in engineering has at least three dierent meanings, as there are also Euler equations for rigid body motion, and Euler equations for ideal uids. The Euler equations discussed here are sometimes called equidimensional Euler or also Euler-Cauchy.

98

Linear Dierential Equations

Cancelling the common factor tm , one gets the indicial equation: 2m(m 1) 9m + 12 = 0, 2m2 11m + 12 = 0. Solving for m, we obtain m = 11 therefore x(t) = At4 + Bt3/2 . 25 /4, i.e., m = 4 or m = 3/2. The general solution is

Example 122 Solve the equation t3 x + t2 x 6t x + 6x = 0. Solution: Proceeding as in the previous example, we get the indicial equation: m(m 1)(m 2) + m(m 1) 6m + 6 = 0. This equation is cubic as expected, because the dierential equation is third-order. However, its easy to spot that a term (m 1) may be immediately factored out: m(m 1)(m 2) + m(m 1) 6(m 1) = 0 (m 1) (m2 m 6) = 0. Hence either m = 1, or m2 m 6 = 0; the latter yields m = 3 and m = 2. The general solution is therefore x(t) = At + Bt3 + ct2 . There are two possible problems with this method: complex roots and multiple roots. Complex roots are handled by means of Eulers formula, as next example shows. Example 123 Solve the equation t2 y + 3ty + 10y = 0. Solution: Look for solutions of the form y = tm ; this leads to the equation m(m 1) + 3m + 10 = 0, i.e., m2 + 2m + 1 = 9.

Its solutions are m = 1 i3; hence the general solution of this dierential equation has the form y = At1+i3 + B 1i3 . To interpret the imaginary powers, substitute t = eln t and use Eulers formula. This yields: ti3 = eln t and ti3 = eln t
i3

= ei3 ln t = cos(3 ln t) + i sin(3 ln t). = ei3 ln t = cos(3 ln t) i sin(3 ln t).

i3

So, the general solution may be written y = At1 cos(3 ln t) + Bt1 sin(3 ln t).

When the equation for m has multiple roots, the methods explained above yield an incomplete solution. However, reduction of order is always an option. Example 124 Solve completely the equation t2 x 5tx + 9x = 0. m Solution: Setting x = t and proceeding like in examples 121123, we get: m2 6m + 9 = 0.

Linear Dierential Equations

99

The indicial equation has the double root m = 3. Therefore, x = t3 is a solution. We need a second solution; to nd it, we let x = t3 u(t) and proceed by reduction of order. Substituting x = 3t2 + t2 and x = 6t u + 6t2 u + t3 u and simplifying, we get the equation u + t u = 0, which is separable: du = u Integrating again, we get u = A ln |t| + B, and nally x = A t3 ln |t| + B t3 , which is a linear combination of two independent solutions, i.e., the general solution Example 125 Solve completely the equation 4t2 x 16tx + 25x = 0. dt t = u= A . t

Solution: We get the indicial equation 4 m2 5m + 25 = 0, which has the double root m = 5 . 4 2 Letting x = t5/2 u(t) and proceeding by reduction of order (do it, as an exercise), one eventually gets the general solution in the form x = A t5/2 ln t + B t5/2 . It may be shown that if the indicial equation has a double root m = k, then the corresponding Euler equation has always two solutions of the form tk and tk ln t. Well look into the proof, because it is a good revision example in the calculus of several variables. Consider a generic 2nd-order Euler equation: c2 t2 x + c1 t x + c0 x = 0 where c2 , c1 and c0 are constant. We want to see if it has a solution of the form x = tm ln t. This means we want to see under what conditions the equation c2 t2 d2 (tm ln t) d(tm ln t) + c1 t + c0 (tm ln t) = 0 dt2 dt

holds identically for every t. The crucial point is the observation that tm ln t = tm ; m

convince yourself of this. So, the previous equation is equivalent to c2 t2 d2 tm d tm tm + c1 t + c0 = 0. 2 dt m dt m m

Since dierentiation with respect to m and dierentiation with respect to t are interchangeable, this may also be written d tm d2 tm + c0 tm = 0, + c1 t c2 t2 m dt2 dt

100

Linear Dierential Equations

and hence

c2 m(m 1)tm + c1 mtm + c0 tm = 0. m

(49)

Now, if the indicial equation has two simple roots m = k and m = l, then it may be written c2 (m k)(m l) = 0. If, however, has only one double root m = k, then it must have the form c2 (m k)2 = 0. In this second case, (49) becomes c2 (m k)2 tm = 0, m 2c2 (m k) tm + c2 (m k)2 tm ln t = 0. Its evident that the left-hand side is identically zero if m = k. Therefore the function x = tk ln t ts the equation, i.e., it is a solution. As an exercise, show that this is not true in the case of two simple roots. Conclusion: If m = k is a double root of the indicial equation, then the Euler equation has two independent solutions of the form tk and tk ln t. This proof may be easily adapted to Euler equations of 3rd-order (where the indicial equation may have three simple roots, or a double root plus a simple root, or one triple root), and higher order. The details are essentially the same. For instance, it may be shown that if m = k is a triple root, then the Euler equation has three independent solutions of the form tk , tk ln t and tk (ln t)2 . Similarly, if m = k is a quadruple root, then one gets the expressions listed above, plus a fourth solution of the form tk (ln t)3 . The rule is extended in the same way to roots of higher algebraic multiplicity. Example 126 Solve completely t3 x + t x + 1. Solution: The indicial equation is m(m 1)(m 2) + m 1 = 0, m3 3m2 + 3m 1 = 0, i.e., (m 1)3 = 0. Hence, the general solution is x = A t + B t ln t + C t(ln t)2 . Example 127 Find all the solutions of t4 x(4) + 4t3 x + t2 x + t x x = 0 [for t > 0]. m(m 1)(m 2)(m 3) + 4m(m 1)(m 2) + m(m 1) + m 1 = 0, which factors immediately as (m 1)[m3 m2 m + 1] = (m 1) [m2 (m 1) (m 1)] = (m 1)3 (m + 1) = 0. Hence, we nd two roots, m = 1 (triple) and m = 1 (simple). The general solution is x = A t + B t ln t + C t(ln t)2 + D t1 . Four terms, of course.

Solution: We get the indicial equation

Linear Dierential Equations

101

THE LAPLACE TRANSFORM Occasionally, if the coecients are polynomials in t, the Laplace transform may be used because multiplication of x(t) by t corresponds to dierentiation of X(s). In this way, a dierential equation of order n for x(t) is turned into a dierential equation of order m for X(s), where m is the highest power of t appearing in the original equation. If m < n this may be a step toward the solution. Example 128 Solve t (t + 1)x + x = 0. x

Solution: This is a second-order linear equation with non-constant coecients. Writing L x(t) = X(s),

L x(t) = sX(s) a,

L x(t) = s2 X(s) sa b,

and recalling the operator identity (17) t we get It follows that 2sX s2 X + a + X + sX sX + a + X = 0, (s2 s)X + (3s 2)X = 2a. We have obtained a rst-order linear equation for X, which may be solved by the method of variation of parameters: writing X(s) = U (s)V (s) and substituting, it follows s(s 1)U V + s(s 1)U V + (3s 2)U V = 2a. Imposing that the part in brackets vanish: s(s 1)U V + (3s 2)U V = 0, canceling U and solving, we get dV = V Integrating, it follows that V = 2 3s ds = s(s 1) 1 , (s 1)s2 2 1 + s s1 ds. d , ds

d 2 d s X sa b + sX a sX a + X = 0. ds ds

102

Linear Dierential Equations

and substituting back: s(s 1)U Hence,

1 = 2a. (s 1)s2

U = 2as U = as2 + c, where c is the integration constant. We then nd X: X = UV = as2 + c = (s 1)s2 a+c c c = . s 1 s s2

Since a has not been specied anyway, we may re-dene a + c; nally, we get x = L1 X = c1 et c2 (t + 1). A REVISION EXAMPLE Example 129 Find the general solution of t2 y 2y = ln t. Solution: This is a non-homogeneous equation. The associate homogeneous equation is t2 z 2z = 0, which is of the Euler type. It has solutions of the form tm , where m is given by the indicial equation m(m 1) 2 = 0. Solving it, we nd m = 2 or m = 1. We proceed by reduction of order, setting y = uv, where v = t2 or v = t1 . Either substitution would work; let us say, y = t2 u. It follows that y = t2 u + 4tu + 2u; substituting into the original equation, we obtain: t2 (t2 u + 4tu + 2u) 2t2 u = ln t. Simplifying, we get: t4 u + 4t3 u = ln t. This is rst-order linear in u, so (by the method of variation of parameters) we set u = wx, Substituting back, we get: t4 wx + t4 wx + 4t3 wx = ln t. We impose that the expression in square brackets vanish, and we get an equation for x : tx + 4x = 0. Solving it, we nd x = t4 ; substituting back we get an equation for w: t4 wt4 = ln t. u = wx + wx.

Linear Dierential Equations

103

Solving this one, we nd w = t ln t t + C. We now have an equation for u: u = wx = (t ln t t + C) t4 = = t3 ln t t3 + Ct4 . Integrating, we get:
1 u = 1 t2 ln t 1 t2 + 1 t2 3 Ct3 + B = 2 4 2 1 = 2 t2 ln t + 1 t2 + At3 + B, 4

where A and B are free. Finally, from y = uv, we get y = ut2 = 1 ln t + 2 and this is the general solution. 9. Systems of Linear ODEs Systems of linear equations with constant coecients may be handled by the Laplace transform exactly as you would expect: by transforming the system of dierential equations in x, y, z, . . . into a system of algebraic equations in X, Y , Z, . . .. Initial conditions (if given) are accounted for in a natural way; there is really nothing new to learn. The homogeneous and non-homogeneous systems correspond, respectively, to homogeneous and non-homogeneous equations, and all the comments made in sections 3.25 apply to systems as well. Example 130 Solve the system x + 2x + y = et y x = 0, with initial values x(0+) = y(0+) = 0.
1 4

+ At1 + Bt2 ,

Solution: Taking the Laplace transform of the system, we get sX + 2X + Y = 1/(s + 1) sY X = 0. This is a system of 2 algebraic equations with 2 unknowns, described by the augmented matrix s + 2 1 1/(s + 1) , 1 s 0 where the coecients are parametrized by s. Solving it by Cramers rule, we get: 1/(s + 1) 1 0 s s/(s + 1) s = 2 = . X(s) = s + 2s + 1 (s + 1)3 s+2 1 1 s Simplifying, it follows that X(s) = 1 1 s+11 = . 3 2 (s + 1) (s + 1) (s + 1)3

104

Linear Dierential Equations

In the same way, we get: s + 2 1/(s + 1) 1 0 1 Y (s) = . = (s + 1)3 s+2 1 1 s Finally, inverting the Laplace transforms, we obtain: x(t) = tet 1 t2 et , 2
1 y(t) = 2 t2 et .

Example 131 In theoretical mechanics, Eulers equations for a free symmetric rotor may be 1 = k2 reduced to a system of the form where k is a constant determined by the rotors 2 = k1 , principal moments. Solve the system, given 1 (0+) = 1, 2 (0+) = 0. Solution: Taking the Laplace transform of the equations, we get s1 1 = k2 s2 = k1 . It follows by Cramers rule that 1 k 0 s s 1 (s) = , = 2 s + k2 s k k s s 1 k 0 k 2 (s) = = 2 , s + k2 s k k s and nally that 1 (t) = cos kt, 2 (t) = sin kt.

Example 132 The system

51 = 36x1 + 4(x2 x1 ) x describes the motion of a double 0.82 = 4(x2 x1 ) x harmonic oscillator [if for example the oscillating masses are 5 kg and 0.8 kg and the two springs have stiness 36 N/m and 4 N/m, respectively]. Given that initially x1 (0) = x2 (0) = x1 (0) = 0, and x2 (0) = 1.25 m/s, determine the motion of the system. Solution: Taking the Laplace transform of the equations of motion, we get 5s2 X1 = 40X1 + 4X2

0.8(s2 X2 + 1.25) = 4X1 4X2 .

Linear Dierential Equations

105

Simple manipulations yield the augmented matrix 5s2 + 40 4 The determinant of the coecient matrix is D(s) = (5s2 + 40) (0.8s2 + 4) 16 = 4s4 + 52s2 + 144 = 4(s2 + 9)(s2 + 4). Solving the system by Cramers rule, we get 0 4 1 0.8s2 + 4 4 X1 (s) = = . 2 + 9)(s2 + 4) 2 + 9)(s2 + 4) 4(s 4(s Expanding in partial fractions, we get X1 (s) = and nally x1 (t) = Similarly, we get: 5s2 + 40 0 4 1 5s2 40 = . X2 (s) = 4(s2 + 9)(s2 + 4) 4(s2 + 9)(s2 + 4) Expanding in partial fractions, we obtain: X2 (s) = nally:
1 x2 (t) = 12 sin 3t 1 2 1 4 1 15 1 5

4 0 . 2 0.8s + 4 1

s2 + 9

1 5

s2 + 4
1 10

sin 3t

sin 2t.

s2

+9

s2

1 , +4

sin 2t.

The determinant of this system is (s 8) 10 2 7 (s + 9) 2

x(0+) = 2, x = 8x 7y 7z Example 133 Solve the system y = 10x 9y 10z with initial values y(0+) = 1, z(0+) = 0. z = 2x + 2y + 3z, Solution: Taking the Laplace transform of the system and rearranging the system we get (s 8)X + 7Y + 7Z = 2 10X + (s + 9)Y + 10Z = 1 2X 2Y + (s 3)Z = 0 7 (s + 9) 10 7 10 = (s 8) + 10 2 (s 3) 2 (s 3) 7 7 +2 (s 3) (s + 9) 7 = 10

106

Linear Dierential Equations

= (s 8)(s2 + 6s 7) + 70(s 1) 14(s 1) = (s 1) (s 8)(s + 7) + 56 = s(s 1)2 . Solving for X by Cramers rule we get: 2 1 0 7 7 (s + 9) 10 2 (s 3) 9 7 (2s + 7)(s 1) 2s + 7 = . = = 2 2 s(s 1) s(s 1) s(s 1) s1 s x(t) = 9et 7. Similarly, we nd that (s 8) 2 7 10 1 10 2 0 (s 3) 11 10 (s + 10)(s 1) s + 10 = = = Y (s) = 2 2 s(s 1) s(s 1) s(s 1) s1 s and (s 8) 7 2 10 (s + 9) 1 2 2 0 2(1 s) 2 2 2 Z(s) = = = = . s(s 1)2 s(s 1)2 s(s 1) s s1 It follows immediately that y(t) = 11et 10 and z(t) = 2 2et .

X(s) =

It follows immediately that

Dierential equations of order n, and systems of n equations of order 1, are two sides of the same coin, so to speak, as the following example illustrates. Example 134 Write the third-order dierential equation x 3 + 3x x = t2 et , as a system x of rst-order equations. Solution: Dene x = y, x = y = z; we get immediately x = y, y=z z = 3z 3y + x + t2 et ; x 0 0 1y + 0 . 3 z t2 et

this may be re-written in matrix form

x 0 1 y = 0 0 1 3 z

It is possible to show that the eigenvalue equation for the square matrix appearing above is formally the same as the characteristic equation D(s) = 0 of the original equation: specically, D(s) = s3 3s2 + 3s 1 = (s 1)3 ,

Linear Dierential Equations

107

and

0 1

1 3

0 1 = ( 1)3 . 3

Note that the only root is s = 1, with algebraic multiplicity 3. Go back to example 100 for a solution of this problem. In exactly the same way, an equation of order n may be converted into a system of n rstorder equations (convince yourself of this). If, in addition, the equation is linear with constant coecients, then its characteristic equation is converted into the eigenvalue equation for the corresponding matrix. Therefore, the study of dierential equations of order n may be seen as a special case of the theory of linear systems of order 1: this is an important result from a theoretical point of view, but in practice, solving a single equation of order n requires probably no more work than solving the corresponding n n system. So, converting high-order equations into rst-order systems is not necessarily a step forward towards the solution. 10. Convolution Integral Equations Equations of the form
t 0

K(t ) x( ) d = f (t),
t 0

x(t) +

K(t ) x( ) d = f (t),

where f (t) and K(t) are given functions, x(t) is unknown, represent an important class of integral equations. The function K(t) is called the kernel of the equation. Recalling that the Laplace transform of a convolution product is the (ordinary) product of the tranforms, we see immediately that the Laplace transform is well suited for this kind of problems. Leaving aside questions such as existence and uniqueness of solutions and other theoretical features, let us see how the method works in practice. Example 135 Solve the integral equation Solution: The equation may be written et x(t) = sin t; taking the Laplace transform of the equation, we get X(s) 1 = 2 . s1 s +1 It follows immediately X(s) = and hence x(t) = cos t sin t. s1 , s2 + 1
t t e x( ) d 0

= sin t.

108

Linear Dierential Equations

Example 136 Solve the integral equation x(t) = cos t + Solution: Transforming the equation x(t) = cos t + t x(t) we get X(s) = It follows immediately X(s) = and nally x(t) =
1 2

t (t 0

) x( ) d .

s X(s) + 2 . 2 1+s s

1 1 s s s3 = 22 + 22 , (s2 + 1)(s2 1) s +1 s 1

cos t +

1 2

cosh t.
t 0

Example 137 Solve the integral equation x(t) = t + Solution: Transforming the equation

sin(t ) x( ) d .

x(t) = t + sin t x(t), we get X(s) = and hence X(s) = Finally, the solution is x(t) = 1 t3 + t. 6 11. Additional Examples All the special tecniques discussed so far may be extended in a natural way to equations of higher order and to systems. The next couple of examples deal with third-order equations. Example 138 The equation t3 x 3t2 x + t(6 t2 )x (6 t2 )x = 0 admits an incomplete solution of the form x = t. Find the general solution. Solution: The incomplete solution is not hard to spot: the term (6 t2 ) appearing twice, gives it away. So, letting x(t) = t u(t) and dierentiating, we get: x = tu + u It follows immediately: t3 (tu + 3) 3t2 (t + 2u) + t(6 t2 )(tu + u) (6 t2 )tu = 0. u u x = t u + 2u x = t u + 3. u X(s) 1 + , 2 s 1 + s2

1 + s2 1 1 = 4 + 2. 4 s s s

Linear Dierential Equations

109

Simplifying, we get: t4 u t4 u = 0, u u = 0. Having started with a third order equation, we have obtained a second order equation in u, which may be solved immediately: u = A cosh t + B sinh t, where A and B are free. Integrating again, we nd that u = A sinh t + B cosh t + C. So, nally, the general solution is x = At sinh t + Bt cosh t + Ct. Example 139 Apply the Taylor series method to the equation t2 x + tx + x = 0. Solution: Proceeding like in section 3.7, we write: x=
n

an tn ,

x=
n

an ntn1 ,

x=
n

an n(n1)tn2 ,

x=
n

an n(n1)(n2)tn3 .

Substituting these expressions into the equation, we get: an n(n 1)(n 2)tn1 + an ntn +
n n

an tn = 0.

Substituting n = m + 1 in the rst sum, and grouping the other two sums, we get:
m=1

am+1 (m + 1)m(m 1)tm +

am (m + 1)tm = 0.
m

Detaching the rst term of the rst sum, it follows: 0 a0 t1 + am+1 (m + 1)m(m 1) + am (m + 1) tm = 0.

This equation is identically satised if 0 a0 = 0 and am+1 (m + 1)m(m 1) + am (m + 1) = 0 [m = 0, 1, 2, 3, . . .] The rst equation is satised regardless of the value of a0 . Proceeding with the other equations, we nd that the next two are special: 0 a1 + a0 = 0 0 a2 + 2a1 = 0 am+1 (m + 1)m(m 1) + am (m + 1) = 0 The rst equation from this set xes a0 , regardless of the value of a1 : a0 = 0. [m = 0 ] [m = 1 ] [m = 2, 3, . . . ]

110

Linear Dierential Equations

So, a0 is not free. Similarly, the second equation xes a1 , regardless of the value of a2 : a1 = 0. If m > 1, then certainly m(m 1) in not zero, hence simplifying we obtain the algorithm am+1 = am m(m 1) [m = 2, 3, . . .]

which yields all other coecients. However, there is no equation for a2 , hence a2 is free. For instance, we get a3 = a2 21 a4 = + a2 3221 a5 = a2 433221

and so forth. The solution thus found may be written x = a2 t2 t4 t5 t6 t7 t3 + + + . 2! 1! 3! 2! 4! 3! 5! 4! 6! 5!

Note that the given equation must have three independent solutions, and we only found one. This means that the other solutions do not have a Taylor series about x = 0. Example 140 A simple harmonic oscillator having mass m = 4 kg and spring constant = 16 N/m, initially at rest, is pulled by a constant force f0 = 8 N for 6 seconds and then released. Describe the subsequent motion. Solution: Let y be the displacement from equilibrium. The equation of motion is m + y = y 0 f0 0 if t < 0, if 0 < t < 6, if t < 0.

Substituting m = 4, = 16 and f0 = 8 and simplifying, we obtain the equation y + 4y = 2 u(t) u(t 6) , where u(t) is Heavisides step function. The initial conditions are: y(0+) = 0, y(0+) = 0. The transformed equation is (s2 + 4)Y (s) = 2 It follows: Y (s) = Now, we observe that L1 2 = 2 + 4) s(s
t 0 1 sin 2 d = 2 (1 cos 2t).

1 e6s . s

2(1 e6s ) . s(s2 + 4)

Linear Dierential Equations

111

Then, by the t-shift property (20), we get


1 y(t) = 1 u(t) (1 cos 2t) 2 u(t 6) (1 cos 2(t 6)) = 2 if t < 0, 0 1 if 0 < t < 6, = 2 (1 cos 2t) 1 (1 cos 2t) 1 1 cos 2(t 6) if t > 6. 2 2

1 Applying the identity cos A cos B = 2 sin 1 (A + B) sin 2 (A B), we nally get: 2

Note that the solution is continuous throughout.

0 y(t) = sin2 t sin 6 sin(2t 6)

if t < 0, if 0 < t < 6, if t > 6.

Next example is a continuation of the preceding one. Example 141 Suppose the oscillator considered in example 140 is initially at 2 m from the equilibrium point, moving toward it with speed 3 m/s. Starting at time t = 0, the oscillator is pulled by a constant force f0 = 8 N for 6 seconds and then released. Describe the subsequent motion. Solution: The equation of motion is the same as in example 140; only the initial conditions dier. So, let us call x the solution of this problem, and y the solution found in example 140. The associate homogeneous equation is 4 + 16z = 0. and its general solution is quickly z found to be z = A cos 2t + B sin 2t (convince yourself of this). So, the general solution of this problem is x(t) = A cos 2t + B sin 2t + y(t), where y(t) is the solution found in example 140. We now require that x(0+) = 2 and x(0+) = 3. Recalling that y(0+) = 0 and y(0+) = 0, these conditions yield: A cos 0 + B sin 0 + 0 = 2, 2A sin 0 + 2B cos 0 + 0 = 3.

3 It follows immediately that A = 2 and B = 2 ; nally, the complete solution is

Example 142 Solve completely the equation t + 2x + tx = 0. x Solution: Taking the Laplace transform of the equation, and writing x(0+) = a, x(0+) = b, we get dX d 2 s X sa b + 2sX 2a = 0, ds ds

0 x = y + z = 2 cos 2t 2 cos 2t

3 2 3 2

sin 2t + sin t sin 2t + sin 6 sin 2(t 3)

if t < 0, if 0 < t < 6, if t > 6.

112

Linear Dierential Equations

which yields s2 X 2sX + a + 2sX X = 0, a X = . 1 + s2 Imposing that X() = 0, it follows that X = a Finally, by the methods of chapter 2, we get x(t) = a sin t . t
s

d . 1 + 2

Since in the process of nding X(s) the constant b dropped o, this solution is incomplete. So, we write sin t v(t) = t and x = uv, = x = uv + uv, = x = uv + 2uv + u. v Replacing these expressions in the original equation, we get t(v + 2uv + u) + 2(uv + uv) + tuv = 0, u v or u tv + 2u(tv + v) + u(t + 2v + tv) = 0. v Now, use the fact that v = sin t/t is a solution of the equation, i.e. that t + 2v + tv = 0. v Simplifying, we get 2(vt + v) 2 cos t u = = . u vt sin t This is immediately integrable: ln |u| = 2 ln | sin t| + ln |c|, where c is a constant, hence u= c , sin2 t = u = c cot t + a,

where a is another constant. Finally, we put u and v together: x = (c cot t + a) cos t sin t sin t = c +a . t t t

Linear Dierential Equations

113

We now have two independent solutions, namely sin t/t and cos t/t. Note that the second solution, cos t/t, does not have a Laplace transform [the integral (3) would diverge]: this is why we failed to nd it by such a method. Example 143 Solve the system of integral equations x(t) = et +

t 0

Solving this system, we get X(s) = Y (s) = Finally, we get

y(t) = t 0 (t Solution: Transforming the system, we get X(s) = 1 + X(s) Y (s) , s1 s s1 Y (s) = 1 X(s) + Y (s) . s2 s2 s

x( ) d

t (t ) e y( ) d, 0 t ) x( ) d + 0 y( ) d.

1 1 (s 1)2 /s 1 s2 s + 1 = = 2 2 . (s 1)3 1 s(s 2)(s2 s + 1) s s2

s2 s + 1 1 s(s 1) + 1 = = , 3 1 2 s + 1) (s 1) (s 2)(s s2

x(t) = e2t , y(t) =


1 2 1 2 e2t .

114

Tutorial Problems

Linear Dierential Equations

PROBLEMS

Homogeneous Linear ODEs with Constant Coecients 34. Find the general solution of the following linear ODEs with constant coecients. (a) x 7x + 10x = 0 (c) x 6 + 12x 8x = 0 x (b) x(4) + 8x(2) + 16x = 0 (d) 4 8x + 1 = 0 x 35. Find the general solution of the following linear ODEs with constant coecients. (a) x(6) + 300x(4) + 30 000 x(2) + 1 000 000 x = 0 (b) x(8) 8x(4) + 16x = 0 (c) x + 8x = 0 (d) x x + 81x 81x = 0 Non-Homogeneous Linear ODEs with Constant Coecients 36. Find the general solution of the following linear ODEs with constant coecients. (g) x(4) + 4x(3) + 6x(2) + 4x(1) + x = et sin t (e) x x = et (h) x + 11 x 11x = e11t x (f) x + x 6x = t e2t Linear ODEs with Constant Coecients: Initial-Value Problems 37. Solve the following initial-value problems. (a) x + x = et , x0 = 1 (b) x + x = 0, x0 = 1, x0 = 0 (c) x 2x + 2x = 1, x 0 = x0 = 0

(d) x + 2x + x = t2 , x0 = 1, x0 = 0 (e) x + 4x = t, x0 = 1, x0 = 0 (f) x 2x + 5x = 1 t, x 0 = x0 = 0

38. Solve the following initial-value problems. 1 if 0 < t < 2, (a) x + 9x = 0 everywhere else; x(0+) = 0, x(0+) = 0. (b) x + 2x = t if 0 < t < 1, 0 everywhere else; x(0+) = 0. A View From Above 39. For the following problems, use the Greens function method (39) to nd the particular solution that satises the initial conditions y(0+) = y(0+) = 0. (a) y y = tanh t, (c) y y = 1/ cosh3 t, (b) y + y = 1/ cos t, (d) y + y = 1/(1 + cos t). Linear First-order ODEs with Variable Coecients 40. Find the general solution of the following equations. (a) x x/t = t (d) cosh2 t y y = tanh2 t 1/t (b) tx 3x = e (e) (t2 + 1)y + ty = (1 2t) t2 + 1 (f) (1 + t2 )y + y = arctan t (c) (t2 1)x + tx = t 1

Linear Dierential Equations

Tutorial Problems

115

41. Solve the following initial-value problems (a) (1 + t)4 y + 3(1 + t)3 y = 1 + t + t2 , y(0) = 0 (c) x x tan t = t, x(0) = 8 3 2 (b) tx x = t + 3t 2t, x(1) = 4 (d) x ln t + 2x/t = 1, x(e) = 3 42. Show that Lagranges method of variation of parameters (i.e., substituting x = uv) works just as well for the so-called Bernoulli equation ax + bx = cxp , where a, b and c are functions of t and p is a (constant) power. The rst-order linear equation corresponds to the special case p = 0 of the Bernoulli equation. 43. Solve the following Bernoulli equations. (a) tx 2x = 3t2 x1 , (b) x tanh tx = tx2 . Taylor Series Method 44. Use the Taylor series method to solve the following equations. (a) x x = 0 (d) x t2 x 2t x = 0 (b) t y + y 4t y = 0 (e) t2 y 2t y + (2 + t2 )y = 0 (c) (t t2 ) z 3z + 2z = 0 (f) z t z + 6z = 0 Reduction of Order 45. Find the general solution of (t 4) (t 3)x + x = 0, given that it has an incomplete x t solution of the form x = e . 46. Find the general solution of x + (1 2 tanh2 t) x = 0, given that it has an incomplete solution of the form x = 1/ cosh t. 47. Find the general solution of x t2 cos t + x t(t sin t 2 cos t) + x (2 cos t t sin t) = 0, given that it has an incomplete solution of the form x = t. 48. Find the general solution of x cosh2 t + 2x = 0, given that it has an incomplete solution of the form x = tanh t. 49. Verify that the equation t (t + 2)x + 3x = 0 has a solution of the form x = t3 . Hence, x using reduction of order, show that the general solution is x = At3 + Bt3 (et /t4 ) dt. 50. Find the general solution of t3 x 3t2 x + t(6 + t2 )x (6 + t2 )x = 0, given that it has an incomplete solution of the form x = t. Euler Equidimensional Equations 51. Solve the following Euler equations. (c) t2 x + 5tx + 4x = 0 (a) 2t2 x + 3tx x = 0 2 3 (d) t2 x + tx + 49x = 0 (b) t x2t x5tx+5x = 0 (e) t3 x + 170tx 170x = 0 3 (f) t x + 3t2 x + tx = 0 (c) 3tx (1 + t sin t)x = 3x4 sin t.

52. Solve the following Euler equations. (a) (t + 4)2 x + 6(t + 4) x + 6x = 0 Hint: Substitute t + 4 = . (b) (t + 2)2 x + (t + 2) x + 2 x = 0 Hint: Substitute t + 2 = . 53. Double complex roots: solve t4 x(4) + 6t3 x(3) + 11t2 x(2) + 5t x(1) + 4 x = 0. Special Techniques 54. Take the Laplace transform of the following equations, solve for X(s) and hence nd x(t). (a) t + (t 1)x x = 0; x0 = 5, x0 = 5. x (b) (2t + 1) 2x (2t + 3)x = 0; x0 = 0, x0 = 1. x

116

Tutorial Problems

Linear Dierential Equations

55. Take the Laplace transform of the following equations, solve for X(s) and hence nd the general solution. (a) t 2x tx = 0 x (b) tx x tx + x = 0. 56. Solve problem 54(a) by the Taylor series method and verify that you get the same answer. Systems of ODEs 57. Solve the following initial-value problems. x0 = 2 x0 = 1 x = 5x y, x + y = 0, (c) (a) y = 3x + y, y + x = 0, y0 = 1 y0 = 1 x0 = 1 x y 2x + 2y = 1 2t, x = 2x + y, (d) (b) x + 2y + x = 0, y = 5x + 4y, y0 = 3 58. Solve the following initial-value problems. x0 = 1 x = 2x 2y 4z, (a) y = 2x + 2y 2z, y0 = 1 z = 5x + 2y + 7z z0 = 1

x0 = x0 = 0 y0 = 0 x0 = 0 y0 = 0 z0 = 2

(b)

x x + y + 2y = 1 + et , y + 2y + z + z = 2 + et , x x + z + z = 3 + et

Convolution Integral Equations 59. Solve the following equations.


t t

(a) x(t) = 1+ 1 6
t

(t )3 x( ) d
0

(c)
0

cosh(t )x( ) d = t
t 0

(b)
0

cos(t )x( ) d = t + t2

(d) x(t) = t + 2

t sin(t ) x( ) d

60. Solve the following system of integral equations. t t x(t) = 2 (t ) x( ) d 4 y( ) d


0 0 t t

ANSWERS 34 35

y(t) = 1

x( ) d

(t ) y( ) d

(a) Ae2t + Be5t (b) A cos 2t+B sin 2t+Ct cos 2t+Dt sin 2t

(c) e2t (A + Bt + Ct2 ) (d) et (A cosh 3t/2 + B sinh 3t/2)

(a) (A + Bt + Ct2 ) cos 10t + (D + Et + F t2 ) sin 10t (b) (A + Bt) cosh 2t + (C + Dt) sinh 2t + (E + F t) cos 2t + (G + Ht) sin 2t (c) A e2t + B et cos 3t + C et sin 3t (d) A et + B cos 9t + D sin 9t.
1 (e) Aet + Bet + 2 tet

36 37

(g) et (A + Bt + Ct2 + Dt3 ) + et sin t


1 2t 25 te

(f) Ae2t + Be3t + (a) (t + 1)et

1 2 2t 10 t e

(h) Aet + Bet Ce11t +

1 11t 120 t e

(b) cos t (c) 1 + 1 et (sin t cos t), 2 2

1 (e) 1 t + cos 2t 8 sin 2t 4 1 3 (f) 25 1 t 25 et 3 cos 2t 4 sin 2t 5

(d) t2 4t + 6 et (5 + t)

Linear Dierential Equations

Tutorial Problems

117

38

(a) x =

39 40

+ sin 3t sin 6 cos 3t) if t 2. if t 0, 0 2t 1 + 2t)/4 if 0 < t < 1, (b) x = (e 2t e (1 + e2 )/4 if t 1. (a) y = sinh t+cosh tarctan(sinh t) (b) y = t sin t + cos t ln | cos t|
2

1 9 (1 cos 3t) 1 9 (cos 3t cos 6

if t 0, if 0 < t < 2,

1 (c) y = 2 sinh t tanh t (d) y = sin t ttan 1 t +cos tln 2

1 1 2+2

cos t

(a) ct + t

(d) ce

41 43

1 (a) ln |1 + t| + 2 t2 /(1 + t)3 1 (b) 2 t3 + 3t2 2t ln t + 1 t 2

(b) ct3 e1/t (t 2t2 + 2t3 ) (c) 2/ t 1 + c/ t2 1

(a) x = t2 C 3/t2 (b) x = sinh t/(cosh t t sinh t + C) (c) x = 3 t/(Cecos t + 3)

(c) t tan t + 1 + 7/ cos t (d) t/ ln t (t 3)/(ln t)2

tanh t 2 tanh t 2 (e) t t2 + c / t2 + 1 (f) arctan t 1 + ce arctan t

tanh t

44 (a) x = a0 1 +

t4 t5 t3 t2 + + + a1 t + + + = a0 cosh t + a1 sinh t 2! 4! 3! 5! 4 6 2 t t t + + + ; the second solution cannot be found by the (b) y = a0 1 + 2 2 (1!) (2!) (3!)2 Taylor series method alone. (c) z = a0 1 + 2 t + 1 t2 + a4 t4 + 2t5 + 3t6 + 4t7 + 5t8 + . 3 3 t3 t6 t9 t7 t10 t4 (d) x = a0 1 + + + + + a1 t + + + + 3 36 369 4 4 7 4 7 10 t5 t7 t6 t8 t4 t3 (e) y = a1 t + + + a2 t2 + + = a1 t cos t + a2 t sin t 2! 4! 7! 3! 5! 7! 2 4 (f) z = a0 1 3t + t + 15t5 15t7 15t9 15 3t11 15 3 5t13 15 3 5 7t15 5t3 + +a1 t 3! 5! 7! 9! 11! 13! 15! x = A (sinh t + t/ cosh t) + B/ cosh t x = At sin t + Bt x = A(t tanh t 1) + B tanh t x = At cos t + Bt sin t + Ct. (a) A t + B/t (b) At + Bt + C/t (c) (A + B ln |t|) t2
5

45 46 47 48 50 51

x = A(3 t) + Bet

(d) A cos(7 ln |t|) + B sin(7 ln |t|)

52 53 54

(a) 5et + c(t 1 + et ), where c is arbitrary, (b) tet .

(a) A (t + 4)3 + B (t + 4)2 , (b) A cos( ln |t + 2|) + B sin( ln |t + 2|). cos( 2 ln |t|) [A + B ln |t|] + sin( 2 ln |t|) [C + D ln |t|].

(e) t[A+B cos(13 ln |t|)+C sin(13 ln |t|)] (f) A + B ln |t| + C(ln |t|)2

118

Tutorial Problems

Linear Dierential Equations

55 57

(a) x = A(1 t)et + B(1 + t)et , (b) x = A cosh t + B sinh t + Ct, or also x = M et + N et + Ct. (a) x = et , y = et . (b) x = 1 et + 1 e3t , y = 1 et + 5 e3t . 2 2 2 2
3 9 (c) x = 2 e2t + 7 e4t , y = 2 e2t + 7 e4t . 2 2 (d) x = 2 2(1 + t)et , y = 2 t 2(1 + t)et . (a) x = e2t (1 10t), y = 5e2t 4e3t , z = (10t 1)e2t + 2e3t .

58 59 60

(a) x(t) = + cosh t) 1 (b) x(t) = 1 + 2t + 1 t2 + 3 t3 2 x(t) = 2(1 t)e ,


t

1 (b) x = 1 + 2 tet + et , 1 2 (cos t

1 y = 1 et 6 e2t , 6

z =2+

1 2

sinh t.

(c) x(t) = 1 1 t2 2 2 (d) x(t) = 3 sinh t +

2 6

sin

2t

y(t) = (1 t)e

Nonlinear Dierential Equations

119

Chapter Four

NONLINEAR DIFFERENTIAL EQUATIONS

1. Nonlinearity Chapter 3 of these notes has been a long introduction to linear ODEs. We conclude now with a discussion of some nonlinear equations. In the words of S. Ulam, to speak of nonlinear science is like calling zoology the study of nonelephant animals. Its a famous comment, its point being that linearity is the exception, not the rule: we live in a nonlinear world but we tend to forget it, perhaps because our mind is primed to think linearly. Examples are everywhere. Simple harmonic motion, which is a corner stone of an engineers education, is based on the false premise that linear springs exist. Every spring will deviate from Hookes law, if we push it too far or pull it too much. The reasons why most textbooks insist so much on classic harmonic motion are, rst of all, the elegance of its theory, but also (perhaps mostly) because nobody can solve anharmonic motion equations, in general, in a simple form. In thermodynamics you learnt that the internal energy of an ideal gas is given by the equation U = CV T , where CV is constant: another classic linear lawbut ideal gases dont exist. Too bad. The laws of friction and Newtons law of collision, which you saw in rst year, are also good examples of nonlinear phenomena that are treated as linear because an exact theory would be too complicated. From the days of Newton to the XX century, scientists and engineers used linearized models and linear equations whenever this approximation could be justied. The explosive growth of computers after 1975 marked a change of attitude, saw the birth of new disciplines, such as chaos theory or catastrophe theory, and gave new life to old ones, like numerical analysis. In a very short time, nonlinear mathematics grew into one of the most fertile research elds. However, in this chapter well see only some simple nonlinear ODEs that may be solved exactly by the methods of classical analysis, i.e., the calculus you studied in rst year. Well also stay away from Lies theory, which an engineer could nd useful, but would lead us too far away. Actually, you have already encountered a nonlinear equation, the Bernoulli equation of problems 4243, tucked away among linear equations where it didnt belong. But that was because the Bernoulli equation is so similar to the rst-order linear, that it would be wasteful to study it separately. Stanislaw M. Ulam (1909-1984), Polish mathematician; invented the Monte-Carlo method for solving mathematical problems using statistical sampling.

120

Nonlinear Dierential Equations

Well consider now three important types of nonlinear ODEs. Last comment before we start: we switch to the notation where the variables are x and y, rather than x and t. This is because with nonlinear equations it is better to give both variables the same status (without specifying which one is dependent and which is independent), whereas the x-t notation is used almost always when x depends on t. 2. Exact and Quasi-Exact Dierential Equations In rst year you were introduced to implicit functions dened by an expression of the form F (x, y) = constant. For example, the standard equation of the ellipse, y2 x2 + 2 = 1, a2 b is implicit. It denes not one but two explicit continuous functions y(x): y=b 1 x2 /a2 and y = b 1 x2 /a2 .

upper half The rst one represents the upper half of the ellipse, the second one the lower half. Similarly, if C is a positive parameter, the equation y2 x2 + 2 =C a2 b represents a family of ellipses with center at the origin, increasing in size as C gets larger. lower half In many applications an implicit solution is all one can obtain in a simple, direct way. The implicit function theorem asserts that the equation
F (x, y) = constant implicitly denes a function y(x) in the vicinity of a point (x0 , y0 ) under the assumption that Fx and Fy are continuous functions of x and y, and Fy = 0 at that point; furthermore Fx dy = . dx Fy The easy way to remember this is by means of dierentials: since F = constant, then the variation of F is zero along the graph of y(x); hence, we write dF = F F dx + dy = 0, x y

and then, formally dividing by dx, we get the formula for dy/dx.

Nonlinear Dierential Equations

121

Obviously x and y may be interchanged, and the corresponding result for x(y) is dx Fy = , dy Fx with the assumption, now, that Fx = 0. These formulas may be interpreted backwards. Consider a dierential equation of the form P (x, y) + Q(x, y) y (x) = 0 or P (x, y) x (y) + Q(x, y) = 0;

we agree that either equation may also be written in the symmetric form P (x, y) dx + Q(x, y) dy = 0, (50)

which is equivalent to both, wherever the assumptions of the implicit function theorem are satised. Now suppose that there exists a function F (x, y) such that F =P x then the formal manipulations P dx + Q dy = 0 = F F dx + dy = 0 x y = dF = 0 = F =C and F =Q: y (51)

(where C = constant) yield the solution in implicit form. Example 144 Find the solution of y + (4x3 y 2 2x)/2x4 y = 0. Solution: First of all, put this equation in the form (50): (4x3 y 2 2x) dx + 2x4 y dy = 0. Then, note that 4x3 y 2 2x = (x4 y 2 x2 ) x and 2x4 y = (x4 y 2 x2 ) ; y

conditions (51) are satised by the function F (x, y) = x4 y 2 x2 , hence the (implicit) solution is x4 y 2 x2 = C, where C = constant. In this example one may go one step further, and write the explicit solutions: C + x2 , y= x2 but in general this last step may not be possible. Denition: An equation of the form P dx + Q dy = 0 for which a function F (x, y) exists so that conditions (51) are satised, is called exact.

122

Nonlinear Dierential Equations

We note immediately that if an equation is exact, then by N. Bernoullis theorem on mixed partial derivatives, we have: 2F 2F = x y y x = P Q = . y x

Hence, if the last condition is not satised, then there is no point in looking for a function F satisfying conditions (51), because such a function cannot exist. So, the mixed-derivatives condition Q P = , (52) y x is necessary for an equation of the form (50) to be exact. Moreover, if P and Q, besides satisfying (52), are also continuous, then it is not hard to show that this condition is also sucient: in other words, a function F exists such that (51) hold, and the equation is exact. Lets see how this comes to work in practice. Example 145 Check that the equation (2xyex 2x) dx + (ex + 3y 2 ) dy = 0 is exact, and solve it. Solution: First of all, we check for exactness. Clearly P and Q are continuous, and
2 2 P = 2xyex 2x = 2xex , y y 2 x2 Q = e + 3y 2 = 2xex : x x 2 2

hence (52) holds, and the equation is exact. Now, to nd F (x, y) we may proceed as follows. We note that 2 F = P = 2xyex 2x; x integrating this equation with respect to x (i.e., treating y as a parameter) we get: F (x, y) = (2xyex 2x) dx = yex x2 + a constant.
2 2

Careful now: a constant here means any expression that does not depend on x, because any function of y alone remains constant if only x is varied. So, we rewrite the last result as F (x, y) = yex x2 + g(y),
2

(53)

where g(y) is a function of y alone. In this way, weve made sure that the rst of conditions (51) is satised, but g is still undetermined. To nd it, we require that the second of conditions (51), i.e., F/y = Q, is also satised. It follows that
2 2 yex x2 + g(y) = ex + 3y 2 , y

ex + g (y) = ex + 3y 2 , g (y) = 3y 2 . Nicholas Bernoulli (1687-1759), a Swiss who lectured at Padua (Italy); not to be confused with his cousin Nicholas Bernoulli (1695-1726), who discovered the St. Petersburg paradox. The Bernoulli dierential equation, which you saw in problems 4243, is named after their uncle Jakob Bernoulli (1654-1705).

Nonlinear Dierential Equations

123

We see that the last equation does not contain x anymore. Hence, by a trivial integration with respect to y, we get g(y) + C = y 3 , where C is a true constant. Combining our results, we nally get that F (x, y) = yex x2 + y 3 = C, and this is the general solution in implicit form.
2

As a rule, after one has established that a given equation is exact, there are several avenues for nding F (x, y), all pretty much equivalent. The procedure used in example 145 went through three steps: rst, integrate P with respect to x, up to an unknown function g(y). Second, take the partial derivative of this integral with respect to y, and compare the result with Q; the resulting equation must contain only y (if it doesnt, its a ag for a mistake in the calculations). Third, integrate g (y) to nd g(y). Alternatively, one may integrate Q with respect to y, up to an unknown function g(x), and then take the partial derivative of this integral with respect to x, comparing the result with P . The resulting equation must contain only x, and one may therefore nd g(x) by integrating g (x). This procedure is the mirror image of the rst one. One may also integrate P with respect to x, up to a function g(x), and then Q with respect to y, up to a function h(y). Comparing the two results, which must be equivalent, one determines g(x) and h(y). Example 146 Solve by comparison the equation of example 145. Solution: Having already found with equation (53) that F (x, y) = yex x2 + g(y), we integrate Q with respect to y; this yields: F (x, y) = (ex + 3y 2 ) dy = yex + y 3 + h(x).
2 2 2

Comparing the two expressions above for F , we deduce that h(x) = x2


2

and

g(y) = y 3 . x2 + y 2 x dy = 0

Therefore, F (x, y) = yex x2 + y 3 , as expected. Example 147 Check that the equation x x2 + y 2 x y dx + y is exact and nd the general solution. Solution: Clearly P and Q are continuous, and Py = x y x2 + y 2 x y = xy x2 + y2 1, Qx = y x

x2 + y 2 x =

xy x2 + y 2

1 :

hence the equation is exact. Integrating Q with respect to y, we nd: F (x, y) = y x2 + y 2 x dy =


1 3

(x2 + y 2 )3/2 xy + g(x),

124

Nonlinear Dierential Equations

where g(x) depends on x alone. Dierentiating this result with respect to x, and comparing the result with P , we get: x
1 3

(x2 + y 2 )3/2 xy + g(x) = x

x2 + y 2 x y

x x2 + y 2 y + g (x) = x x2 + y 2 x y g (x) = x. The last line contains x alone, hence by integration we nd that
1 g(x) + C = 2 x2 ,

which nally yields F (x, y) =


1 3

(x2 + y 2 )3/2 xy 1 x2 = C. 2

This is the general solution in implicit form.

Exact dierential equations are closely connected to conservative vector elds. The link becomes clear if one considers a two-dimensional vector eld f dened as f = P + Q , where P = P (x, y) and Q = Q(x, y). Then we know that f is conservative if f = 0, i.e., (Qx Py ) k = 0;

the last condition is equivalent to (52). Furthermore, if f physically corresponds to a force, then the expression P dx + Qdy represents the innitesimal work done by f on a particle moving from (x, y) to (x + dx, y + dy); the solutions of equation (50) P dx + Q dy = 0, which are given by F (x, y) = C, are curves along which f does no work. Finally: since f = P + Q = F , then F is the potential energy associated with the force f . INTEGRATING FACTORS An unpleasant feature of exact equations is that if they are altered in an insignicant way, they may lose their exactness. For example, consider the equation y = y, which has the general solution y = Cex . If written dx dy =0 y

Nonlinear Dierential Equations

125

it is exact, because P = 1, Q = 1/y, and Py = Qx = 0. But if written y dx dy = 0, which is clearly equivalent, it is not exact because P = y, Q = 1, Py = 1, Qx = 0, and Qx = Py : the mixed-derivatives condition (52) does not hold. Whats more disturbing, if one multiplies through the equation above by y 1 , which at rst sight looks like a pointless move, y 1 (y dx dy) = 0, one obtains the exact equation again. This example is purely academic because the equation y = y is so easy that we can solve it by inspection; but the point is important. It may be shown that any equation of the form (50) may, under reasonable assumptions, be made exact if its multiplied through by an appropriate function (x, y). Unfortunately, the same theorem that guarantees the existence of does not say a word about how to nd it. Denition: A function (x, y) such that an equation of the form P dx + Q dy = 0 becomes exact if it is multiplied through by , is called an integrating factor for the equation. Example 148 Show that (x, y) = x is an integrating factor for the equation (2y 3x) dx + x dy = 0, and solve it. Solution: The equation (2y 3x) dx + x dy = 0 is not exact because Qx = 1 and Py = 2. However, if its multiplied through by x, one gets (2xy 3x2 ) dx + x2 dy = 0. Now, Qx = 2x, Py = 2x, and the equation has become exact; hence = x is an integrating factor. We proceed by integrating Q with respect to y: F (x, y) = x2 y + g(x), where g is determined by the rst of conditions (51): Fx = 2xy + g (x) = P = 2xy 3x2 . It follows that g (x) = 3x2 , g(x) + C = x3 , and nally that F (x, y) = x2 y x3 = C. Note that in this example one can write an explicit solution in the form y = C/x2 + x. Example 149 Show that the equation (xy2y 2 ) dx+(3xyx2 ) dy = 0 admits the integrating factor (x, y) = 1/xy 2 ; nd the general solution. Solution: The equation (xy 2y 2 ) dx + (3xy x2 ) dy = 0

is not exact because Py = x 4y and Qx = 3y 2x. However, if its divided through by xy 2 , one gets 3xy x2 xy 2y 2 dx + dy = 0 xy 2 xy 2 1 2 x 3 2 dy = 0. dx + y x y y

126

Nonlinear Dierential Equations

Written in this form, the equation is exact because Py = 1/y 2 and Qx = 1/y 2 . Integrating P with respect to x, we get: F (x, y) = 2 1 y x dx = x 2 ln |x| + g(y), y

where g(y) is determined by the second of conditions (51). It follows that: x x 3 + g (y) = 2 , y2 y y

and hence that g (y) = 3/y, g(y) + C = 3 ln |y|, and nally that x 2 ln |x| + 3 ln |y| = C. y

Example 150 Show that the equation (x2 + 2x + y) dx + (1 x2 y) dy = 0 admits the integrating factor (x, y) = exy ; nd the general solution. Solution: The equation (x2 + 2x + y) dx + (1 x2 y) dy = 0 is not exact because Py = 1 and Qx = 2x. However, if its multiplied through by exy , one gets exy (x2 + 2x + y) dx + exy (1 x2 y) dy = 0; now, by the product rule, we obtain: Py = exy (x2 + 2x + y) + exy 1 = exy (1 x2 2x y)

Qx = exy (1 x2 y) + exy (2x) = exy (1 x2 2x y)

and the equation is exact. Integrating (by parts) Q with respect to y, we get that F (x, y) = exy (y + x2 ) + g(x), where g is determined by the rst of (51): Fx = exy (y + x2 ) + exy 2x + g (x) = exy (x2 + 2x + y). It follows that g (x) = 0, g = C, and nally that exy (y + x2 ) = C.

Example 151 The rst principle of thermodynamics says that if heat enters a system at a rate Q, then Q = U + W, where U is the rate at which the internal energy increases, and W is the rate at which the system is doing work. (These quantities have the same dimensions as power, so they could be measured in Watt.) Integrating with respect to time, we get Q = U + W,

Nonlinear Dierential Equations

127

which is an equivalent (and perhaps more familiar) statement of the rst principle. The disadvantage of this notation is that, since the time interval of integration may be made arbitrarily small, one is tempted to write ? dQ = dU + dW. But this is wrong, because dQ is not an exact dierential. There is no guarantee that there is a state function Q such that its dierential is equal to dU + dW . It may be shown, however, that an integrating factor for dQ always exists, and that it is equal to 1/T . This, in turn, means that the path integral S= dU dW + T T (54)

depends only on the initial and nal states of the system, and not on the type of transformation (such as adiabatic, isobaric, isothermal, etc) that connects themas long as it is a reversible transformation, obviously. For example, consider an ideal gas: you have probably seen in your physics courses that the internal energy of an ideal gas is U = CV T , where T is the absolute temperature, and CV is a constant. You also know that the work done by an expanding gas is W = p dV , where p and V are pressure and volume, respectively. Substituting back, we get dQ = CV dT + p dV ; but remember that p depends on T and V through the equation of state which, for an ideal gas, is p = nRT /V . It follows that nRT ? dV, dQ = CV dT + V and we see immediately that the right-hand side is not exact: from the exactness condition (52), we get CV nRT nR = 0, , = V T V V and the two are not equal. Hence, there is no function Q(T, V ) such that (Q/T )V = CV and (Q/V )T = nRT /V . If however we multiply dQ by the integrating factor 1/T and then integrate, we get (54), which for an ideal gas becomes S= where CV nRT dT + dV T VT S T CV , T
2 ?

= CV ln T + nR ln V + constant,

=
V

S V

=
T

nR . V

The exactness condition (52) holds because both S/V T and 2 S/T V are = 0. The principle of existence of the integrating factor 1/T is mathematically equivalent to the second principle of thermodynamics. However, the proof of this fairly advanced concept (Carathodorys principle) is beyond the scope of these notes; interested students can nd a brief e description in H. Margenau and G.M. Murphy, The Mathematics for Physics and Chemistry, Krieger (USA) 1976.

128

Nonlinear Dierential Equations

QUASI-EXACT EQUATIONS If you thought that there was practically no way of guessing the integrating factors in the last examples, you have a point. As we saw, the theorem that asserts the existence of integrating factors does not provide any method for nding them; for a generic dierential equation of the form (50) we have to live with that. There are exceptions, though. Among them, well consider only the class of quasi-exact equations: these are equations that admit an integrating factor that depends on one variable only, but not on the other. For quasi-exact equations there is a simple procedure for nding . Supposefor the sake of the argumentthat the integrating factor for the equation (50) depend on x alone, i.e., assume that = (x). In other words, assume the equation (x) P (x, y) dx + (x) Q(x, y) dy = 0 is exact. Then,by the product rule, (52) yields: ( P ) ( Q) = y x Py = Q + Qx , where = d/dx. Simple manipulations lead to the equation Py Qx = . Q Recall, we assumed that depends only on x; hence, the left-hand side above depends on x alone. Therefore, if the right-hand side depends on x and y, the method fails. If, however, (Py Qx )/Q depends on x alone, then we may nd by integration: ln |(x)| = Py Qx dx. Q

Make sure you understand this point: the integration of the right-hand side is meaningful only if (Py Qx )/Q depends on x alone. In exactly the same way, one may see that if (Qx Py )/P depends on y alone, then an integrating factor is given by the equation ln |(y)| = Qx Py dy. P

In summary, an equation is quasi-exact if either (Py Qx )/Q depends on x alone, or (Qx Py )/P depends on y alone. In the rst case, = (x), and in the second case = (y). Example 152 Find the integrating factor of example 148. Solution: The equation (2y 3x) dx + x dy = 0 is quasi-exact because 21 Py Qx = Q x depends only on x. Hence, the integrating factor is given by ln || = dx = ln |x| + C, x

Nonlinear Dierential Equations

129

which yields = x, as expected (the integration constant is irrelevant).

You should convince yourself that the integration constant arising in the calculation of may always be set equal to zero without loss of generality. Therefore, for the rest of this section well ignore it. Example 153 Find an integrating factor of the equation (xy 2 ) dx+2xy dy = 0, and solve it. Solution: The equation is quasi-exact because Py Qx 2y 2y 2 = = ; Q 2xy x therefore ln || = 2 dx = ln x2 . x

An integrating factor is = x2 ; dividing the original equation by x2 , we get: 1 y2 2 x x dx + 2y dy = 0, x

which is exact. Integrating Q with respect to y, we nd that F (x, y) = y2 2y dy = + g(x), x x

where g(x) is determined by the rst of conditions (51): Fx = y2 1 y2 + g (x) = 2 . x2 x x

It follows that g (x) = 1/x, g(x) + C = ln |x|, and nally y 2 /x + ln |x| = C, which is the general solution. Example 154 Find an integrating factor of the equation (2y + 3y 2 ex ) dx + (1 + 2yex ) dy = 0, and solve it. Solution: The equation is quasi-exact because 2 + 6yex 2yex Py Qx = = 2; Q 1 + 2yex therefore ln || = 2 dx = 2x.

An integrating factor is = e2x ; multiplying the original equation by e2x we get: (2ye2x + 3y 2 e3x ) dx + (e2x + 2ye3x ) dy = 0, which is exact. Integrating P with respect to x we nd that: F (x, y) = (2ye2x + 3y 2 e3x ) dx = ye2x + y 2 e3x + g(y),

130

Nonlinear Dierential Equations

where g(y) is determined by the second of conditions (51): Fy = e2x + 2ye3x + g (y) = e2x + 2ye3x . It follows that g (y) = 0, g(y) = C, and nally that e2x + 2ye3x = C, which is the general 1 solution. This one may be made explicit: y = 1 Ce3x 2 ex . 2 Example 155 Find an integrating factor of the equation y dx (x + 1 y 2 ) dy = 0, and solve it. Solution: We note immediately that Py Qx = 1 (1) = 2; unfortunately Py Qx 2 = , Q (x + 1 y 2 ) 2 Qx Py = P y is a function of y alone. Therefore, there is an integrating factor of the form = (y), given by the equation (y) 2 = . (y) y A simple integration yields ln || = ln |y|2 , i.e., = y 2 . Dividing the original equation by y 2 , we get: x 1 dx + 2 1 dy = 0, 2 y y y which is exact. Integrating P with respect to x we nd that F (x, y) = Integrating Q with respect to y we nd that F (x, y) = By comparison, we see that g(y) = hence 1 +y y and h(x) = 0; x 1 + + y + h(x). y y x + g(y). y

which certainly is not a function of x alone. However, observe that P = y: therefore,

x 1 + +y =C y y

is the general solution. In explicit form, this may be written x = Cy y 2 1.

Nonlinear Dierential Equations

131

3. Scale-Invariant Equations If an equation retains the same form when x and y are scaled by the same amount, it is called scale-invariant. A rst-order scale-invariant equation is easy to recognize when its written in the form y = f (y/x), where f (y/x) is some function of the ratio y/x. It is rather unfortunate that most books call these equations homogeneous, a choice that can be confusing for a beginner since they have nothing to do with the homogeneous equations we met in chapter 3. The name scale-invariant has been suggested, among others, by Bender and Orszag. If a rst-order equation is scale-invariant, then either of the substitutions u= y x or u= x y

will transform it into a separable equation. Example 156 Show that the dierential equation (2xy y 2 ) dxx2 dy = 0 is scale-invariant, and solve it. Solution: If one introduces the scaled variables x = aX then the equation becomes a2 (2XY Y 2 ) a dX a2 X 2 a dY = 0, which (after canceling the common factor a3 ) has the same form as the original. Note also that the equation may be written as 2xy y 2 2(y/x) (y/x)2 dy = , = dx x2 1 which shows immediately the scale-invariance property. Method 1: Substitute u= y x = y = xu, y = u + xu , y = aY,

where u = u(x) is the new unknown. It follows that u + xu = 2u u2 dx du = u(1 u) x 1 1 dx ; du = u u1 x Advanced Mathematical Methods for Scientists and Engineers, McGraw-Hill (1978).

132

Nonlinear Dierential Equations

integrating, we get that

ln

where B = eC is a new constant replacing C, and nally that y = Bx(y x). Method 2: Substitute u= x y = x = yu, x = u + yu ,

u = ln |x| + C, u1 y/x = Bx, y/x 1

where u = u(y) is the new unknown. It follows that dx (x/y)2 = dy 2(x/y) 1 u2 . u + yu = 2u 1 From here onwards, follow the steps of method 1 (separate the variables, use partial fractions, integrate with respect to y). Do it, as an exercise. You may also observe that the original equation becomes a Bernoulli equation if formally divided by dx: x dy/dx 2y = y 2 /x. See problem 42. Example 157 Solve the equation xy = y + x2 + y 2 . Solution: That the equation is scale-invariant is evident when its rewritten as follows: y = y+ x2 + y 2 y = + x x 1+ y2 . x2

Substituting y = xu, y = u + xu , one gets immediately: u + xu = u + du dx = . x 1 + u2 Hence, after integration: arsinh u = ln |x| + C, earsinh u = Bx, where B = eC is a new constant replacing C. Using the trivial identities earsinh u = cosh(arsinh u) + sinh(arsinh u) = one nally gets: 1 + y 2 /x2 + y/x = Bx, x2 + y 2 + y = Bx2 . Example 158 Solve the equation xy = y(ln y ln x). 1 + u2 + u, 1 + u2 ,

Nonlinear Dierential Equations

133

Solution: Rewriting the equation as y = (y/x) ln(y/x) (note the scale-invariance) and substituting y = xu, y = u + xu , one gets immediately: u + xu = u ln u. Hence, xu = u (ln u 1) du dx = . u (ln u 1) x

Integrating, we get:

du = ln ln u 1 = ln |x| + C, u (ln u 1) and then (introducing a new constant B that may take negative values): It follows that ln(y/x) = 1 + Bx, and nally that y = xe1+Bx . 4. Autonomous Equations An equation is called autonomous if the independent variable is missing. If the equation is of second order, the substitution p = dy/dx, combined with the chain rule, transforms it into a rst-order equation where p(y) is the unknown: ln u 1 = Bx.

dy d2 y dp dp dy dp =p = = = = p. (55) 2 dx dx dx dy dx dy The best-known autonomous equation is almost certainly the simple harmonic motion equation: x + 2 x = 0, or (with the notation of this chapter) y + 2 y = 0. Example 159 Solve the harmonic motion equation without using the methods of chapter 3. Solution: Following (55), we let y = p and hence y = p dp/dy. It follows: p dp = 2 y, dy y dy.

p dp = 2 A simple integration yields: C = 1 2 A2 , we get 2


1 2 2p

= C 1 2 y 2 . Dening a new constant A through the equation 2 p= dy dy = || A2 y 2 dx

= dx; A2 y 2 in the last step, is allowed negative values. Another simple integration gives (where = constant), which nally yields the classic solution y = A cos(x + ). Example 160 Solve the autonomous equation y 3 y = 1. Solution: By means of the substitution (55), we get: arccos y/A = x +

134

Nonlinear Dierential Equations

y3p

dp = 1, dy y 3 dy.

p dp = Integrating, we obtain:
1 2 p 2

1 1 = 1 y 2 + C = 2 y 2 + 2 a2 , 2

where a is a constant replacing C. It follows that p= Integrating again, we get that where b is another constant, and nally that a2 y 2 1 = (a2 x + b)2 . Example 161 Solve the autonomous equation y y = y + y 2 . Solution: By (55), we get: dp = p2 + y 2 . yp dy This is a scale-invariant equation. Proceeding along the lines of examples 157158 (but bearing in mind that the variables are now p and y), we rst rewrite the last equation as follows: p y dp = + dy y p (which shows the scale invariance). We then introduce a new variable u: p = uy and substitute back into (56): 1 du y+u=u+ . dy u Simplifying, we get: u du = dy y 1 2 2 u = ln |y| + C 2 ln |y| + 2C 2 ln |y| + 2C. = du dp = y + u, dy dy (56)
2

dy = dx =

a2 y 2 dx.

y dy a2 y 2 1

a2 y 2 1 = a2 x + b,

|u| = p = y

But p = dy/dx; substituting back and rearranging, we get: y dy = 2 ln |y| + 2C dx.

Nonlinear Dierential Equations

135

Integration is trivial; it follows that 2 ln |y| + 2C = x + B, and hence that 2 ln |y| + 2C = (x + B)2 =

= x2 + 2Bx + B 2 .

An explicit solution is obtained as follows:


1 ln |y| = 2 x2 + Bx + 1 B 2 C, 2

y = exp( 1 x2 + Bx + 1 B 2 C), 2 2 y = A exp( 1 x2 + Bx) 2

where A = exp( 1 B 2 C) is a new (possibly negative) constant. 2 Example 162 Solve the dierential equation (1 + y 2 )y yy = 0. Solution: Substitution (55) gives: (1 + y 2 )p dp = yp2 . dy
2

Therefore, either p = 0, hence y = 0, y = constant; or, dividing through by p, it follows: (1 + y 2 ) Separating the variables, one gets: dp = yp. dy

y dy dp = . p 1 + y2

A straightforward integration yields: ln |p| = hence p=


1 2

ln |1 + y 2 | + C, 1 + y 2 eC .

dy = dx dy 1 + y2

Dening A = eC , and separating again the variables, one nds that = A dx.

One more integration nally yields: arsinh y = Ax + B, which may also be written y = sinh(Ax + B). Note that the explicit solution of this equation can be written both in the form x = x(y) and y = y(x).

136

Tutorial Problems

Nonlinear Dierential Equations

PROBLEMS

Exact ODEs 61. Check that the following ODEs are exact and hence nd the general solution. (a) (3e3x y 2x) dx + e3x dy = 0 (b) (6x5 y 3 + 4y 5 x3 + 21x2 ) dx + (3x6 y 2 + 5x4 y 4 10y) dy = 0 (c) (cos x + cos y) dx x sin y dy = 0 (d) (1 + yexy ) dx + (2y + xexy ) dy = 0 Integrating Factors 62. Solve the following equations, using the suggested integrating factors. (a) (3y + 4xy 2 ) dx + (2x + 3x2 y) dy = 0, = x2 y 2 2 (b) y dx + x(1 + 3x y ) dy = 0, = 1/(xy)3 (c) y dx (y 5 + y 3 x2 + x) dy = 0, = 1/(x2 + y 2 ) 63. Show that the rst order linear equation c1 (t) dy + c0 (t) y dt = f (t) dt [go back to (40) in section 3.6] admits the integrating factor (t) = exp (c0 /c1 ) dt . c1

Many books present this as the best method for solving rst order linear ODEs. Quasi-Exact Equations 64. The following equations admit an integrating factor that depends on x alone or y alone. Find it, and hence solve the equation. (a) (x2 xy + y 3 + 2x y) dx + (3y 2 x) dy = 0 (b) (x4 + y 4 ) dx xy 3 dy = 0 (c) 7xy dx + (x2 + y 2 + y) dy = 0 (d) (y + eyx ) dx + (1 + xeyx ) dy = 0 Scale Invariant Equations 65. Find the general solution of the following equations. (a) xy = y x sin(y/x) (b) xy y = x4 + y 4 + y 2 (c) y = (x + yex/y ) y (d) (xy + y 2 ) dx (xy x2 ) dy = 0 (e) y 2 dx + (x2 xy y 2 ) dy = 0 (f) 4y = (x/y)3 + 5(y/x) (g) xy x2 cos(y/x) = y 2 /y

Nonlinear Dierential Equations

Tutorial Problems

137

Autonomous Equations 66. Find the general solution of the following equations. 3 (a) y 2 y = y 2 (b) yy + y = yy 2 (c) y = 1/ y 2 (d) y = y tan y 2 (e) y = (ey 1)y (f) y = y 3 y ANSWERS 61 (a) e3x y x2 = C (b) x6 y 3 + x4 y 5 + 7x3 5y 2 = C (c) x cos y + sin x = C (d) x + y 2 + exy = C (a) x3 y 2 + x4 x3 = C (b) 3 ln |y| 1/2x2 y 2 = C (c) arctan(y/x) 1 y 4 = C 4

62

64

65

66

(a) = 1/x3 , F (x, y) = ex (x2 xy + y 3 ) = C; (b) = 1/x5 , F (x, y) = ln |x| y 4 /4x4 = C; 7 7 (c) = y 5/7 , F (x, y) = 7 xy 2/7 + 16 y 16/7 + 9 y 9/7 = C; 2 x x y (d) = e , F (x, y) = ye + xe = C. (a) cos y/x = 1 2/(Bx2 + 1), where B > 0. (b) B 2 x4 = 2By 2 + 1, where B > 0. (c) x = y ln ln |y| + C . (d) xy = Cey/x . (e) (x y)y 2 = C(x + y). (f) y = 4 |Ax5 x4 |. (g) sin(y/x) = (A2 y 2 1)/(A2 y 2 + 1). (a) x = B + Cy + ln |y|. (b) y + 1 = (Ax + B)ey . 3/2 1/2 2 (c) x = B + 3 y + C 2C y + C . (d) sin y + B = Cx. (e) exp(ey ) = Ax + B. (f) y 2 = A2 A4 1 sin(2x + B), where |A| 1.

You might also like